Формулы давления жидкости: Недопустимое название — Викиверситет

Формулы давления жидкости: Недопустимое название — Викиверситет

Содержание

Сила, Давление — Формулы по физике

По
рыхлому снегу человек идёт с большим трудом, глубоко проваливаясь при
каждом шаге. Но, надев лыжи, он может идти, почти не проваливаясь в
него. Почему? На лыжах или без лыж человек действует на снег с одной и
той же силой, равной своему весу. Однако действие этой силы в обоих
случаях различно, потому что различна площадь поверхности, на которую
давит человек, с лыжами и без лыж. Площадь поверхности лыж почти в 20
раз больше площади подошвы. Поэтому, стоя на лыжах, человек действует на
каждый квадратный сантиметр площади поверхности снега с силой, в 20 раз
меньшей, чем стоя на снегу без лыж.

Ученик,
прикалывая кнопками газету к доске, действует на каждую кнопку с
одинаковой силой. Однако кнопка, имеющая более острый конец, легче
входит в дерево.

Значит,
результат действия силы зависит не только от её модуля, направления и
точки приложения, но и от площади той поверхности, к которой она
приложена (перпендикулярно которой она действует).

Этот вывод подтверждают физические опыты.

Опыт.Результат действия данной силы зависит от того, какая сила действует на единицу площади поверхности.

По
углам небольшой доски надо вбить гвозди. Сначала гвозди, вбитые в
доску, установим на песке остриями вверх и положим на доску гирю. В этом
случае шляпки гвоздей лишь незначительно вдавливаются в песок. Затем
доску перевернем и поставим гвозди на острие. В этом случае площадь
опоры меньше, и под действием той же силы гвозди значительно углубляются
в песок.

Опыт. Вторая иллюстрация.

От того, какая сила действует на каждую единицу площади поверхности, зависит результат действия этой силы.

В
рассмотренных примерах силы действовали перпендикулярно поверхности
тела. Вес человека был перпендикулярен поверхности снега; сила,
действовавшая на кнопку, перпендикулярна поверхности доски.

Величина, равная отношению силы, действующей перпендикулярно поверхности, к площади этой поверхности, называется давлением.

Чтобы определить давление, надо силу, действующую перпендикулярно поверхности, разделить на площадь поверхности:

давление = сила / площадь.

Обозначим величины, входящие в это выражение: давление — p, сила, действующая на поверхность, — F и площадь поверхности — S.

Тогда получим формулу:

p = F/S

Понятно, что бóльшая по значению сила, действующую на ту же площадь, будет производить большее давление.

За единицу давления принимается такое давление, которое производит сила в 1 Н, действующая на поверхность площадью 1 м2 перпендикулярно этой поверхности.

Единица давления — ньютон на квадратный метр ( 1 Н / м2 ). В честь французского ученого Блеза Паскаля она называется паскалем (Па). Таким образом,

1 Па = 1 Н / м2 .

Используется также другие единицы давления: гектопаскаль (гПа) и килопаскаль (кПа).

1 кПа = 1000 Па;

1 гПа = 100 Па;

1 Па = 0,001 кПа;

1 Па = 0,01 гПа.

Пример.
Рассчитать давление, производимое на пол мальчиком, масса которого 45
кг, а площадь подошв его ботинок, соприкасающихся с полом, равна 300 см2.

Запишем условие задачи и решим её.

Дано: m = 45 кг, S = 300 см2; p = ?

В единицах СИ: S = 0,03 м2

Решение:

p = F/S,

F = P,

P = g·m,

P = 9,8 Н · 45 кг ≈ 450 Н,

p = 450/0,03 Н / м2 = 15000 Па = 15 кПа

‘Ответ’: p = 15000 Па = 15 кПа

Способы уменьшения и увеличения давления.

Тяжелый
гусеничный трактор производит на почву давление равное 40 — 50 кПа, т.
е. всего в 2 — 3 раза больше, чем давление мальчика массой 45 кг. Это
объясняется тем, что вес трактора распределяется на бóльшую площадь за
счёт гусеничной передачи. А мы установили, что чем больше площадь опоры, тем меньше давление, производимое одной и той же силой на эту опору.

В
зависимости от того, нужно ли получить малое или большое давление,
площадь опоры увеличивается или уменьшается. Например, для того, чтобы
грунт мог выдержать давление возводимого здания, увеличивают площадь
нижней части фундамента.

Шины
грузовых автомобилей и шасси самолетов делают значительно шире, чем
легковых. Особенно широкими делают шины у автомобилей, предназначенных
для передвижения в пустынях.

Тяжелые
машины, как трактор, танк или болотоход, имея большую опорную площадь
гусениц, проходят по болотистой местности, по которой не пройдет
человек.

С
другой стороны, при малой площади поверхности можно небольшой силой
произвести большое давление. Например, вдавливая кнопку в доску, мы
действуем на нее с силой около 50 Н. Так как площадь острия кнопки
примерно 1 мм2, то давление, производимое ею, равно:

p = 50 Н/ 0, 000 001 м2 = 50 000 000 Па = 50 000 кПа.

Для
сравнения, это давление в 1000 раз больше давления, производимого
гусеничным трактором на почву. Можно найти еще много таких примеров.

Лезвие
режущих и острие колющих инструментов (ножей, ножниц, резцов, пил, игл и
др.) специально остро оттачивается. Заточенный край острого лезвия
имеет маленькую площадь, поэтому при помощи даже малой силы создается
большое давление, и таким инструментом легко работать.

Режущие
и колющие приспособления встречаются и в живой природе: это зубы,
когти, клювы, шипы и др. — все они из твердого материала, гладкие и
очень острые.

Давление

Известно, что молекулы газа беспорядочно движутся.

Опыт. Здесь мы узнаем, что газ давит на стенки сосуда по всем направлениям одинаково.

Мы
уже знаем, что газы, в отличие от твердых тел и жидкостей, заполняют
весь сосуд, в котором находятся. Например, стальной баллон для хранения
газов, камера автомобильной шины или волейбольный мяч. При этом газ
оказывает давление на стенки, дно и крышку баллона, камеры или любого
другого тела, в котором он находится. Давление газа обусловлено иными
причинами, чем давление твердого тела на опору.

Известно,
что молекулы газа беспорядочно движутся. При своем движении они
сталкиваются друг с другом, а также со стенками сосуда, в котором
находится газ. Молекул в газе много, поэтому и число их ударов очень
велико. Например, число ударов молекул воздуха, находящегося в комнате, о
поверхность площадью 1 см2 за
1 с выражается двадцатитрехзначным числом. Хотя сила удара отдельной
молекулы мала, но действие всех молекул на стенки сосуда значительно, —
оно и создает давление газа.

Итак, давление газа на стенки сосуда (и на помещенное в газ тело) вызывается ударами молекул газа.

Рассмотрим
следующий опыт. Под колокол воздушного насоса поместим резиновый шарик.
Он содержит небольшое количество воздуха и имеет неправильную форму.
Затем насосом откачиваем воздух из-под колокола. Оболочка шарика, вокруг
которой воздух становится все более разреженным, постепенно раздувается
и принимает форму правильного шара.

Как объяснить этот опыт?

Для хранения и перевозки сжатого газа используются специальные прочные стальные баллоны.

В
нашем опыте движущиеся молекулы газа непрерывно ударяют о стенки шарика
внутри и снаружи. При откачивании воздуха число молекул в колоколе
вокруг оболочки шарика уменьшается. Но внутри шарика их число не
изменяется. Поэтому число ударов молекул о внешние стенки оболочки
становится меньше, чем число ударов о внутренние стенки. Шарик
раздувается до тех пор, пока сила упругости его резиновой оболочки не
станет равной силе давления газа. Оболочка шарика принимает форму шара.
Это показывает, что газ давит на ее стенки по всем направлениям одинаково.
Иначе говоря, число ударов молекул, приходящихся на каждый квадратный
сантиметр площади поверхности, по всем направлениям одинаково.
Одинаковое давление по всем направлениям характерно для газа и является
следствием беспорядочного движения огромного числа молекул.

Попытаемся
уменьшить объем газа, но так, чтобы масса его осталась неизменной. Это
значит, что в каждом кубическом сантиметре газа молекул станет больше,
плотность газа увеличится. Тогда число ударов молекул о стенки
увеличится, т. е. возрастет давление газа. Это можно подтвердить опытом.

На рисунке а изображена
стеклянная трубка, один конец которой закрыт тонкой резиновой пленкой. В
трубку вставлен поршень. При вдвигании поршня объем воздуха в трубке
уменьшается, т. е. газ сжимается. Резиновая пленка при этом выгибается
наружу, указывая на то, что давление воздуха в трубке увеличилось.

Наоборот,
при увеличении объема этой же массы газа, число молекул в каждом
кубическом сантиметре уменьшается. От этого уменьшится число ударов о
стенки сосуда — давление газа станет меньше. Действительно, при
вытягивании поршня из трубки объем воздуха увеличивается, пленка
прогибается внутрь сосуда. Это указывает на уменьшение давления воздуха в
трубке. Такие же явления наблюдались бы, если бы вместо воздуха в
трубке находился бы любой другой газ.

Итак, при
уменьшении объема газа его давление увеличивается, а при увеличении
объема давление уменьшается при условии, что масса и температура газа
остаются неизменными
.

А
как изменится давление газа, если нагреть его при постоянном объеме?
Известно, что скорость движения молекул газа при нагревании
увеличивается. Двигаясь быстрее, молекулы будут ударять о стенки сосуда
чаще. Кроме того, каждый удар молекулы о стенку будет сильнее.
Вследствие этого, стенки сосуда будут испытывать большее давление.

Следовательно, давление газа в закрытом сосуде тем больше, чем выше температура газа, при условии, что масса газа и объем не изменяются.

Из этих опытов можно сделать общий вывод, что давление газа тем больше, чем чаще и сильнее молекулы ударяют о стенки сосуда.

Для
хранения и перевозки газов их сильно сжимают. При этом давление их
возрастает, газы необходимо заключать в специальные, очень прочные
баллоны. В таких баллонах, например, содержат сжатый воздух в подводных
лодках, кислород, используемый при сварке металлов. Конечно же, мы
должны навсегда запомнить, что газовые баллоны нельзя нагревать, тем
более, когда они заполнены газом. Потому что, как мы уже понимаем, может
произойти взрыв с очень неприятными последствиями.

Закон Паскаля.

Давление передается в каждую точку жидкости или газа.

Давление поршня передается в каждую точку жидкости, заполняющей шар.

Теперь газ.

В
отличие от твердых тел отдельные слои и мелкие частицы жидкости и газа
могут свободно перемещаться относительно друг друга по всем
направлениям. Достаточно, например, слегка подуть на поверхность воды в
стакане, чтобы вызвать движение воды. На реке или озере при малейшем
ветерке появляется рябь.

Подвижностью частиц газа и жидкости объясняется, что давление, производимое на них, передается не только в направлении действия силы, а в каждую точку. Рассмотрим это явление подробнее.

На рисунке, а изображен
сосуд, в котором содержится газ (или жидкость). Частицы равномерно
распределены по всему сосуду. Сосуд закрыт поршнем, который может
перемещаться вверх и вниз.

Прилагая
некоторую силу, заставим поршень немного переместиться внутрь и сжать
газ (жидкость), находящийся непосредственно под ним. Тогда частицы
(молекулы) расположатся в этом месте более плотно, чем прежде(рис, б).
Благодаря подвижности частицы газа будут перемещаться по всем
направлениям. Вследствие этого их расположение опять станет равномерным,
но более плотным, чем раньше (рис, в). Поэтому давление газа всюду
возрастет. Значит, добавочное давление передается всем частицам газа или
жидкости. Так, если давление на газ (жидкость) около самого поршня
увеличится на 1 Па, то во всех точках внутри газа
или жидкости давление станет больше прежнего на столько же. На 1 Па
увеличится давление и на стенки сосуда, и на дно, и на поршень.

Давление, производимое на жидкость или газ, передается на любую точку одинаково во всех направлениях.

Это утверждение называется законом Паскаля.

На основе закона Паскаля легко объяснить следующие опыты.

На
рисунке изображен полый шар, имеющий в различных местах небольшие
отверстия. К шару присоединена трубка, в которую вставлен поршень. Если
набрать воды в шар и вдвинуть в трубку поршень, то вода польется из всех
отверстий шара. В этом опыте поршень давит на поверхность воды в
трубке. Частицы воды, находящиеся под поршнем, уплотняясь, передают его
давление другим слоям, лежащим глубже. Таким образом, давление поршня
передается в каждую точку жидкости, заполняющей шар. В результате часть
воды выталкивается из шара в виде одинаковых струек, вытекающих из всех
отверстий.

Если
шар заполнить дымом, то при вдвигании поршня в трубку из всех отверстий
шара начнут выходить одинаковые струйки дыма. Это подтверждает, что и газы передают производимое на них давление во все стороны одинаково.

Давление в жидкости и газе.

Под действием веса жидкости резиновое дно в трубке прогнется.

На
жидкости, как и на все тела на Земле, действует сила тяжести. Поэтому,
каждый слой жидкости, налитой в сосуд, своим весом создает давление,
которое по закону Паскаля передается по всем направлениям.
Следовательно, внутри жидкости существует давление. В этом можно
убедиться на опыте.

В
стеклянную трубку, нижнее отверстие которой закрыто тонкой резиновой
пленкой, нальем воду. Под действием веса жидкости дно трубки прогнется.

Опыт
показывает, что, чем выше столб воды над резиновой пленкой, тем больше
она прогибается. Но всякий раз после того, как резиновое дно прогнулось,
вода в трубке приходит в равновесие (останавливается), так как, кроме
силы тяжести, на воду действует сила упругости растянутой резиновой
пленки.

По мере опускания трубки

резиновая пленка постепенно выпрямляется.

Силы, действующие на резиновую пленку,

одинаковы с обеих сторон.

Иллюстрация.

Дно отходит от цилиндра вследствие давления на него силы тяжести.

Опустим
трубку с резиновым дном, в которую налита вода, в другой, более широкий
сосуд с водой. Мы увидим, что по мере опускания трубки резиновая пленка
постепенно выпрямляется. Полное выпрямление пленки показывает, что
силы, действующие на нее сверху и снизу, равны. Наступает полное
выпрямление пленки тогда, когда уровни воды в трубке и сосуде совпадают.

Такой
же опыт можно провести с трубкой, в которой резиновая пленка закрывает
боковое отверстие, как это показано на рисунке, а. Погрузим эту трубку с
водой в другой сосуд с водой, как это изображено на рисунке, б.
Мы заметим, что пленка снова выпрямится, как только уровни воды в
трубке и сосуде сравняются. Это означает, что силы, действующие на
резиновую пленку, одинаковы со всех сторон.

Возьмем
сосуд, дно которого может отпадать. Опустим его в банку с водой. Дно
при этом окажется плотно прижатым к краю сосуда и не отпадет. Его
прижимает сила давления воды, направленная снизу вверх.

Будем
осторожно наливать воду в сосуд и следить за его дном. Как только
уровень воды в сосуде совпадет с уровнем воды в банке, оно отпадет от
сосуда.

В
момент отрыва на дно давит сверху вниз столб жидкости в сосуде, а снизу
вверх на дно передается давление такого же по высоте столба жидкости,
но находящейся в банке. Оба эти давления одинаковы, дно же отходит от
цилиндра вследствие действия на него собственной силы тяжести.

Выше были описаны опыты с водой, но если взять вместо воды любую другую жидкость, результаты опыта будут те же.

Итак, опыты показывают, что внутри
жидкости существует давление, и на одном и том же уровне оно одинаково
по всем направлениям. С глубиной давление увеличивается
.

Газы
в этом отношении не отличаются от жидкостей, ведь они тоже имеют вес.
Но надо помнить, что плотность газа в сотни раз меньше плотности
жидкости. Вес газа, находящегося в сосуде, мал, и его «весовое» давление
во многих случаях можно не учитывать.

основные формулы, определение и примеры

Гидромеханика

Основные формулы

Формула давления

Здесь р — давление (Па), — сила давления (Н), S — площадь опоры .

Давление столба жидкости

Здесь р — давление (Па), — плотность жидкости , g — ускорение свободного падения , h — высота столба жидкости (м).

Выталкивающая (архимедова) сила

Здесь — выталкивающая сила (Н), — плотность жидкости , g — ускорение свободного падения , — объем тела, погруженного в жидкость .

Формула гидравлического пресса

Здесь — силы, действующие на поршни пресса (Н), — площади поршней .

Уравнение неразрывности струи (теорема Эйлера)

Здесь — скорость жидкости (м/с) в сечении площадью — скорость жидкости (м/с) в сечении площадью .

Уравнение Бернулли

Здесь — плотность жидкости , g — ускорение свободного падения — высоты элемента жидкости над землей (м), — скорости на этих высотах (м/с), и — давления в жидкости (Па).

В основе гидродинамики лежат законы Ньютона, следствием которых являются все основные законы гидродинамики. Особенность здесь состоит в том, что эти законы применяют не к твердым телам, сохраняющим в процессе перемещения свою форму, а к жидкостям, не сохраняющим формы в процессе движения. Кроме того, если давление силы, приложенной к твердому телу, передается только в направлении ее действия, то давление, производимое на жидкость или газ, передается по всем направлениям одинаково. В этом состоит закон Паскаля — один из основных законов гидродинамики. Поэтому и силы давления распространяются по всей поверхности жидкости.

Давлением р называется отношение силы давления к площади опоры тела S.

Силой давления называют силу, действующую на тело перпендикулярно его площади опоры. Следует знать, что, хоть сила давления — величина векторная, но давление р — величина скалярная, оно не имеет направления.

С увеличением глубины жидкости давление в ней возрастает, т.к. увеличивается высота столба жидкости над уровнем, на котором определяется давление. Если жидкость налита в сосуд, то с увеличением ее глубины давление растет линейно с высотой столба жидкости, поэтому среднее давление жидкости на стенку сосуда равно половине ее давления на дно:

Если сверху на данный уровень давит несколько жидкостей, то давление на данном уровне равно сумме давлений каждой жидкости в отдельности.

В поле сил тяжести и в условиях земной атмосферы давление жидкости р на глубине h складывается из давления атмосферы на поверхность жидкости и давления самой жидкости на глубине h:

Следствием закона Паскаля является закон сообщающихся сосудов: в неподвижных и открытых сообщающихся сосудах любой формы давление жидкости на любом горизонтальном уровне одинаково.

Из закона сообщающихся сосудов вытекают два следствия.

Следствие 1: в неподвижных и открытых сообщающихся

сосудах высоты столбов жидкостей, отсчитываемые от уровня тп, ниже которого жидкость однородна, обратно пропорциональны плотностям этих жидкостей (рис. 68):

Следствие 2: в неподвижных и открытых сообщающихся

сосудах однородная жидкость всегда устанавливается на одинаковом уровне независимо от формы сосудов (рис. 69):

На законе Паскаля основано действие гидравлического пресса (рис. 70). — устройства, позволяющего получить выигрыш в силе во столько раз, во сколько площадь большего поршня больше площади меньшего поршня. Формула гидравлического пресса:

Согласно золотому правилу механики выигрыша в работе гидравлический пресс не дает, так как во сколько раз мы выигрываем на большом поршне в силе, во столько раз он проходит меньшее расстояние по сравнению с малым поршнем.

Другим законом гидродинамики, определяющим действие жидкостей и газов на погруженные в них тела, является закон Архимеда: на тело, погруженное в жидкость или газ, действует выталкивающая сила, направленная вверх и равная весу жидкости или газа, вытесненных телом.

Выталкивающая сила прямо пропорциональна плотности жидкости и объему погруженного в нее тела.

Выталкивающая (архимедова) сила не всегда направлена вверх. Как и всякая сила давления жидкости, она всегда направлена перпендикулярно поверхности жидкости. Если сосуд с жидкостью движется с ускорением горизонтально, то ее поверхность располагается под углом к горизонту, тем большим, чем больше ускорение. Поэтому выталкивающая сила, которая всегда перпендикулярна поверхности жидкости, уже не будет направлена вертикально.

Благодаря действию выталкивающей силы тела плавают в жидкости или газе. Условие плавания тел: тело плавает в жидкости, когда выталкивающая сила равна весу тела.

Когда плотность тела значительно меньше плотности жидкости, то равновесия может не наступить, если вес тела при всплытии все время будет меньше выталкивающей силы. При этом тело будет находиться на поверхности жидкости, совсем не погружаясь в нее, как это делает надувной шарик, брошенный в воду. Если плотность тела равна плотности жидкости, в которую оно полностью погружено, то тело будет плавать в жидкости во взвешенном состоянии, т.е. не поднимаясь и не опускаясь, поскольку при этом вес тела будет равен выталкивающей силе.

Если вес тела окажется больше выталкивающей силы, то оно утонет.

Нашу Землю окружает атмосфера, простирающаяся на высоту в несколько тысяч километров. Вследствие земного тяготения на атмосферный воздух действует сила тяжести, в результате чего верхние слои атмосферы давят на нижние. Атмосферное давление на тело обусловлено весом воздушных слоев, расположенных над ним.

На уровне моря величина атмосферного давления в среднем составляет 760 мм рт. ст. или Па. С увеличением высоты над уровнем моря атмосферное давление убывает вместе с весом воздушных слоев из-за ослабления земного тяготения, уменьшаясь через каждые сто метров примерно на 10 мм рт. ст. = 1330 Па.

Одним из первых измерил атмосферное давление итальянский ученый Торричелли. Это случилось три столетия назад. Торричелли взял тонкую стеклянную трубку длиной около метра, запаянную с одного конца, и наполнил ее доверху ртутью. Затем, закрыв открытый конец трубки, перевернул ее и опустил этим концом в открытую чашу с ртутью, после чего открыл трубку. Сначала под действием силы тяжести ртуть стала выливаться из трубки в чашу, а затем перестала. Это случилось в тот момент, когда давление ртути в трубке на уровне открытой поверхности ртути в чаше стало равно атмосферному давлению на открытую поверхность ртути в чаше. Так был создан первый в мире ртутный барометр.

Над ртутью в трубке образовалось замкнутое пространство, заполненное парами ртути, давление которых мало по сравнению с атмосферным, поэтому им пренебрегают. Это пространство было названо торричеллиевой пустотой.

Когда атмосферное давление увеличивалось, т.е. атмосфера сильнее давила на открытую поверхность ртути в чаше, уровень ртути в трубке повышался, а когда оно уменьшалось, то понижался. Присоединив к трубке шкалу, проградуированную в единицах давления, стали измерять давление атмосферы с высокой степенью точности.

Нормальным атмосферным давлением называется давление атмосферы, численно равное давлению столбика ртути высотой 760 мм. Это давление называют также физической атмосферой, сокращенно атм.

Таким образом, нормальное атмосферное давление порядка Па.

Барометры — это приборы, применяемые для измерения атмосферного давления.

Первым ртутным барометром была трубка Торричелли. Ртутные барометры — очень точные приборы, поэтому их применяют там, где необходима высокая точность измерений, например, при научных экспериментах. Но у них есть ряд недостатков: они некомпактны, ртуть дорога, ее пары ядовиты, она может разлиться, стекло — разбиться и т. д. Поэтому в быту и технике широко применяют другие барометры — анероиды.

Большинство жидкостей, в том числе и вода, практически несжимаемы. Их плотность везде одинакова и с течением времени не меняется.

Теорема о неразрывности струи или теорема Эйлера: произведение скорости течения жидкости по трубе переменного сечения и площади поперечного сечения трубы в любом месте одинаково:

Теорема о неразрывности струи является выражением закона сохранения массы движущейся жидкости. Ее можно применять к реальным жидкостям, сжимаемостью которых можно пренебречь.

Другим важнейшим уравнением гидродинамики является уравнение Бернулли, представляющее собой закон сохранения механической энергии, примененный к течению жидкости:

Из этого уравнения следует, что если скорость в потоке жидкости возрастает, то давление в ней падает, и наоборот, там, где скорость меньше, давление больше. Например, если лодку, оставленную на ночь у берега, забыть привязать, то утром ее можно обнаружить уплывшей далеко по течению. Это произойдет вследствие того, что из-за большего давления воды, медленно текущей вблизи берега, лодку вытеснит на середину, туда, где течение имеет большую скорость и, следовательно, меньшее давление.

Сформулированная выше зависимость давления от скорости течения среды справедлива и применительно к газам, когда их скорость невелика, так как при этом можно пренебречь сжимаемостью газов. Все должны знать, что вблизи мчащегося поезда стоять опасно, потому что воздух вблизи стенок вагонов увлекается поездом и движется с большей скоростью перед стоящим человеком, чем позади него. В результате, давление воздуха за спиной человека будет больше, чем между ним и поездом, и человека может толкнуть прямо под колеса.

Следствием уравнения Бернулли является возникновение подъемной силы крыла самолета. Подъемная сила крыла самолета обусловлена особым профилем крыла — профилем Жуковского, названным так в честь замечательного русского ученого-механика Н.Е. Жуковского, основоположника отечественной авиации.

Крыло самолета имеет особую несимметричную форму. Его профиль образует с линией горизонта угол атаки — угол между вектором скорости набегающего на крыло горизонтального потока воздуха и нижней плоскостью крыла (рис. 71). Благодаря несимметричности формы крыла и наличию угла атаки а воздушные массы за одно и то же время проходят над верхней поверхностью крыла больший путь, чем под нижней. В результате, давление , соответственно, меньше давления pt. Наличие разности давлений над и под крылом приводит к появлению подъемной силы, направленной снизу вверх, — оттуда, где давление больше, туда, где оно меньше. Величина подъемной силы в значительной степени зависит от угла атаки и при некотором критическом угле атаки достигает максимальной величины, после чего начинает убывать с дальнейшим ростом угла атаки. Расчет критического угла атаки является одной из важных задач самолетостроения.

Эта теория со страницы подробного решения задач по физике, там расположена теория и подробное решения задач по всем темам физики:

Задачи по физике с решением

Возможно вам будут полезны эти страницы:

Формула расчета силы давления. Формула давления воздуха, пара, жидкости или твердого тела

Рассмотрим, как можно рассчитать давление жидкости на дно и стенки сосуда. Решим сначала задачу с числовыми данными.
Прямоугольный бак наполнен водой (рис. 96). Площадь дна бака 16 м2, высота его 5 м. Определим давление воды на дно бака.

Сила, с которой вода давит на дно сосуда, равна весу столба воды высотой 5 м и площадью основания 16 м2, иначе говоря, эта сила равна весу всей воды в баке.

Чтобы найти вес воды, надо знать ее массу. Массу воды можно вычислить по объему и плотности. Найдем объем воды в баке, умножив площадь дна бака на его высоту: V= 16 м2*5 м=80 м3.
Теперь определим массу воды, для этого умножим ее плотность p = 1000 кг/м3 на объем: m =
1000 кг/м3 * 80 м3 = 80 000 кг. Мы знаем, что для определения веса тела надо его массу умножить на 9,8 Н/кг, так как тело массой 1 кг весит 9,8 Н.

Следовательно, вес воды в баке равен P =
9,8 Н/кг * 80 000 кг ≈ 800 000 Н. С такой силой вода давит на дно бака.

Разделив вес воды на площадь дна бака, найдем давление p:

p = 800000 H/16 м2 = 50 000 Па = 50 кПа.

Давление жидкости на дно сосуда можно рассчитать, пользуясь формулой, что значительно проще. Чтобы вывести эту формулу, вер­немся к задаче, но только решим ее в общем виде.

Обозначим высоту столба жидкости в сосуде буквой h, а площадь дна сосуда S.

Объем столба жидкости V=
Sh.

Масса жидкости
т
= pV,или m = pSh.

Вес этой жидкости P =
gm,
или P =
gpSh.

Так как вес столба жидкости равен силе, с которой жидкость давит на дно сосуда, то, разделив вес P
на площадь S,
получим давление р:

p = P/S, или p = gpSh/S

p =
gph.

Мы получили формулу для расчета давления жидкости на дно со­суда. Из этой формулы видно, что давление жидкости на дно сосуда прямо пропорционально плотности и высоте столба жидкости.

По этой формуле можно вычислять и давление на стенки, сосуда, а также давление внутри жидкости, в том числе давление снизу вверх, так как давление на одной и той же глубине одинаково по всем направлениям.

При расчете давления по формуле:

p =
gph

надо плотность p выражать в килограммах на кубический метр (кг/м3), а высоту столба жидкости h
— в метрах (м), g
= 9,8 Н/кг, тогда давление будет выражено в, паскалях (Па).

Пример. Определить давление нефти на дно цистерны, если высота столба нефти 10 м, а плотность ее 800 кг/м3.

Вопросы.
1. От каких величин зависит давление жидкости на дно сосуда? 2. Как зависит давление жидкости на дно сосуда от высоты столба жидкости? 3. Как зависит давление жидкости на дно сосуда от плотности жидкости?
4. Какие величины надо знать, чтобы рассчитать давление жидкости на стенки сосуда? 5. По какой формуле рассчитывают давление жидкости на дно и стенки сосуда?

Упражнения.
1. Определите давление на глубине 0,6 м в воде, керосине, ртути. 2. Вычислите давление воды на дно одной из глубочайших морских впадин, глубина, которой 10 900 м, Плотность морской воды 1030 кг/м3. 3. На рисунке 97 изображена футбольная камера, соединенная с вертикально расположенной стеклянной трубкой. В камере и трубке находится вода.
На камеру положена дощечка, а на нее — гиря массой 5 кг. Высота столба воды в трубке 1 м. Определите площадь соприкосновения дощечки с камерой.

Задания.
1. Возьмите высокий сосуд. В боковой поверхности его по прямой, на разной высоте от дна сделайте три небольших отверстия. Закройте отверстия спичками и налейте в сосуд до верха воды. Откройте отверстия и проследите за струйками вытекающей воды (рис. 98). Ответьте на вопросы: почему вода вытекает из отверстий? Из чего следует, что давление увеличивается с глубиной?
2. Прочтите в конце учебника параграфы «Гидростатический парадокс. Опыт Паскаля», «Давление на дне морей и океанов. Исследование морских глубин».

Жидкости и газы передают по всем направлениям приложенное к ним давление. Об этом гласит закон Паскаля и практический опыт.

Но существует еще и собственный вес, который тоже должен влиять на давление, существующее в жидкостях и газах. Вес собственных частей или слоев. Верхние слои жидкости давят на средние, средние на нижние, а последние — на дно. То есть мы можем говорить о существовании давления столба покоящейся жидкости на дно.

Формула давления столба жидкости

Формула для расчета давления столба жидкости высотой h имеет следующий вид:

где ρ — плотность жидкости,
g — ускорение свободного падения,
h — высота столба жидкости.

Это формула так называемого гидростатического давления жидкости.

Давление столба жидкости и газа

Гидростатическое давление, то есть, давление, оказываемое покоящейся жидкостью, на любой глубине не зависит от формы сосуда, в котором находится жидкость. Одно и то же количество воды, находясь в разных сосудах, будет оказывать разное давление на дно. Благодаря этому можно создать огромное давление даже небольшим количеством воды.

Это очень убедительно продемонстрировал Паскаль в семнадцатом веке. В закрытую бочку, полную воды, он вставил очень длинную узкую трубку. Поднявшись на второй этаж, он вылил в эту трубку всего лишь одну кружку воды. Бочка лопнула. Вода в трубке из-за малой толщины поднялась до очень большой высоты, и давление выросло до таких значений, что бочка не выдержала. То же самое справедливо и для газов. Однако, масса газов обычно намного меньше массы жидкостей, поэтому давление в газах, обусловленное собственным весом можно часто не учитывать на практике. Но в ряде случаев приходится считаться с этим. Например, атмосферное давление, которое давит на все находящиеся на Земле предметы, имеет большое значение в некоторых производственных процессах.

Благодаря гидростатическому давлению воды могут плавать и не тонуть корабли, которые весят зачастую не сотни, а тысячи килограмм, так как вода давит на них, как бы выталкивая наружу. Но именно по причине того же гидростатического давления на большой глубине у нас закладывает уши, а на очень большую глубину нельзя спуститься без специальных приспособлений — водолазного костюма или батискафа. Лишь немногие морские и океанические обитатели приспособились жить в условиях сильного давления на большой глубине, но по той же причине они не могут существовать в верхних слоях воды и могут погибнуть, если попадут на небольшую глубину.

Жидкости и газы передают по всем направлениям не только оказываемое на них внешнее давление, но и то давление, которое существует внутри их благодаря весу собственных частей. Верхние слои жидкости давят на средние, те — на нижние, а последние — на дно.

Давление, оказываемое покоящейся жидкостью, называется гидростатическим
.

Получим формулу для расчета гидростатического давления жидкости на произвольной глубине h
(в окрестности точки А
на рисунке 98). Сила давления, действующая в этом месте со стороны вышележащего узкого вертикального столба жидкости, может быть выражена двумя способами:
во-первых, как произведение давления в основании этого столба на площадь его сечения:

F = pS
;

во-вторых, как вес того же столба жидкости, т. е. произведение массы жидкости (которая может быть найдена по формуле m = ρV
, где объем V = Sh
) на ускорение свободного падения g
:

F = mg = ρShg
.

Приравняем оба выражения для силы давления:

pS = ρShg
.

Разделив обе части этого равенства на площадь S, найдем давление жидкости на глубине h
:

p = ρgh

. (37.1)

Мы получили формулу гидростатического давления
. Гидростатическое давление на любой глубине внутри жидкости не зависит от формы сосуда, в котором находится жидкость, и равно произведению плотности жидкости, ускорения свободного падения и глубины, на которой рассматривается давление.

Одно и то же количество воды, находясь в разных сосудах, может оказывать разное давление на дно. Поскольку это давление зависит от высоты столба жидкости, то в узких сосудах оно будет больше, чем в широких. Благодаря этому даже небольшим количеством воды можно создать очень большое давление. В 1648 г. это очень убедительно продемонстрировал Б. Паскаль. Он вставил в закрытую бочку, наполненную водой, узкую трубку и, поднявшись на балкон второго этажа дома, вылил в эту трубку кружку воды. Из-за малой толщины трубки вода в ней поднялась до большой высоты, и давление в бочке увеличилось настолько, что крепления бочки не выдержали, и она треснула (рис. 99).
Полученные нами результаты справедливы не только для жидкостей, но и для газов. Их слои также давят друг на друга, и потому в них тоже существует гидростатическое давление.

1. Какое давление называют гидростатическим? 2. От каких величин зависит это давление? 3. Выведите формулу гидростатического давления на произвольном глубине. 4. Каким образом с помощью небольшого количества воды можно создать большое давление? Расскажите об опыте Паскаля.
Экспериментальное задание.
Возьмите высокий сосуд и сделайте в его стенке три небольших отверстия на разной высоте. Закройте отверстия пластилином и наполните сосуд водой. Откройте отверстия и проследите за струями вытекающей воды (рис. 100). Почему вода вытекает из отверстий? Из чего следует, что давление воды увеличивается с глубиной?


Гидростатикой называется раздел гидравлики, в котором изучаются законы равновесия жидкостей и рассматривается практическое приложение этих законов.
Для того, чтобы понять гидростатику необходимо определиться в некоторых понятиях и определениях.

Закон Паскаля для гидростатики.

В 1653 году французским ученым Б. Паскалем был открыт закон, который принято называть основным законом гидростатики.

Звучит он так:

Давление на поверхность жидкости, произведенное внешними силами, передается в жидкости одинаково во всех направлениях.

Закон Паскаля легко понимается если взглянуть на молекулярное строение вещества. В жидкостях и газах молекулы обладают относительной свободой, они способны перемещаться друг относительно друга, в отличии от твердых тел. В твердых телах молекулы собраны в кристаллические решетки.

Относительная свобода, которой обладают молекулы жидкостей и газов, позволяет передавать давление производимое на жидкость или газ не только в направлении действия силы, но и во всех других направлениях.

Закон Паскаля для гидростатики нашел широкое распространение в промышленности. На этом законе основана работа гидроавтоматики, управляющей станками с ЧПУ, автомобилями и самолетами и многих других гидравлических машин .

Определение и формула гидростатического давления

Из описанного выше закона Паскаля вытекает, что:

Гидростатическое давление – это давление, производимое на жидкость силой тяжести.

Величина гидростатического давления не зависит от формы сосуда, в котором находится жидкость и определяется произведением

P = ρgh , где

ρ – плотность жидкости

g – ускорение свободного падения

h – глубина, на которой определяется давление.

Для иллюстрации этой формулы посмотрим на 3 сосуда разной формы.

Во всех трёх случаях давление жидкости на дно сосуда одинаково.

Полное давление жидкости в сосуде равно

P = P0 + ρgh, где

P0 – давление на поверхности жидкости. В большинстве случаев принимается равным атмосферному.

Сила гидростатического давления

Выделим в жидкости, находящейся в равновесии, некоторый объем, затем рассечем его произвольной плоскостью АВ на две части и мысленно отбросим одну из этих частей, например верхнюю. При этом мы должны приложить к плоскости АВ силы, действие которых будет эквивалентно действию отброшенной верхней части объема на оставшуюся нижнюю его часть.

Рассмотрим в плоскости сечения АВ замкнутый контур площадью ΔF, включающий в себя некоторую произвольную точку a. Пусть на эту площадь воздействует сила ΔP.

Тогда гидростатическое давление формула которого выглядит как

Рср = ΔP / ΔF

представлет собой силу, действующую на единицу площади, будет называться средним гидростатическим давлением или средним напряжением гидростатического давления по площади ΔF.

Истинное давление в разных точках этой площади может быть разным: в одних точках оно может быть больше, в других – меньше среднего гидростатического давления. Очевидно, что в общем случае среднее давление Рср будет тем меньше отличаться от истинного давления в точке а, чем меньше будет площадь ΔF, и в пределе среднее давление совпадет с истинным давлением в точке а.

Для жидкостей, находящихся в равновесии, гидростатическое давление жидкости аналогично напряжению сжатия в твердых телах.

Единицей измерения давления в системе СИ является ньютон на квадратный метр (Н/м 2) – её называют паскалем (Па). Поскольку величина паскаля очень мала, часто применяют укрупненные единицы:

килоньютон на квадратный метр – 1кН/м 2 = 1*10 3 Н/м 2

меганьютон на квадратный метр – 1МН/м 2 = 1*10 6 Н/м 2

Давление равное 1*10 5 Н/м 2 называется баром (бар).

В физической системе единицей намерения давления является дина на квадратный сантиметр (дина/м 2), в технической системе – килограмм-сила на квадратный метр (кгс/м 2). Практически давление жидкости обычно измеряют в кгс/см 2 , а давление равное 1 кгс/см 2 называется технической атмосферой (ат).

Между всеми этими единицами существует следующее соотношение:

1ат = 1 кгс/см 2 = 0,98 бар = 0,98 * 10 5 Па = 0,98 * 10 6 дин = 10 4 кгс/м 2

Следует помнить что между технической атмосферой (ат) и атмосферой физической (Ат) существует разница. 1 Ат = 1,033 кгс/см 2 и представляет собой нормальное давление на уровне моря. Атмосферное давление зависит от высоты расположения места над уровнем моря.

Измерение гидростатического давления

На практике применяют различные способы учета величины гидростатического давления. Если при определении гидростатического давления принимается во внимание и атмосферное давление, действующее на свободную поверхность жидкости, его называют полным или абсолютным. В этом случае величина давления обычно измеряется в технических атмосферах, называемых абсолютными (ата).

Часто при учете давления атмосферное давление на свободной поверхности не принимают во внимание, определяя так называемое избыточное гидростатическое давление, или манометрическое давление, т.е. давление сверх атмосферного.

Манометрическое давление определяют как разность между абсолютным давлением в жидкости и давлением атмосферным.

Рман = Рабс – Ратм

и измеряют также в технических атмосферах, называемых в этом случае избыточными.

Случается, что гидростатическое давление в жидкости оказывается меньше атмосферного. В этом случае говорят, что в жидкости имеется вакуум. Величина вакуума равняется разнице между атмосферным и и абсолютным давлением в жидкости

Рвак = Ратм – Рабс

и измеряется в пределах от нуля до атмосферы.

Гидростатическое давление воды обладает двумя основными свойствами:
Оно направлено по внутренней нормали к площади, на которую действует;
Величина давления в данной точке не зависит от направления (т.е. от ориентированности в пространстве площадки, на которой находится точка).

Первое свойство является простым следствием того положения, что в покоящейся жидкости отсутствуют касательные и растягивающие усилия.

Предположим, что гидростатическое давление направлено не по нормали, т.е. не перпендикулярно, а под некоторым углом к площадке. Тогда его можно разложить на две составляющие – нормальную и касательную. Наличие касательной составляющей из-за отсутствия в покоящейся жидкости сил сопротивления сдвигающим усилиям неизбежно привело бы к движению жидкости вдоль площадки, т.е. нарушило бы её равновесие.

Поэтому единственным возможным направлением гидростатического давления является его направление по нормали к площадке.

Если предположить что гидростатическое давление направлено не по внутренней, а по внешней нормали, т.е. не внутрь рассматриваемого объекта а наружу от него, то вследствие того, что жидкость не оказывает сопротивления растягивающим усилиям – частицы жидкости пришли бы в движение и её равновесие было бы нарушено.

Следовательно, гидростатическое давление воды всегда направлено по внутренней нормали и представляет собой сжимающее давление.

Из этого же правило следует, что если измениться давление в какой-то точке, то на такую же величину измениться давление в любой другой точке этой жидкости. В этом заключается закон Паскаля, который формулируется следующим образом: Давление производимое на жидкость, передается внутри жидкости во все стороны с одинаковой силой.

На применение этого закона основываются действие машин, работающих под гидростатическим давлением.

Видео по теме

Ещё одним фактором влияющим на величину давления является вязкость жидкости , которой до недавнего времени приято было пренебрегать. С появлением агрегатов работающих на высоком давлении вязкость пришлось так же учитывать. Оказалось, что при изменении давления, вязкость некоторых жидкостей, таких как масла, может изменяться в несколько раз. А это уже определяет возможность использовать такие жидкости в качестве рабочей среды.

Давление — это физическая величина, которая играет особую роль в природе и жизни человека. Это незаметное глазу явление не только влияет на состояние окружающей среды, но и очень хорошо ощущается всеми. Давайте разберемся, что это такое, какие виды его существуют и как находить давление (формула) в разных средах.

Что называется давлением в физике и химии

Данным термином именуется важная термодинамическая величина, которая выражается в соотношении перпендикулярно оказываемой силы давления на площадь поверхности, на которую она воздействует. Это явление не зависит от размера системы, в которой действует, поэтому относится к интенсивным величинам.

В состоянии равновесия, по давление одинаково для всех точек системы.

В физике и химии оное обозначается с помощью буквы «Р», что является сокращением от латинского названия термина — pressūra.

Если речь идет об осмотическом давлении жидкости (равновесие между давлением внутри и снаружи клетки), используется буква «П».

Единицы давления

Согласно стандартам Международной системы СИ, рассматриваемое физическое явление измеряется в паскалях (кириллицей — Па, латиницей — Ра).

Исходя из формулы давления получается, что один Па равен одному Н (ньютон — разделенному на один квадратный метр (единица измерения площади).

Однако на практике применять паскали довольно сложно, поскольку эта единица очень мала. В связи с этим, помимо стандартов системы СИ, данная величина может измеряться по-другому.

Ниже приведены наиболее известные ее аналоги. Большинство из них широко используется на просторах бывшего СССР.

  • Бары
    . Один бар равен 105 Па.
  • Торры, или миллиметры ртутного столба.
    Приблизительно один торр соответствует 133, 3223684 Па.
  • Миллиметры водяного столба.
  • Метры водяного столба.
  • Технические атмосферы.
  • Физические атмосферы.
    Одна атм равна 101 325 Па и 1,033233 ат.
  • Килограмм-силы на квадратный сантиметр.
    Также выделяются тонна-сила и грамм-сила. Помимо этого, есть аналог фунт-сила на квадратный дюйм.

Общая формула давления (физика 7-го класса)

Из определения данной физической величины можно определить способ ее нахождения. Выглядит он таким образом, как на фото ниже.

В нем F — это сила, а S — площадь. Иными словами, формула нахождения давления — это его сила, разделенная на площадь поверхности, на которую оно воздействует.

Также она может быть записана так: Р = mg / S или Р = pVg / S. Таким образом, эта физическая величина оказывается связанной с другими термодинамическими переменными: объемом и массой.

Для давления действует следующий принцип: чем меньше пространство, на которое влияет сила — тем большее количество давящей силы на него приходится. Если, же площадь увеличивается (при той же силе) — искомая величина уменьшается.

Формула гидростатического давления

Разные агрегатные состояния веществ, предусматривают наличие у них отличных друг от друга свойств. Исходя из этого, способы определения Р в них тоже будут другими.

К примеру, формула давления воды (гидростатического) выглядит вот так: Р = pgh. Также она применима и к газам. При этом ее нельзя использовать для вычисления атмосферного давления, из-за разности высот и плотностей воздуха.

В данной формуле р — плотность, g — ускорение свободного падения, а h — высота. Исходя из этого, чем глубже погружается предмет или объект, тем выше оказываемое на него давление внутри жидкости (газа).

Рассматриваемый вариант является адаптацией классической примера Р = F / S.

Если вспомнить, что сила равна производной массы на скорость свободного падения (F= mg), а масса жидкости — это производная объема на плотность (m = pV), то формулу давление можно записать как P = pVg / S. При этом объем — это площад, умноженная на высоту (V = Sh).

Если вставить эти данные, получится, что площадь в числителе и знаменателе можно сократить и на выходе — вышеупомянутая формула: Р = pgh.

Рассматривая давление в жидкостях, стоит помнить, что, в отличие от твердых тел, в них часто возможно искривление поверхностного слоя. А это, в свою очередь, способствует образованию дополнительного давления.

Для подобных ситуаций применяется несколько другая формула давления: Р = Р 0 + 2QH. В данном случае Р 0 — давление не искривленного слоя, а Q — поверхность натяжения жидкости. Н — это средняя кривизна поверхности, которую определяют по Закону Лапласа: Н = ½ (1/R 1 + 1/R 2). Составляющие R 1 и R 2 — это радиусы главной кривизны.

Парциальное давление и его формула

Хотя способ Р = pgh применим как для жидкостей, так и для газов, давление в последних лучше вычислять несколько другим путем.

Дело в том, что в природе, как правило, не очень часто встречаются абсолютно чистые вещества, ведь в ней преобладают смеси. И это касается не только жидкостей, но и газов. А как известно, каждый из таких компонентов осуществляет разное давление, называемое парциальным.

Определить его довольно просто. Оно равно сумме давления каждого компонента рассматриваемой смеси (идеальный газ).

Из этого следует, что формула парциального давления выглядит таким образом: Р = Р 1 + Р 2 + Р 3 … и так далее, согласно количеству составляющих компонентов.

Нередки случаи, когда необходимо определить давление воздуха. Однако некоторые по ошибке проводят вычисления только с кислородом по схеме Р = pgh. Вот только воздух — это смесь из разных газов. В нем встречаются азот, аргон, кислород и другие вещества. Исходя из сложившейся ситуации, формула давления воздуха — это сумма давлений всех его составляющих. А значит, следует приметь вышеупомянутую Р = Р 1 + Р 2 + Р 3 …

Наиболее распространенные приборы для измерения давления

Несмотря на то что высчитать рассматриваемую термодинамическую величину по вышеупомянутым формулам не сложно, проводить вычисление иногда попросту нет времени. Ведь нужно всегда учитывать многочисленные нюансы. Поэтому для удобства за несколько столетий был разработан ряд приборов, делающих это вместо людей.

Фактически почти все аппараты такого рода являются разновидностями манометра (помогает определять давление в газах и жидкостях). При этом они отличаются по конструкции, точности и сфере применения.

  • Атмосферное давление измеряется с помощью манометра, именуемого барометром. Если необходимо определить разряжение (то есть давление ниже атмосферного) — применяются другая его разновидность, вакуумметр.
  • Для того чтобы узнать артериальное давление у человека, в ход идет сфигмоманометр. Большинству он более известен под именем неинвазивного тонометра. Таких аппаратов существуют немало разновидностей: от ртутных механических до полностью автоматических цифровых. Их точность зависит от материалов, из которых они изготавливаются и места измерения.
  • Перепады давления в окружающей среде (по-английски — pressure drop) определяются с помощью или дифнамометров (не путать с динамометрами).

Виды давления

Рассматривая давление, формулу его нахождения и ее вариации для разных веществ, стоит узнать о разновидностях этой величины. Их пять.

  • Абсолютное.
  • Барометрическое
  • Избыточное.
  • Вакуумметрическое.
  • Дифференциальное.

Абсолютное

Так называется полное давление, под которым находится вещество или объект, без учета влияния других газообразных составляющих атмосферы.

Измеряется оно в паскалях и являет собою сумму избыточного и атмосферного давлений. Также он является разностью барометрического и вакуумметрического видов.

Вычисляется оно по формуле Р = Р 2 + Р 3 или Р = Р 2 — Р 4 .

За начало отсчета для абсолютного давления в условиях планеты Земля, берется давление внутри емкости, из которой удален воздух (то есть классический вакуум).

Только такой вид давления используется в большинстве термодинамических формул.

Барометрическое

Этим термином именуется давление атмосферы (гравитации) на все предметы и объекты, находящие в ней, включая непосредственно поверхность Земли. Большинству оно также известно под именем атмосферного.

Его причисляют к а его величина меняется относительно места и времени измерения, а также погодных условий и нахождения над/ниже уровня моря.

Величина барометрического давления равна модулю силы атмосферы на площади единицу по нормали к ней.

В стабильной атмосфере величина данного физического явления равна весу столпа воздуха на основание с площадью, равной единице.

Норма барометрического давления — 101 325 Па (760 мм рт. ст. при 0 градусов Цельсия). При этом чем выше объект оказывается от поверхности Земли, тем более низким становится давление на него воздуха. Через каждый 8 км оно снижается на 100 Па.

Благодаря этому свойству в горах вода в чайниках закивает намного быстрее, чем дома на плите. Дело в том, что давление влияет на температуру кипения: с его снижением последняя уменьшается. И наоборот. На этом свойстве построена работа таких кухонных приборов, как скороварка и автоклав. Повышение давления внутри их способствуют формированию в посудинах более высоких температур, нежели в обычных кастрюлях на плите.

Используется для вычисления атмосферного давления формула барометрической высоты. Выглядит она таким образом, как на фото ниже.

Р — это искомая величина на высоте, Р 0 — плотность воздуха возле поверхности, g — свободного падения ускорение, h — высота над Землей, м — молярная масса газа, т — температура системы, r — универсальная газовая постоянная 8,3144598 Дж⁄(моль х К), а е — это число Эйклера, равное 2.71828.

Часто в представленной выше формуле давления атмосферного вместо R используется К — постоянная Больцмана. Через ее произведение на число Авогадро нередко выражается универсальная газовая постоянная. Она более удобна для расчетов, когда число частиц задано в молях.

При проведении вычислений всегда стоит брать во внимание возможность изменения температуры воздуха из-за смены метеорологической ситуации или при наборе высоты над уровнем моря, а также географическую широту.

Избыточное и вакуумметрическое

Разницу между атмосферным и измеренным давлением окружающей среды называют избыточным давлением. В зависимости от результата, меняется название величины.

Если она положительная, ее называют манометрическим давлением.

Если же полученный результат со знаком минус — его именуют вакуумметрическим. Стоит помнить, что он не может быть больше барометрического.

Дифференциальное

Данная величина является разницей давлений в различных точках измерения. Как правило, ее используют для определения падения давления на каком-либо оборудовании. Особенно это актуально в нефтедобывающей промышленности.

Разобравшись с тем, что за термодинамическая величина называется давлением и с помощью каких формул ее находят, можно сделать вывод, что это явление весьма важно, а потому знания о нем никогда не будут лишними.

Формула вычисления давления. Формула давления воздуха, пара, жидкости или твердого тела. Как находить давление (формула)

Рассмотрим, как можно рассчитать давление жидкости на дно и стенки сосуда. Решим сначала задачу с числовыми данными.
Прямоугольный бак наполнен водой (рис. 96). Площадь дна бака 16 м2, высота его 5 м. Определим давление воды на дно бака.

Сила, с которой вода давит на дно сосуда, равна весу столба воды высотой 5 м и площадью основания 16 м2, иначе говоря, эта сила равна весу всей воды в баке.

Чтобы найти вес воды, надо знать ее массу. Массу воды можно вычислить по объему и плотности. Найдем объем воды в баке, умножив площадь дна бака на его высоту: V= 16 м2*5 м=80 м3.
Теперь определим массу воды, для этого умножим ее плотность p = 1000 кг/м3 на объем: m =
1000 кг/м3 * 80 м3 = 80 000 кг. Мы знаем, что для определения веса тела надо его массу умножить на 9,8 Н/кг, так как тело массой 1 кг весит 9,8 Н.

Следовательно, вес воды в баке равен P =
9,8 Н/кг * 80 000 кг ≈ 800 000 Н. С такой силой вода давит на дно бака.

Разделив вес воды на площадь дна бака, найдем давление p:

p = 800000 H/16 м2 = 50 000 Па = 50 кПа.

Давление жидкости на дно сосуда можно рассчитать, пользуясь формулой, что значительно проще. Чтобы вывести эту формулу, вер­немся к задаче, но только решим ее в общем виде.

Обозначим высоту столба жидкости в сосуде буквой h, а площадь дна сосуда S.

Объем столба жидкости V=
Sh.

Масса жидкости
т
= pV,или m = pSh.

Вес этой жидкости P =
gm,
или P =
gpSh.

Так как вес столба жидкости равен силе, с которой жидкость давит на дно сосуда, то, разделив вес P
на площадь S,
получим давление р:

p = P/S, или p = gpSh/S

p =
gph.

Мы получили формулу для расчета давления жидкости на дно со­суда. Из этой формулы видно, что давление жидкости на дно сосуда прямо пропорционально плотности и высоте столба жидкости.

По этой формуле можно вычислять и давление на стенки, сосуда, а также давление внутри жидкости, в том числе давление снизу вверх, так как давление на одной и той же глубине одинаково по всем направлениям.

При расчете давления по формуле:

p =
gph

надо плотность p выражать в килограммах на кубический метр (кг/м3), а высоту столба жидкости h
— в метрах (м), g
= 9,8 Н/кг, тогда давление будет выражено в, паскалях (Па).

Пример. Определить давление нефти на дно цистерны, если высота столба нефти 10 м, а плотность ее 800 кг/м3.

Вопросы.
1. От каких величин зависит давление жидкости на дно сосуда? 2. Как зависит давление жидкости на дно сосуда от высоты столба жидкости? 3. Как зависит давление жидкости на дно сосуда от плотности жидкости?
4. Какие величины надо знать, чтобы рассчитать давление жидкости на стенки сосуда? 5. По какой формуле рассчитывают давление жидкости на дно и стенки сосуда?

Упражнения.
1. Определите давление на глубине 0,6 м в воде, керосине, ртути. 2. Вычислите давление воды на дно одной из глубочайших морских впадин, глубина, которой 10 900 м, Плотность морской воды 1030 кг/м3. 3. На рисунке 97 изображена футбольная камера, соединенная с вертикально расположенной стеклянной трубкой. В камере и трубке находится вода.
На камеру положена дощечка, а на нее — гиря массой 5 кг. Высота столба воды в трубке 1 м. Определите площадь соприкосновения дощечки с камерой.

Задания.
1. Возьмите высокий сосуд. В боковой поверхности его по прямой, на разной высоте от дна сделайте три небольших отверстия. Закройте отверстия спичками и налейте в сосуд до верха воды. Откройте отверстия и проследите за струйками вытекающей воды (рис. 98). Ответьте на вопросы: почему вода вытекает из отверстий? Из чего следует, что давление увеличивается с глубиной?
2. Прочтите в конце учебника параграфы «Гидростатический парадокс. Опыт Паскаля», «Давление на дне морей и океанов. Исследование морских глубин».

Жидкости и газы передают по всем направлениям приложенное к ним давление. Об этом гласит закон Паскаля и практический опыт.

Но существует еще и собственный вес, который тоже должен влиять на давление, существующее в жидкостях и газах. Вес собственных частей или слоев. Верхние слои жидкости давят на средние, средние на нижние, а последние — на дно. То есть мы можем говорить о существовании давления столба покоящейся жидкости на дно.

Формула давления столба жидкости

Формула для расчета давления столба жидкости высотой h имеет следующий вид:

где ρ — плотность жидкости,
g — ускорение свободного падения,
h — высота столба жидкости.

Это формула так называемого гидростатического давления жидкости.

Давление столба жидкости и газа

Гидростатическое давление, то есть, давление, оказываемое покоящейся жидкостью, на любой глубине не зависит от формы сосуда, в котором находится жидкость. Одно и то же количество воды, находясь в разных сосудах, будет оказывать разное давление на дно. Благодаря этому можно создать огромное давление даже небольшим количеством воды.

Это очень убедительно продемонстрировал Паскаль в семнадцатом веке. В закрытую бочку, полную воды, он вставил очень длинную узкую трубку. Поднявшись на второй этаж, он вылил в эту трубку всего лишь одну кружку воды. Бочка лопнула. Вода в трубке из-за малой толщины поднялась до очень большой высоты, и давление выросло до таких значений, что бочка не выдержала. То же самое справедливо и для газов. Однако, масса газов обычно намного меньше массы жидкостей, поэтому давление в газах, обусловленное собственным весом можно часто не учитывать на практике. Но в ряде случаев приходится считаться с этим. Например, атмосферное давление, которое давит на все находящиеся на Земле предметы, имеет большое значение в некоторых производственных процессах.

Благодаря гидростатическому давлению воды могут плавать и не тонуть корабли, которые весят зачастую не сотни, а тысячи килограмм, так как вода давит на них, как бы выталкивая наружу. Но именно по причине того же гидростатического давления на большой глубине у нас закладывает уши, а на очень большую глубину нельзя спуститься без специальных приспособлений — водолазного костюма или батискафа. Лишь немногие морские и океанические обитатели приспособились жить в условиях сильного давления на большой глубине, но по той же причине они не могут существовать в верхних слоях воды и могут погибнуть, если попадут на небольшую глубину.

Возьмем
цилиндрический сосуд с горизонтальным дном и вертикальными стенками,
наполненный жидкостью до высоты (рис. 248).

Рис. 248. В
сосуде с вертикальными стенками сила давления на дно равна весу всей налитой
жидкости

Рис. 249. Во
всех изображенных сосудах сила давления на дно одинакова. В первых двух сосудах
она больше веса налитой жидкости, в двух других — меньше

Гидростатическое
давление в каждой точке дна сосуда будет одно и то же:

Если
дно сосуда имеет площадь , то сила давления жидкости на дно
сосуда ,
т. е. равна весу жидкости, налитой в сосуд.

Рассмотрим
теперь сосуды, отличающиеся по форме, но с одинаковой площадью дна (рис. 249).
Если жидкость в каждом из них налита до одной и той же высоты , то давление на
дно . во
всех сосудах одно и то же. Следовательно, сила давления на дно, равная

также
одинакова во всех сосудах. Она равна весу столба жидкости с основанием, равным
площади дна сосуда, и высотой, равной высоте налитой жидкости. На рис. 249 этот
столб показан около каждого сосуда штриховыми линиями. Обратите внимание на то,
что сила давления на дно не зависит от формы сосуда и может быть как больше,
так и меньше веса налитой жидкости.

Рис. 250.
Прибор Паскаля с набором сосудов. Сечения одинаковы у всех сосудов

Рис. 251.
Опыт с бочкой Паскаля

Этот
вывод можно проверить на опыте при помощи прибора, предложенного Паскалем (рис.
250). На подставке можно закреплять сосуды различной формы, не имеющие дна.
Вместо дна снизу к сосуду плотно прижимается подвешенная к коромыслу весов
пластинка. При наличии жидкости в сосуде на пластинку действует сила давления,
которая отрывает пластинку, когда сила давления начнет превосходить вес гири,
стоящей на другой чашке весов.

У
сосуда с вертикальными стенками (цилиндрический сосуд) дно открывается, когда
вес налитой жидкости достигает веса гири. У сосудов другой формы дно
открывается при той же самой высоте столба жидкости, хотя вес налитой воды
может быть и больше (расширяющийся кверху сосуд), и меньше (суживающийся сосуд)
веса гири.

Этот
опыт приводит к мысли, что при надлежащей форме сосуда можно с помощью
небольшого количества воды получить огромные силы давления на дно. Паскаль
присоединил к плотно законопаченной бочке, налитой водой, длинную тонкую
вертикальную трубку (рис. 251). Когда трубку заполняют водой, сила
гидростатического давления на дно становится равной весу столба воды, площадь
основания которого равна площади дна бочки, а высота равна высоте трубки.
Соответственно увеличиваются и силы давления на стенки и верхнее днище бочки.
Когда Паскаль заполнил трубку до высоты в несколько метров, для чего потребовалось
лишь несколько кружек воды, возникшие силы давления разорвали бочку.

Как
объяснить, что сила давления на дно сосуда может быть, в зависимости от формы
сосуда, больше или меньше веса жидкости, содержащейся в сосуде? Ведь сила,
действующая со стороны сосуда на жидкость, должна уравновешивать вес жидкости.
Дело в том, что на жидкость в сосуде действует не только дно, но и стенки
сосуда. В расширяющемся кверху сосуде силы, с которыми стенки действуют на
жидкость, имеют составляющие, направленные вверх: таким образом, часть веса
жидкости уравновешивается силами давления стенок и только часть должна быть
уравновешена силами давления со стороны дна. Наоборот, в суживающемся кверху
сосуде дно действует на жидкость вверх, а стенки — вниз; поэтому сила давления
на дно оказывается больше веса жидкости. Сумма же сил, действующих на жидкость
со стороны дна сосуда и его стенок, всегда равна весу жидкости. Рис. 252
наглядно показывает распределение сил, действующих со стороны стенок на
жидкость в сосудах различной формы.

Рис. 252.
Силы, действующие на жидкость со стороны стенок в сосудах различной формы

Рис. 253. При
наливании воды в воронку цилиндр поднимается вверх.

В
суживающемся кверху сосуде со стороны жидкости на стенки действует сила,
направленная вверх. Если стенки такого сосуда сделать подвижными, то жидкость
поднимет их. Такой опыт можно произвести на следующем приборе: поршень
неподвижно закреплен, и на него надет цилиндр, переходящий в вертикальную
трубку (рис. 253). Когда пространство над поршнем заполняется водой, силы
давления на участках и стенок цилиндра поднимают цилиндр
вверх.


Гидростатикой называется раздел гидравлики, в котором изучаются законы равновесия жидкостей и рассматривается практическое приложение этих законов.
Для того, чтобы понять гидростатику необходимо определиться в некоторых понятиях и определениях.

Закон Паскаля для гидростатики.

В 1653 году французским ученым Б. Паскалем был открыт закон, который принято называть основным законом гидростатики.

Звучит он так:

Давление на поверхность жидкости, произведенное внешними силами, передается в жидкости одинаково во всех направлениях.

Закон Паскаля легко понимается если взглянуть на молекулярное строение вещества. В жидкостях и газах молекулы обладают относительной свободой, они способны перемещаться друг относительно друга, в отличии от твердых тел. В твердых телах молекулы собраны в кристаллические решетки.

Относительная свобода, которой обладают молекулы жидкостей и газов, позволяет передавать давление производимое на жидкость или газ не только в направлении действия силы, но и во всех других направлениях.

Закон Паскаля для гидростатики нашел широкое распространение в промышленности. На этом законе основана работа гидроавтоматики, управляющей станками с ЧПУ, автомобилями и самолетами и многих других гидравлических машин .

Определение и формула гидростатического давления

Из описанного выше закона Паскаля вытекает, что:

Гидростатическое давление – это давление, производимое на жидкость силой тяжести.

Величина гидростатического давления не зависит от формы сосуда, в котором находится жидкость и определяется произведением

P = ρgh , где

ρ – плотность жидкости

g – ускорение свободного падения

h – глубина, на которой определяется давление.

Для иллюстрации этой формулы посмотрим на 3 сосуда разной формы.

Во всех трёх случаях давление жидкости на дно сосуда одинаково.

Полное давление жидкости в сосуде равно

P = P0 + ρgh, где

P0 – давление на поверхности жидкости. В большинстве случаев принимается равным атмосферному.

Сила гидростатического давления

Выделим в жидкости, находящейся в равновесии, некоторый объем, затем рассечем его произвольной плоскостью АВ на две части и мысленно отбросим одну из этих частей, например верхнюю. При этом мы должны приложить к плоскости АВ силы, действие которых будет эквивалентно действию отброшенной верхней части объема на оставшуюся нижнюю его часть.

Рассмотрим в плоскости сечения АВ замкнутый контур площадью ΔF, включающий в себя некоторую произвольную точку a. Пусть на эту площадь воздействует сила ΔP.

Тогда гидростатическое давление формула которого выглядит как

Рср = ΔP / ΔF

представлет собой силу, действующую на единицу площади, будет называться средним гидростатическим давлением или средним напряжением гидростатического давления по площади ΔF.

Истинное давление в разных точках этой площади может быть разным: в одних точках оно может быть больше, в других – меньше среднего гидростатического давления. Очевидно, что в общем случае среднее давление Рср будет тем меньше отличаться от истинного давления в точке а, чем меньше будет площадь ΔF, и в пределе среднее давление совпадет с истинным давлением в точке а.

Для жидкостей, находящихся в равновесии, гидростатическое давление жидкости аналогично напряжению сжатия в твердых телах.

Единицей измерения давления в системе СИ является ньютон на квадратный метр (Н/м 2) – её называют паскалем (Па). Поскольку величина паскаля очень мала, часто применяют укрупненные единицы:

килоньютон на квадратный метр – 1кН/м 2 = 1*10 3 Н/м 2

меганьютон на квадратный метр – 1МН/м 2 = 1*10 6 Н/м 2

Давление равное 1*10 5 Н/м 2 называется баром (бар).

В физической системе единицей намерения давления является дина на квадратный сантиметр (дина/м 2), в технической системе – килограмм-сила на квадратный метр (кгс/м 2). Практически давление жидкости обычно измеряют в кгс/см 2 , а давление равное 1 кгс/см 2 называется технической атмосферой (ат).

Между всеми этими единицами существует следующее соотношение:

1ат = 1 кгс/см 2 = 0,98 бар = 0,98 * 10 5 Па = 0,98 * 10 6 дин = 10 4 кгс/м 2

Следует помнить что между технической атмосферой (ат) и атмосферой физической (Ат) существует разница. 1 Ат = 1,033 кгс/см 2 и представляет собой нормальное давление на уровне моря. Атмосферное давление зависит от высоты расположения места над уровнем моря.

Измерение гидростатического давления

На практике применяют различные способы учета величины гидростатического давления. Если при определении гидростатического давления принимается во внимание и атмосферное давление, действующее на свободную поверхность жидкости, его называют полным или абсолютным. В этом случае величина давления обычно измеряется в технических атмосферах, называемых абсолютными (ата).

Часто при учете давления атмосферное давление на свободной поверхности не принимают во внимание, определяя так называемое избыточное гидростатическое давление, или манометрическое давление, т.е. давление сверх атмосферного.

Манометрическое давление определяют как разность между абсолютным давлением в жидкости и давлением атмосферным.

Рман = Рабс – Ратм

и измеряют также в технических атмосферах, называемых в этом случае избыточными.

Случается, что гидростатическое давление в жидкости оказывается меньше атмосферного. В этом случае говорят, что в жидкости имеется вакуум. Величина вакуума равняется разнице между атмосферным и и абсолютным давлением в жидкости

Рвак = Ратм – Рабс

и измеряется в пределах от нуля до атмосферы.

Гидростатическое давление воды обладает двумя основными свойствами:
Оно направлено по внутренней нормали к площади, на которую действует;
Величина давления в данной точке не зависит от направления (т.е. от ориентированности в пространстве площадки, на которой находится точка).

Первое свойство является простым следствием того положения, что в покоящейся жидкости отсутствуют касательные и растягивающие усилия.

Предположим, что гидростатическое давление направлено не по нормали, т.е. не перпендикулярно, а под некоторым углом к площадке. Тогда его можно разложить на две составляющие – нормальную и касательную. Наличие касательной составляющей из-за отсутствия в покоящейся жидкости сил сопротивления сдвигающим усилиям неизбежно привело бы к движению жидкости вдоль площадки, т.е. нарушило бы её равновесие.

Поэтому единственным возможным направлением гидростатического давления является его направление по нормали к площадке.

Если предположить что гидростатическое давление направлено не по внутренней, а по внешней нормали, т.е. не внутрь рассматриваемого объекта а наружу от него, то вследствие того, что жидкость не оказывает сопротивления растягивающим усилиям – частицы жидкости пришли бы в движение и её равновесие было бы нарушено.

Следовательно, гидростатическое давление воды всегда направлено по внутренней нормали и представляет собой сжимающее давление.

Из этого же правило следует, что если измениться давление в какой-то точке, то на такую же величину измениться давление в любой другой точке этой жидкости. В этом заключается закон Паскаля, который формулируется следующим образом: Давление производимое на жидкость, передается внутри жидкости во все стороны с одинаковой силой.

На применение этого закона основываются действие машин, работающих под гидростатическим давлением.

Видео по теме

Ещё одним фактором влияющим на величину давления является вязкость жидкости , которой до недавнего времени приято было пренебрегать. С появлением агрегатов работающих на высоком давлении вязкость пришлось так же учитывать. Оказалось, что при изменении давления, вязкость некоторых жидкостей, таких как масла, может изменяться в несколько раз. А это уже определяет возможность использовать такие жидкости в качестве рабочей среды.

Давление жидкости – формула и описание кратко (физика, 7 класс)

Давление — это мера распределения силы на некоторой площади. Давление может создаваться не только твёрдыми телами, но и жидкими или газообразными. Использование этой физической величины для жидкостей имеет некоторые особенности, которые изучают на уроках физики в 7 классе. Рассмотрим их подробнее, выведем формулу давления жидкости.

Распределение давления в жидкости

Давление подразумевает действие некоторой силы. Для жидкостей такая сила может иметь два источника. И первый возможный источник — это внешняя сила.

Представим себе вертикальный цилиндр с поршнем в верхней части, полностью заполненный жидкостью. Теперь, если со стороны поршня на молекулы жидкости начнёт действовать сила, то ближайшие к поршню молекулы жидкости начнут смещаться вниз. При этом они встретят на пути противодействие молекул более глубоких слоёв и начнут передавать усилие им. Молекулы более глубоких слоёв в свою очередь будут передавать усилия ещё более глубоким слоям, и так далее, до самого дна поршня.

Напомним, что молекулы в жидкости связаны друг с другом слабо и совершают хаотичные тепловые движения. Если на молекулу действует сила со стороны вышележащего слоя, а вниз её «не пускает» нижележащий слой, то молекула начинает смещение вбок. Получается, что усилие поршня, направленное вниз, будет передаваться в жидкости не только вниз, но и в стороны. При этом за счёт хаотичного движения молекул «траектория передачи силы» может быть любой, сколь угодно извилистой — сила будет передаваться по этому пути через жидкость и создавать давление в любой точке.

Давление, производимое на жидкость, передаётся в любую точку жидкости без изменения во всех направлениях. Данный закон был открыт Б. Паскалем, и носит его имя. В честь этого физика также была названа единица измерения давления.

Рис. 1. Закон Паскаля.

Давление в глубине жидкости

Вторым источником давления жидкости является её собственный вес.

Рис. 2. Вес тела

Этот вес будет распределён по площади дна ёмкости, содержащей жидкость, и мера этого распределения характеризуется давлением.

Если сосуд имеет площадь дна $S$, и столб жидкости в нём будет иметь высоту $h$, то объём этой жидкости будет равен:

$$V=Sh$$

Если плотность жидкости равна $rho$, то масса жидкости равна:

$$m=rho V=rho Sh$$

Вес покоящегося тела равен силе тяжести. То есть:

$$P=mmathrm{g} =rho Sh mathrm{g}$$

Этот вес распределён по площади дна $S$. Следовательно, давление на дне сосуда будет равно:

$$p={Pover S}={ rho Sh mathrm{g}over S} $$

Сокращая значение площади, получаем формулу давления жидкости на глубине $h$:

$$p = rho mathrm{g} h$$

В данную формулу не входит площадь дна сосуда. Сосуд может иметь любую форму, давление жидкости на его дно будет определяться только высотой столба жидкости и её плотностью. По этой же формуле определяется давление в любой точке жидкости, не только на дне.

Рис. 3. Давление столба жидкости

Что мы узнали?

Давление в жидкости распространяется во все стороны без изменений. Источником этого давления может являться внешняя сила или вес самой жидкости. В последнем случае давление жидкости зависит только от плотности этой жидкости и от глубины точки измерения.

Предыдущая

ФизикаТаблица физических величин – обозначения и единицы измерения в системе СИ, формулы кратко (7 класс)

Следующая

ФизикаОтносительная влажность воздуха – формула, определение, таблица с расчетом (8 класс)

Формула давления через плотность. Формула давления воздуха, пара, жидкости или твердого тела

В ходе этого урока с помощью математических преобразований и логических умозаключений будет получена формула для расчета давления жидкости на дно и стенки сосуда.

Тема: Давление твердых тел, жидкостей и газов

Урок: Расчет давления жидкости на дно и стенки сосуда

Для того чтобы упростить вывод формулы для расчета давления на дно и стенки сосуда, удобнее всего использовать сосуд в форме прямоугольного параллелепипеда (Рис. 1).

Рис. 1. Сосуд для расчета давления жидкости

Площадь дна этого сосуда — S
, его высота — h
. Предположим, что сосуд наполнен жидкостью на всю высоту h
. Чтобы определить давление на дно, нужно силу, действующую на дно, разделить на площадь дна. В нашем случае сила — это вес жидкости P
, находящейся в сосуде

Поскольку жидкость в сосуде неподвижна, ее вес равен силе тяжести, которую можно вычислить, если известна масса жидкости m

Напомним, что символом g
обозначено ускорение свободного падения.

Для того чтобы найти массу жидкости, необходимо знать ее плотность ρ
и объем V

Объем жидкости в сосуде мы получим, умножив площадь дна на высоту сосуда

Эти величины изначально известны. Если их по очереди подставить в приведенные выше формулы, то для вычисления давления получим следующее выражение:

В этом выражении числитель и знаменатель содержат одну и ту же величину S
— площадь дна сосуда. Если на нее сократить, получится искомая формула для расчета давления жидкости на дно сосуда:

Итак, для нахождения давления необходимо умножить плотность жидкости на величину ускорения свободного падения и высоту столба жидкости.

Полученная выше формула называется формулой гидростатического давления. Она позволяет найти давление на дно
сосуда. А как рассчитать давление на боковые
стенки
сосуда? Чтобы ответить на этот вопрос, вспомним, что на прошлом уроке мы установили, что давление на одном и том же уровне одинаково во всех направлениях. Это значит, давление в любой точке жидкости на заданной глубине h
может быть найдено по той же формуле.

Рассмотрим несколько примеров.

Возьмем два сосуда. В одном из них находится вода, а в другом — подсолнечное масло. Уровень жидкости в обоих сосудах одинаков. Одинаковым ли будет давление этих жидкостей на дно сосудов? Безусловно, нет. В формулу для расчета гидростатического давления входит плотность жидкости. Поскольку плотность подсолнечного масла меньше, чем плотность воды, а высота столба жидкостей одинакова, то масло будет оказывать на дно меньшее давление, чем вода (Рис. 2).

Рис. 2. Жидкости с различной плотностью при одной высоте столба оказывают на дно различные давления

Еще один пример. Имеются три различных по форме сосуда. В них до одного уровня налита одна и та же жидкость. Будет ли одинаковым давление на дно сосудов? Ведь масса, а значит, и вес жидкостей в сосудах различен. Да, давление будет одинаковым (Рис. 3). Ведь в формуле гидростатического давления нет никакого упоминания о форме сосуда, площади его дна и весе налитой в него жидкости. Давление определяется исключительно плотностью жидкости и высотой ее столба.

Рис. 3. Давление жидкости не зависит от формы сосуда

Мы получили формулу для нахождения давления жидкости на дно и стенки сосуда. Этой формулой можно пользоваться и для расчета давления в объеме жидкости на заданной глубине. Она может быть использована для определения глубины погружения аквалангиста, при расчете конструкции батискафов, подводных лодок, для решения множества других научных и инженерных задач.

Список литературы

  1. Перышкин А. В. Физика. 7 кл. — 14-е изд., стереотип. — М.: Дрофа, 2010.
  2. Перышкин А. В. Сборник задач по физике, 7-9 кл.: 5-е изд., стереотип. — М: Издательство «Экзамен», 2010.
  3. Лукашик В. И., Иванова Е. В. Сборник задач по физике для 7-9 классов общеобразовательных учреждений. — 17-е изд. — М.: Просвещение, 2004.
  1. Единая коллекция цифровых образовательных ресурсов ().

Домашнее задание

  1. Лукашик В. И., Иванова Е. В. Сборник задач по физике для 7-9 классов №504-513.

Человек на лыжах, и без них.

По рыхлому снегу человек идёт с большим трудом, глубоко проваливаясь при каждом шаге. Но, надев лыжи, он может идти, почти не проваливаясь в него. Почему? На лыжах или без лыж человек действует на снег с одной и той же силой, равной своему весу. Однако действие этой силы в обоих случаях различно, потому что различна площадь поверхности, на которую давит человек, с лыжами и без лыж. Площадь поверхности лыж почти в 20 раз больше площади подошвы. Поэтому, стоя на лыжах, человек действует на каждый квадратный сантиметр площади поверхности снега с силой, в 20 раз меньшей, чем стоя на снегу без лыж.

Ученик, прикалывая кнопками газету к доске, действует на каждую кнопку с одинаковой силой. Однако кнопка, имеющая более острый конец, легче входит в дерево.

Значит, результат действия силы зависит не только от её модуля, направления и точки приложения, но и от площади той поверхности, к которой она приложена (перпендикулярно которой она действует).

Этот вывод подтверждают физические опыты.

Опыт.Результат действия данной силы зависит от того, какая сила действует на единицу площади поверхности.

По углам небольшой доски надо вбить гвозди. Сначала гвозди, вбитые в доску, установим на песке остриями вверх и положим на доску гирю. В этом случае шляпки гвоздей лишь незначительно вдавливаются в песок. Затем доску перевернем и поставим гвозди на острие. В этом случае площадь опоры меньше, и под действием той же силы гвозди значительно углубляются в песок.

Опыт. Вторая иллюстрация.

От того, какая сила действует на каждую единицу площади поверхности, зависит результат действия этой силы.

В рассмотренных примерах силы действовали перпендикулярно поверхности тела. Вес человека был перпендикулярен поверхности снега; сила, действовавшая на кнопку, перпендикулярна поверхности доски.

Величина, равная отношению силы, действующей перпендикулярно поверхности, к площади этой поверхности, называется давлением
.

Чтобы определить давление, надо силу, действующую перпендикулярно поверхности, разделить на площадь поверхности:

давление = сила / площадь
.

Обозначим величины, входящие в это выражение: давление — p
, сила, действующая на поверхность, — F
и площадь поверхности — S
.

Тогда получим формулу:

p = F/S

Понятно, что бóльшая по значению сила, действующую на ту же площадь, будет производить большее давление.

За единицу давления принимается такое давление, которое производит сила в 1 Н, действующая на поверхность площадью 1 м 2 перпендикулярно этой поверхности
.

Единица давления — ньютон на квадратный метр
(1 Н / м 2). В честь французского ученого Блеза Паскаля

она называется паскалем (Па
). Таким образом,

1 Па = 1 Н / м 2
.

Используется также другие единицы давления: гектопаскаль
(гПа
) и килопаскаль
(кПа
).

1 кПа = 1000 Па;

1 гПа = 100 Па;

1 Па = 0,001 кПа;

1 Па = 0,01 гПа.

Запишем условие задачи и решим её.

Дано

: m = 45 кг, S = 300 см 2 ; p = ?

В единицах СИ: S = 0,03 м 2

Решение:

p
= F
/S
,

F
= P
,

P
= g·m
,

P
= 9,8 Н · 45 кг ≈ 450 Н,

p
= 450/0,03 Н / м 2 = 15000 Па = 15 кПа

«Ответ»: p = 15000 Па = 15 кПа

Способы уменьшения и увеличения давления.

Тяжелый гусеничный трактор производит на почву давление равное 40 — 50 кПа, т. е. всего в 2 — 3 раза больше, чем давление мальчика массой 45 кг. Это объясняется тем, что вес трактора распределяется на бóльшую площадь за счёт гусеничной передачи. А мы установили, что чем больше площадь опоры, тем меньше давление, производимое одной и той же силой на эту опору

.

В зависимости от того, нужно ли получить малое или большое давление, площадь опоры увеличивается или уменьшается. Например, для того, чтобы грунт мог выдержать давление возводимого здания, увеличивают площадь нижней части фундамента.

Шины грузовых автомобилей и шасси самолетов делают значительно шире, чем легковых. Особенно широкими делают шины у автомобилей, предназначенных для передвижения в пустынях.

Тяжелые машины, как трактор, танк или болотоход, имея большую опорную площадь гусениц, проходят по болотистой местности, по которой не пройдет человек.

С другой стороны, при малой площади поверхности можно небольшой силой произвести большое давление. Например, вдавливая кнопку в доску, мы действуем на нее с силой около 50 Н. Так как площадь острия кнопки примерно 1 мм 2 , то давление, производимое ею, равно:

p = 50 Н/ 0, 000 001 м 2 = 50 000 000 Па = 50 000 кПа.

Для сравнения, это давление в 1000 раз больше давления, производимого гусеничным трактором на почву. Можно найти еще много таких примеров.

Лезвие режущих и острие колющих инструментов (ножей, ножниц, резцов, пил, игл и др.) специально остро оттачивается. Заточенный край острого лезвия имеет маленькую площадь, поэтому при помощи даже малой силы создается большое давление, и таким инструментом легко работать.

Режущие и колющие приспособления встречаются и в живой природе: это зубы, когти, клювы, шипы и др. — все они из твердого материала, гладкие и очень острые.

Давление

Известно, что молекулы газа беспорядочно движутся.

Мы уже знаем, что газы, в отличие от твердых тел и жидкостей, заполняют весь сосуд, в котором находятся. Например, стальной баллон для хранения газов, камера автомобильной шины или волейбольный мяч. При этом газ оказывает давление на стенки, дно и крышку баллона, камеры или любого другого тела, в котором он находится. Давление газа обусловлено иными причинами, чем давление твердого тела на опору.

Известно, что молекулы газа беспорядочно движутся. При своем движении они сталкиваются друг с другом, а также со стенками сосуда, в котором находится газ. Молекул в газе много, поэтому и число их ударов очень велико. Например, число ударов молекул воздуха, находящегося в комнате, о поверхность площадью 1 см 2 за 1 с выражается двадцатитрехзначным числом. Хотя сила удара отдельной молекулы мала, но действие всех молекул на стенки сосуда значительно, — оно и создает давление газа.

Итак, давление газа на стенки сосуда (и на помещенное в газ тело) вызывается ударами молекул газа

.

Рассмотрим следующий опыт. Под колокол воздушного насоса поместим резиновый шарик. Он содержит небольшое количество воздуха и имеет неправильную форму. Затем насосом откачиваем воздух из-под колокола. Оболочка шарика, вокруг которой воздух становится все более разреженным, постепенно раздувается и принимает форму правильного шара.

Как объяснить этот опыт?

Для хранения и перевозки сжатого газа используются специальные прочные стальные баллоны.

В нашем опыте движущиеся молекулы газа непрерывно ударяют о стенки шарика внутри и снаружи. При откачивании воздуха число молекул в колоколе вокруг оболочки шарика уменьшается. Но внутри шарика их число не изменяется. Поэтому число ударов молекул о внешние стенки оболочки становится меньше, чем число ударов о внутренние стенки. Шарик раздувается до тех пор, пока сила упругости его резиновой оболочки не станет равной силе давления газа. Оболочка шарика принимает форму шара. Это показывает, что газ давит на ее стенки по всем направлениям одинаково
. Иначе говоря, число ударов молекул, приходящихся на каждый квадратный сантиметр площади поверхности, по всем направлениям одинаково. Одинаковое давление по всем направлениям характерно для газа и является следствием беспорядочного движения огромного числа молекул.

Попытаемся уменьшить объем газа, но так, чтобы масса его осталась неизменной. Это значит, что в каждом кубическом сантиметре газа молекул станет больше, плотность газа увеличится. Тогда число ударов молекул о стенки увеличится, т. е. возрастет давление газа. Это можно подтвердить опытом.

На рисунке а
изображена стеклянная трубка, один конец которой закрыт тонкой резиновой пленкой. В трубку вставлен поршень. При вдвигании поршня объем воздуха в трубке уменьшается, т. е. газ сжимается. Резиновая пленка при этом выгибается наружу, указывая на то, что давление воздуха в трубке увеличилось.

Наоборот, при увеличении объема этой же массы газа, число молекул в каждом кубическом сантиметре уменьшается. От этого уменьшится число ударов о стенки сосуда — давление газа станет меньше. Действительно, при вытягивании поршня из трубки объем воздуха увеличивается, пленка прогибается внутрь сосуда. Это указывает на уменьшение давления воздуха в трубке. Такие же явления наблюдались бы, если бы вместо воздуха в трубке находился бы любой другой газ.

Итак, при уменьшении объема газа его давление увеличивается, а при увеличении объема давление уменьшается при условии, что масса и температура газа остаются неизменными
.

А как изменится давление газа, если нагреть его при постоянном объеме? Известно, что скорость движения молекул газа при нагревании увеличивается. Двигаясь быстрее, молекулы будут ударять о стенки сосуда чаще. Кроме того, каждый удар молекулы о стенку будет сильнее. Вследствие этого, стенки сосуда будут испытывать большее давление.

Следовательно, давление газа в закрытом сосуде тем больше, чем выше температура газа
, при условии, что масса газа и объем не изменяются.

Из этих опытов можно сделать общий вывод, что давление газа тем больше, чем чаще и сильнее молекулы ударяют о стенки сосуда

.

Для хранения и перевозки газов их сильно сжимают. При этом давление их возрастает, газы необходимо заключать в специальные, очень прочные баллоны. В таких баллонах, например, содержат сжатый воздух в подводных лодках, кислород, используемый при сварке металлов. Конечно же, мы должны навсегда запомнить, что газовые баллоны нельзя нагревать, тем более, когда они заполнены газом. Потому что, как мы уже понимаем, может произойти взрыв с очень неприятными последствиями.

Закон Паскаля.

Давление передается в каждую точку жидкости или газа.

Давление поршня передается в каждую точку жидкости, заполняющей шар.

Теперь газ.

В отличие от твердых тел отдельные слои и мелкие частицы жидкости и газа могут свободно перемещаться относительно друг друга по всем направлениям. Достаточно, например, слегка подуть на поверхность воды в стакане, чтобы вызвать движение воды. На реке или озере при малейшем ветерке появляется рябь.

Подвижностью частиц газа и жидкости объясняется, что давление, производимое на них, передается не только в направлении действия силы, а в каждую точку
. Рассмотрим это явление подробнее.

На рисунке, а
изображен сосуд, в котором содержится газ (или жидкость). Частицы равномерно распределены по всему сосуду. Сосуд закрыт поршнем, который может перемещаться вверх и вниз.

Прилагая некоторую силу, заставим поршень немного переместиться внутрь и сжать газ (жидкость), находящийся непосредственно под ним. Тогда частицы (молекулы) расположатся в этом месте более плотно, чем прежде(рис, б). Благодаря подвижности частицы газа будут перемещаться по всем направлениям. Вследствие этого их расположение опять станет равномерным, но более плотным, чем раньше (рис, в). Поэтому давление газа всюду возрастет. Значит, добавочное давление передается всем частицам газа или жидкости. Так, если давление на газ (жидкость) около самого поршня увеличится на 1 Па, то во всех точках внутри
газа или жидкости давление станет больше прежнего на столько же. На 1 Па увеличится давление и на стенки сосуда, и на дно, и на поршень.

Давление, производимое на жидкость или газ, передается на любую точку одинаково во всех направлениях

.

Это утверждение называется законом Паскаля
.

На основе закона Паскаля легко объяснить следующие опыты.

На рисунке изображен полый шар, имеющий в различных местах небольшие отверстия. К шару присоединена трубка, в которую вставлен поршень. Если набрать воды в шар и вдвинуть в трубку поршень, то вода польется из всех отверстий шара. В этом опыте поршень давит на поверхность воды в трубке. Частицы воды, находящиеся под поршнем, уплотняясь, передают его давление другим слоям, лежащим глубже. Таким образом, давление поршня передается в каждую точку жидкости, заполняющей шар. В результате часть воды выталкивается из шара в виде одинаковых струек, вытекающих из всех отверстий.

Если шар заполнить дымом, то при вдвигании поршня в трубку из всех отверстий шара начнут выходить одинаковые струйки дыма. Это подтверждает, что и газы передают производимое на них давление во все стороны одинаково
.

Давление в жидкости и газе.

Под действием веса жидкости резиновое дно в трубке прогнется.

На жидкости, как и на все тела на Земле, действует сила тяжести. Поэтому, каждый слой жидкости, налитой в сосуд, своим весом создает давление, которое по закону Паскаля передается по всем направлениям. Следовательно, внутри жидкости существует давление. В этом можно убедиться на опыте.

В стеклянную трубку, нижнее отверстие которой закрыто тонкой резиновой пленкой, нальем воду. Под действием веса жидкости дно трубки прогнется.

Опыт показывает, что, чем выше столб воды над резиновой пленкой, тем больше она прогибается. Но всякий раз после того, как резиновое дно прогнулось, вода в трубке приходит в равновесие (останавливается), так как, кроме силы тяжести, на воду действует сила упругости растянутой резиновой пленки.

Силы, действующие на резиновую пленку,

одинаковы с обеих сторон.

Иллюстрация.

Дно отходит от цилиндра вследствие давления на него силы тяжести.

Опустим трубку с резиновым дном, в которую налита вода, в другой, более широкий сосуд с водой. Мы увидим, что по мере опускания трубки резиновая пленка постепенно выпрямляется. Полное выпрямление пленки показывает, что силы, действующие на нее сверху и снизу, равны. Наступает полное выпрямление пленки тогда, когда уровни воды в трубке и сосуде совпадают.

Такой же опыт можно провести с трубкой, в которой резиновая пленка закрывает боковое отверстие, как это показано на рисунке, а. Погрузим эту трубку с водой в другой сосуд с водой, как это изображено на рисунке, б
. Мы заметим, что пленка снова выпрямится, как только уровни воды в трубке и сосуде сравняются. Это означает, что силы, действующие на резиновую пленку, одинаковы со всех сторон.

Возьмем сосуд, дно которого может отпадать. Опустим его в банку с водой. Дно при этом окажется плотно прижатым к краю сосуда и не отпадет. Его прижимает сила давления воды, направленная снизу вверх.

Будем осторожно наливать воду в сосуд и следить за его дном. Как только уровень воды в сосуде совпадет с уровнем воды в банке, оно отпадет от сосуда.

В момент отрыва на дно давит сверху вниз столб жидкости в сосуде, а снизу вверх на дно передается давление такого же по высоте столба жидкости, но находящейся в банке. Оба эти давления одинаковы, дно же отходит от цилиндра вследствие действия на него собственной силы тяжести.

Выше были описаны опыты с водой, но если взять вместо воды любую другую жидкость, результаты опыта будут те же.

Итак, опыты показывают, что внутри жидкости существует давление, и на одном и том же уровне оно одинаково по всем направлениям. С глубиной давление увеличивается
.

Газы в этом отношении не отличаются от жидкостей, ведь они тоже имеют вес. Но надо помнить, что плотность газа в сотни раз меньше плотности жидкости. Вес газа, находящегося в сосуде, мал, и его «весовое» давление во многих случаях можно не учитывать.

Расчет давления жидкости на дно и стенки сосуда.

Расчет давления жидкости на дно и стенки сосуда.

Рассмотрим, как можно рассчитывать давление жидкости на дно и стенки сосуда. Решим сначала задачу для сосуда, имеющего форму прямоугольного параллелепипеда.

Сила F
, с которой жидкость, налитая в этот сосуд, давит на его дно, равна весу P
жидкости, находящейся в сосуде. Вес жидкости можно определить, зная ее массу m
. Массу, как известно, можно вычислить по формуле: m = ρ·V
. Объем жидкости, налитой в выбранный нами сосуд, легко рассчитать. Если высоту столба жидкости, находящейся в сосуде, обозначить буквой h
, а площадь дна сосуда S
, то V = S·h
.

Масса жидкости m = ρ·V
, или m = ρ·S·h
.

Вес этой жидкости P = g·m
, или P = g·ρ·S·h
.

Так как вес столба жидкости равен силе, с которой жидкость давит на дно сосуда, то, разделив вес P
на площадь S
, получим давление жидкости p
:

p = P/S , или p = g·ρ·S·h/S,

Мы получили формулу для расчета давления жидкости на дно сосуда. Из этой формулы видно, что давление жидкости на дно сосуда зависит только от плотности и высоты столба жидкости
.

Следовательно, по выведенной формуле можно рассчитывать давление жидкости, налитой в сосуд любой формы
(строго говоря, наш расчет годится только для сосудов, имеющих форму прямой призмы и цилиндра. В курсах физики для института доказано, что формула верна и для сосуда произвольной формы). Кроме того, по ней можно вычислить и давление на стенки сосуда. Давление внутри жидкости, в том числе давление снизу вверх, также рассчитывается по этой формуле, так как давление на одной и той же глубине одинаково по всем направлениям.

При расчете давления по формуле p = gρh
надо плотность ρ
выражать в килограммах на кубический метр (кг/м 3), а высоту столба жидкости h
— в метрах (м), g
= 9,8 Н/кг, тогда давление будет выражено в паскалях (Па).

Пример
. Определите давление нефти на дно цистерны, если высота столба нефти 10 м, а плотность ее 800 кг/м 3 .

Запишем условие задачи и запишем ее.

Дано

:

ρ = 800 кг/м 3

Решение

:

p = 9.8 Н/кг · 800 кг/м 3 · 10 м ≈ 80 000 Па ≈ 80 кПа.

Ответ

: p ≈ 80 кПа.

Сообщающиеся сосуды.

Сообщающиеся сосуды.

На рисунке изображены два сосуда, соединённые между собой резиновой трубкой. Такие сосуды называются сообщающимися
. Лейка, чайник, кофейник — примеры сообщающихся сосудов. Из опыта мы знаем, что вода, налитая, например, в лейку, стоит всегда на одном уровне в носике и внутри.

Сообщающиеся сосуды встречаются нам часто. Например, им может быть чайник, лейка или кофейник.

Поверхности однородной жидкости устанавливаются на одном уровне в сообщающихся сосудах любой формы.

Разные по плотности жидкости.

С сообщающимися сосудами можно проделать следующий простой опыт. В начале опыта резиновую трубку зажимаем в середине, и в одну из трубок наливаем воду. Затем зажим открываем, и вода вмиг перетекает в другую трубку, пока поверхности воды в обеих трубках не установятся на одном уровне. Можно закрепить одну из трубок в штативе, а другую поднимать, опускать или наклонять в разные стороны. И в этом случае, как только жидкость успокоится, ее уровни в обеих трубках уравняются.

В сообщающихся сосудах любой формы и сечения поверхности однородной жидкости устанавливаются на одном уровне
(при условии, что давление воздуха над жидкостью одинаково) (рис. 109).

Это можно обосновать следующим образом. Жидкость покоится, не перемещаясь из одного сосуда в другой. Значит, давления в обоих сосудах на любом уровне одинаковы. Жидкость в обоих сосудах одна и та же, т. е. имеет одинаковую плотность. Следовательно, должны быть одинаковы и ее высоты. Когда мы поднимаем один сосуд или доливаем в него жидкость, давление в нем увеличивается и жидкость перемещается в другой сосуд до тех пор, пока давления не уравновесятся.

Если в один из сообщающихся сосудов налить жидкость одной плотности, а во второй — другой плотности, то при равновесии уровни этих жидкостей не будут одинаковыми. И это понятно. Мы ведь знаем, что давление жидкости на дно сосуда прямо пропорционально высоте столба и плотности жидкости. А в этом случае плотности жидкостей будут различны.

При равенстве давлений высота столба жидкости с большей плотностью будет меньше высоты столба жидкости с меньшей плотностью (рис.).

Опыт. Как определить массу воздуха.

Вес воздуха. Атмосферное давление.

Существование атмосферного давления.

Атмосферное давление больше, чем давление разреженного воздуха в сосуде.

На воздух, как и на всякое тело, находящееся на Земле, действует сила тяжести, и, значит, воздух обладает весом. Вес воздуха легко вычислить, зная его массу.

На опыте покажем, как вычислить массу воздуха. Для этого нужно взять прочный стеклянный шар с пробкой и резиновой трубкой с зажимом. Выкачаем из него насосом воздух, зажмем трубку зажимом и уравновесим на весах. Затем, открыв зажим на резиновой трубке, впустим в него воздух. Равновесие весов при этом нарушится. Для его восстановления на другую чашку весов придется положить гири, масса которых будет равна массе воздуха в объеме шара.

Опытами установлено, что при температуре 0 °С и нормальном атмосферном давлении масса воздуха объемом 1 м 3 равна 1,29 кг. Вес этого воздуха легко вычислить:

P = g·m, P = 9,8 Н/кг · 1,29 кг ≈ 13 Н.

Воздушная оболочка, окружающая Землю, называется атмосфера

(от греч. атмос
— пар, воздух, и сфера
— шар).

Атмосфера, как показали наблюдения за полетом искусственных спутников Земли, простирается на высоту нескольких тысяч километров.

Вследствие действия силы тяжести верхние слои атмосферы, подобно воде океана, сжимают нижние слои. Воздушный слой, прилегающий непосредственно к Земле, сжат больше всего и, согласно закону Паскаля, передает производимое на него давление по всем направлениям.

В результате этого земная поверхность и телá, находящиеся на ней, испытывают давление всей толщи воздуха, или, как обычно говорится в таких случаях, испытывают атмосферное давление

.

Существованием атмосферного давления могут быть объяснены многие явления, с которыми мы встречаемся в жизни. Рассмотрим некоторые из них.

На рисунке изображена стеклянная трубка, внутри которой находится поршень, плотно прилегающий к стенкам трубки. Конец трубки опущен воду. Если поднимать поршень, то за ним будет подниматься и вода.

Это явление используется в водяных насосах и некоторых других устройствах.

На рисунке показан цилиндрический сосуд. Он закрыт пробкой, в которую вставлена трубка с краном. Из сосуда насосом откачивается воздух. Затем конец трубки помещается в воду. Если теперь открыть кран, то вода фонтаном брызнет в внутрь сосуда. Вода поступает в сосуд потому, что атмосферное давление больше давления разреженного воздуха в сосуде.

Почему существует воздушная оболочка Земли.

Как и все тела, молекулы газов, входящих в состав воздушной оболочки Земли, притягиваются к Земле.

Но почему же тогда все они не упадут на поверхность Земли? Каким образом сохраняется воздушная оболочка Земли, ее атмосфера? Чтобы понять это, надо учесть, что молекулы газов находятся в непрерывном и беспорядочном движении. Но тогда возникает другой вопрос: почему эти молекулы не улетают в мировое пространство, то есть в космос.

Для того, чтобы совсем покинуть Землю, молекула, как и космический корабль или ракета, должна иметь очень большую скорость (не меньше 11,2 км/с). Это так называемая вторая космическая скорость
. Скорость большинства молекул воздушной оболочки Земли значительно меньше этой космической скорости. Поэтому большинство их привязано к Земле силой тяжести, лишь ничтожно малое количество молекул улетает за пределы Земли в космос.

Беспорядочное движение молекул и действие на них силы тяжести приводят в результате к тому, что молекулы газов «парят» в пространстве около Земли, образуя воздушную оболочку, или известную нам атмосферу.

Измерения показывают, что плотность воздуха быстро уменьшается с высотой. Так, на высоте 5,5 км над Землей плотность воздуха в 2 раза меньше его плотность у поверхности Земли, на высоте 11 км — в 4 раза меньше, и т. д. Чем выше, тем воздух разреженнее. И наконец, в самых верхних слоях (сотни и тысячи километров над Землей) атмосфера постепенно переходит в безвоздушное пространство. Четкой границы воздушная оболочка Земли не имеет.

Строго говоря, вследствие действия силы тяжести плотность газа в любом закрытом сосуде неодинакова по всему объему сосуда. Внизу сосуда плотность газа больше, чем в верхних его частях, поэтому и давление в сосуде неодинаково. На дне сосуда оно больше, чем вверху.
Однако для газа, содержащегося в сосуде, это различие в плотности и давлении столь мало, что его можно во многих случаях совсем не учитывать, просто знать об этом. Но для атмосферы, простирающейся на несколько тысяч километров, различие это существенно.

Измерение атмосферного давления. Опыт Торричелли.

Рассчитать атмосферное давление по формуле для вычисления давления столба жидкости (§ 38) нельзя. Для такого расчета надо знать высоту атмосферы и плотность воздуха. Но определенной границы у атмосферы нет, а плотность воздуха на разной высоте различна. Однако измерить атмосферное давление можно с помощью опыта, предложенного в 17 веке итальянским ученым Эванджелиста Торричелли

, учеником Галилея.

Опыт Торричелли состоит в следующем: стеклянную трубку длиной около 1 м, запаянную с одного конца, наполняют ртутью. Затем, плотно закрыв второй конец трубки, ее переворачивают и опускают в чашку с ртутью, где под уровнем ртути открывают этот конец трубки. Как и в любом опыте с жидкостью, часть ртути при этом выливается в чашку, а часть ее остается в трубке. Высота столба ртути, оставшейся в трубке, равна примерно 760 мм. Над ртутью внутри трубки воздуха нет, там безвоздушное пространство, поэтому никакой газ не оказывает давления сверху на столб ртути внутри этой трубки и не влияет на измерения.

Торричелли, предложивший описанный выше опыт, дал и его объяснение. Атмосфера давит на поверхность ртути в чашке. Ртуть находится в равновесии. Значит, давление в трубке на уровне аа
1 (см. рис) равно атмосферному давлению. При изменении атмосферного давления меняется и высота столба ртути в трубке. При увеличении давления столбик удлиняется. При уменьшении давления — столб ртути уменьшает свою высоту.

Давление в трубке на уровне аа1 создается весом столба ртути в трубке, так как в верхней части трубки над ртутью воздуха нет. Отсюда следует, что атмосферное давление равно давлению столба ртути в трубке

, т. е.

p
атм = p
ртути.

Чем больше атмосферное давление, тем выше столб ртути в опыте Торричелли. Поэтому на практике атмосферное давление можно измерить высотой ртутного столба (в миллиметрах или сантиметрах). Если, например, атмосферное давление равно 780 мм рт. ст. (говорят «миллиметров ртутного столба»), то это значит, что воздух производит такое же давление, какое производит вертикальный столб ртути высотой 780 мм.

Следовательно, в этом случае за единицу измерения атмосферного давления принимается 1 миллиметр ртутного столба (1 мм рт. ст.). Найдем соотношение между этой единицей и известной нам единицей — паскалем
(Па).

Давление столба ртути ρ ртути высотой 1 мм равно:

p
= g·ρ·h
, p
= 9,8 Н/кг · 13 600 кг/ м 3 · 0,001 м ≈ 133,3 Па.

Итак, 1 мм рт. ст. = 133,3 Па.

В настоящее время атмосферное давление принято измерять в гектопаскалях (1 гПа = 100 Па). Например, в сводках погоды может быть объявлено, что давление равно 1013 гПа, это то же самое, что 760 мм рт. ст.

Наблюдая ежедневно за высотой ртутного столба в трубке, Торричелли обнаружил, что эта высота меняется, т. е. атмосферное давление непостоянно, оно может увеличиваться и уменьшаться. Торричелли заметил также, что атмосферное давление связано с изменением погоды.

Если к трубке с ртутью, использовавшейся в опыте Торричелли, прикрепить вертикальную шкалу, то получится простейший прибор — ртутный барометр

(от греч. барос
— тяжесть, метрео
— измеряю). Он служит для измерения атмосферного давления.

Барометр — анероид.

В практике для измерения атмосферного давления используют металлический барометр, называемый анероидом

(в переводе с греческого — безжидкостный
). Так барометр называют потому, что в нем нет ртути.

Внешний вид анероида изображен на рисунке. Главная часть его — металлическая коробочка 1 с волнистой (гофрированной) поверхностью (см. др. рис.). Из этой коробочки выкачан воздух, а чтобы атмосферное давление не раздавило коробочку, ее крышка 2 пружиной оттягивается вверх. При увеличении атмосферного давления крышка прогибается вниз и натягивает пружину. При уменьшении давления пружина выпрямляет крышку. К пружине с помощью передаточного механизма 3 прикреплена стрелка-указатель 4, которая продвигается вправо или влево при изменении давления. Под стрелкой укреплена шкала, деления которой нанесены по показаниям ртутного барометра. Так, число 750, против которого стоит стрелка анероида (см. рис.), показывает, что в данный момент в ртутном барометре высота ртутного столба 750 мм.

Следовательно, атмосферное давление равно 750 мм рт. ст. или ≈ 1000 гПа.

Значение атмосферного давления весьма важно для предвидения погоды на ближайшие дни, так как изменение атмосферного давления связано с изменением погоды. Барометр — необходимый прибор для метеорологических наблюдений.

Атмосферное давление на различных высотах.

В жидкости давление, как мы знаем, зависит от плотности жидкости и высоты ее столба. Вследствие малой сжимаемости плотность жидкости на различных глубинах почти одинакова. Поэтому, вычисляя давление, мы считаем ее плотность постоянной и учитываем только изменение высоты.

Сложнее дело обстоит с газами. Газы сильно сжимаемы. А чем сильнее газ сжат, тем больше его плотность, и тем большее давление он производит. Ведь давление газа создается ударами его молекул о поверхность тела.

Слои воздуха у поверхности Земли сжаты всеми вышележащими слоями воздуха, находящимися над ними. Но чем выше от поверхности слой воздуха, тем слабее он сжат, тем меньше его плотность. Следовательно, тем меньшее давление он производит. Если, например, воздушный шар поднимается над поверхностью Земли, то давление воздуха на шар становиться меньше. Это происходит не только потому, что высота столба воздуха над ним уменьшается, но еще и потому, что уменьшается плотность воздуха. Вверху она меньше, чем внизу. Поэтому зависимость давления воздуха от высоты сложнее, чем жидкости.

Наблюдения показывают, что атмосферное давление в местностях, лежащих на уровне моря, в среднем равно 760 мм рт. ст.

Атмосферное давление, равное давлению столба ртути высотой 760 мм при температуре 0 °С, называется нормальным атмосферным давлением
.

Нормальное атмосферное давление
равно 101 300 Па = 1013 гПа.

Чем больше высота над уровнем моря, тем давление меньше.

При небольших подъемах, в среднем, на каждые 12 м подъема давление уменьшается на 1 мм рт. ст. (или на 1,33 гПа).

Зная зависимость давления от высоты, можно по изменению показаний барометра определить высоту над уровнем моря. Анероиды, имеющие шкалу, по которой непосредственно можно измерить высоту над уровнем моря, называются высотомерами

. Их применяют в авиации и при подъеме на горы.

Манометры.

Мы уже знаем, что для измерения атмосферного давления применяют барометры. Для измерения давлений, бóльших или меньших атмосферного, используется манометры

(от греч. манос
— редкий, неплотный, метрео
— измеряю). Манометры бывают жидкостные
и металлические
.

Рассмотрим сначала устройство и действие открытого жидкостного манометра
. Он состоит из двухколенной стеклянной трубки, в которую наливается какая-нибудь жидкость. Жидкость устанавливается в обоих коленах на одном уровне, так как на ее поверхность в коленах сосуда действует только атмосферное давление.

Чтобы понять, как работает такой манометр, его можно соединить резиновой трубкой с круглой плоской коробкой, одна сторона которой затянута резиновой пленкой. Если надавить пальцем на пленку, то уровень жидкости в колене манометра, соединенном в коробкой, понизится, а в другом колене повысится. Чем это объясняется?

При надавливании на пленку увеличивается давление воздуха в коробке. По закону Паскаля это увеличение давления передается и жидкости в том колене манометра, которое присоединено к коробке. Поэтому давление на жидкость в этом колене будет больше, чем в другом, где на жидкость действует только атмосферное давление. Под действием силы этого избыточного давления жидкость начнет перемещаться. В колене со сжатым воздухом жидкость опустится, в другом — поднимется. Жидкость придет в равновесие (остановится), когда избыточное давление сжатого воздуха уравновесится давлением, которое производит избыточный столб жидкости в другом колене манометра.

Чем сильнее давить на пленку, тем выше избыточный столб жидкости, тем больше его давление. Следовательно, об изменении давления можно судить по высоте этого избыточного столба
.

На рисунке показано, как таким манометром можно измерять давление внутри жидкости. Чем глубже погружается в жидкость трубочка, тем больше становится разность высот столбов жидкости в коленах манометра
, тем, следовательно, и большее давление производит жидкость
.

Если установить коробочку прибора на какой-нибудь глубине внутри жидкости и поворачивать ее пленкой вверх, вбок и вниз, то показания манометра при этом не будут меняется. Так и должно быть, ведь на одном и том же уровне внутри жидкости давление одинаково по всем направлениям
.

На рисунке изображен металлический манометр

. Основная часть такого манометра — согнутая в трубу металлическая трубка 1
, один конец которой закрыт. Другой конец трубки с помощью крана 4
сообщается с сосудом, в котором измеряют давление. При увеличении давления трубка разгибается. Движение её закрытого конца при помощи рычага 5
и зубчатки 3
передается стрелке 2
, движущейся около шкалы прибора. При уменьшении давления трубка, благодаря своей упругости, возвращается в прежнее положение, а стрелка — к нулевому делению шкалы.

Поршневой жидкостный насос.

В опыте, рассмотренном нами ранее (§ 40), было установлено, что вода в стеклянной трубке под действием атмосферного давления поднималась вверх за поршнем. На этом основано действие поршневых
насосов.

Насос схематически изображен на рисунке. Он состоит из цилиндра, внутри которого ходит вверх и вниз, плотно прилегая к стенкам сосуда, поршень 1
. В нижней части цилиндра и в самом поршне установлены клапаны 2
, открывающиеся только вверх. При движении поршня вверх вода под действием атмосферного давления входит в трубу, поднимает нижний клапан и движется за поршнем.

При движении поршня вниз вода, находящаяся под поршнем, давит на нижний клапан, и он закрывается. Одновременно под давлением воды открывается клапан внутри поршня, и вода переходит в пространство над поршнем. При следующем движении поршня вверх в месте с ним поднимается и находящаяся над ним вода, которая и выливается в отводящую трубу. Одновременно за поршнем поднимается и новая порция воды, которая при последующем опускании поршня окажется над ним, и вся эта процедура повторяется вновь и вновь, пока работает насос.

Гидравлический пресс.

Закон Паскаля позволяет объяснить действие гидравлической машины

(от греч. гидравликос
— водяной). Это машины, действие которых основано на законах движения и равновесия жидкостей.

Основной частью гидравлической машины служат два цилиндра разного диаметра, снабженные поршнями и соединительной трубкой. Пространство под поршнями и трубку заполняют жидкостью (обычно минеральным маслом). Высоты столбов жидкости в обоих цилиндрах одинаковы, пока на поршни не действуют силы.

Допустим теперь, что силы F
1 и F
2 — силы, действующие на поршни, S
1 и S
2 — площади поршней. Давление под первым (малым) поршнем равно p
1 = F
1 / S
1 , а под вторым (большим) p
2 = F
2 / S
2 . По закону Паскаля давление покоящейся жидкостью во все стороны передается одинаково, т. е. p
1 = p
2 или F
1 / S
1 = F
2 / S
2 , откуда:

F
2 / F
1 = S
2 / S
1 .

Следовательно, сила F
2 во столько раз больше силы
F
1 , во сколько раз площадь большого поршня больше площади малого поршня
. Например, если площадь большого поршня 500 см 2 , а малого 5 см 2 , и на малый поршень действует сила 100 Н, то на больший поршень будет действовать сила, в 100 раз бóльшая, то есть 10 000 Н.

Таким образом, с помощью гидравлической машины можно малой силой уравновесить бóльшую силу.

Отношение F
1 / F
2 показывает выигрыш в силе. Например, в приведенном примере выигрыш в силе равен 10 000 Н / 100 Н = 100.

Гидравлическая машина, служащая для прессования (сдавливания), называется гидравлическим прессом

.

Гидравлические прессы применяются там, где требуется большая сила. Например, для выжимания масла из семян на маслобойных заводах, для прессования фанеры, картона, сена. На металлургических заводах гидравлические прессы используют для изготовления стальных валов машин, железнодорожных колес и многих других изделий. Современные гидравлические прессы могут развивать силу в десятки и сотни миллионов ньютонов.

Устройство гидравлического пресса схематически показано на рисунке. Прессуемое тело 1 (A) кладут на платформу, соединенную с большим поршнем 2 (B). При помощи малого поршня 3 (D) создается большое давление на жидкость. Это давление передается в каждую точку жидкости, заполняющей цилиндры. Поэтому такое же давление действует и на второй, большой поршень. Но так как площадь 2-го (большого) поршня больше площади малого, то и сила, действующая на него, будет больше силы, действующей на поршень 3 (D). Под действием этой силы поршень 2 (B) будет подниматься. При подъеме поршня 2 (B) тело (A) упирается в неподвижную верхнюю платформу и сжимается. При помощи манометра 4 (M) измеряется давление жидкости. Предохранительный клапан 5 (P) автоматически открывается, когда давление жидкости превышает допустимое значение.

Из малого цилиндра в большой жидкость перекачивается повторными движениями малого поршня 3 (D). Это осуществляется следующим образом. При подъеме малого поршня (D) клапан 6 (K) открывается, и в пространство, находящееся под поршнем, засасывается жидкость. При опускании малого поршня под действием давления жидкости клапан 6 (K) закрывается, а клапан 7 (K») открывается, и жидкость переходит в большой сосуд.

Действие воды и газа на погруженное в них тело.

Под водой мы легко можем поднять камень, который с трудом поднимается в воздухе. Если погрузить пробку под воду и выпустить ее из рук, то она всплывет. Как можно объяснить эти явления?

Мы знаем (§ 38), что жидкость давит на дно и стенки сосуда. И если внутрь жидкости поместить какое-нибудь твердое тело, то оно также будет подвергаться давлению, как и стенки сосуда.

Рассмотрим силы, которые действуют со стороны жидкости на погруженное в нее тело. Чтобы легче было рассуждать, выберем тело, которое имеет форму параллелепипеда с основаниями, параллельными поверхности жидкости (рис.). Силы, действующие на боковые грани тела, попарно равны и уравновешивают друг друга. Под действием этих сил тело сжимается. А вот силы, действующие на верхнюю и нижнюю грани тела, неодинаковы. На верхнюю грань давит сверху силой F
1 столб жидкости высотой h
1 . На уровне нижней грани давление производит столб жидкости высотой h
2 . Это давление, как мы знаем (§ 37), передается внутри жидкости во все стороны. Следовательно, на нижнюю грань тела снизу вверх с силой F
2 давит столб жидкости высотой h
2 . Но h
2 больше h
1 , следовательно, и модуль силы F
2 больше модуля силы F
1 . Поэтому тело выталкивается из жидкости с силой F
выт, равной разности сил F
2 — F
1 , т. е.

Но S·h = V, где V — объем параллелепипеда, а ρ ж ·V = m ж — масса жидкости в объеме параллелепипеда. Следовательно,

F выт = g·m ж = P ж,

т. е. выталкивающая сила равна весу жидкости в объеме погруженного в нее тела
(выталкивающая сила равна весу жидкости такого же объёма, как и объём погруженного в нее тела).

Существование силы, выталкивающей тело из жидкости, легко обнаружить на опыте.

На рисунке а
изображено тело, подвешенное к пружине со стрелкой-указателем на конце. Стрелка отмечает на штативе растяжение пружины. При отпускании тела в воду пружина сокращается (рис., б
). Такое же сокращение пружины получится, если действовать на тело снизу вверх с некоторой силой, например, нажать рукой (приподнять).

Следовательно, опыт подтверждает, что на тело, находящееся в жидкости, действует сила, выталкивающая это тело из жидкости
.

К газам, как мы знаем, также применим закон Паскаля. Поэтому на тела, находящиеся в газе, действует сила, выталкивающая их из газа
. Под действием этой силы воздушные шары поднимаются вверх. Существование силы, выталкивающей тело из газа, можно также наблюдать на опыте.

К укороченной чашке весов подвесим стеклянный шар или большую колбу, закрытую пробкой. Весы уравновешиваются. Затем под колбу (или шар) ставят широкий сосуд так, чтобы он окружал всю колбу. Сосуд наполняется углекислым газом, плотность которого больше плотности воздуха (поэтому углекислый газ опускается вниз и заполняет сосуд, вытесняя из него воздух). При этом равновесие весов нарушается. Чашка с подвешенной колбой поднимается вверх (рис.). На колбу, погруженную в углекислый газ, действует бóльшая выталкивающая сила, по сравнению с той, которая действует на нее в воздухе.

Сила, выталкивающая тело из жидкости или газа, направлена противоположно силе тяжести, приложенной к этому телу
.

Поэтому пролкосмосе). Именно этим объясняется, что в воде мы иногда легко поднимаем тела, которые с трудом удерживаем в воздухе.

К пружине подвешивается небольшое ведерко и тело цилиндрической формы (рис., а). Стрелка на штативе отмечает растяжение пружины. Она показывает вес тела в воздухе. Приподняв тело, под него подставляется отливной сосуд, наполненный жидкостью до уровня отливной трубки. После чего тело погружается целиком в жидкость (рис., б). При этом часть жидкости, объем которой равен объему тела, выливается
из отливного сосуда в стакан. Пружина сокращается, и указатель пружины поднимается вверх, показывая уменьшение веса тела в жидкости. В данном случае на тело, кроме силы тяжести, действует еще одна сила, выталкивающая его из жидкости. Если в верхнее ведерко вылить жидкость из стакана (т. е. ту, которую вытеснило тело), то указатель пружины возвратится к своему начальному положению (рис., в).

На основании этого опыта можно заключить, что сила, выталкивающая целиком погруженное в жидкость тело, равна весу жидкости в объеме этого тела

. Такой же вывод мы получили и в § 48.

Если подобный опыт проделать с телом, погруженным в какой-либо газ, то он показал бы, что сила, выталкивающая тело из газа, также равна весу газа, взятого в объеме тела

.

Сила, выталкивающая тело из жидкости или газа, называется архимедовой силой
, в честь ученого Архимеда

, который впервые указал на ее существование и рассчитал ее значение.

Итак, опыт подтвердил, что архимедова (или выталкивающая) сила равна весу жидкости в объеме тела, т. е. F
А = P
ж = g·m
ж. Массу жидкости m ж, вытесняемую телом, можно выразить через ее плотность ρ ж и объем тела V т, погруженного в жидкость (так как V ж — объем вытесненной телом жидкости равен V т — объему тела, погруженного в жидкость), т. е. m ж = ρ ж ·V т. Тогда получим:

F
A = g·ρ
ж ·V
т

Следовательно, архимедова сила зависит от плотности жидкости, в которую погружено тело, и от объема этого тела. Но она не зависит, например, от плотности вещества тела, погружаемого в жидкость, так как эта величина не входит в полученную формулу.

Определим теперь вес тела, погруженного в жидкость (или в газ). Так как две силы, действующие на тело в этом случае, направлены в противоположные стороны (сила тяжести вниз, а архимедова сила вверх), то вес тела в жидкости P 1 будет меньше веса тела в вакууме P = g·m
на архимедову силу F
А = g·m
ж (где m
ж — масса жидкости или газа, вытесненной телом).

Таким образом, если тело погружено в жидкость или газ, то оно теряет в своем весе столько, сколько весит вытесненная им жидкость или газ
.

Пример
. Определить выталкивающую силу, действующую на камень объемом 1,6 м 3 в морской воде.

Запишем условие задачи и решим ее.

Когда всплывающее тело достигнет поверхности жидкости, то при дальнейшем его движении вверх архимедова сила будет уменьшаться. Почему? А потому, что будет уменьшаться объем части тела, погруженной в жидкость, а архимедова сила равна весу жидкости в объеме погруженной в нее части тела.

Когда архимедова сила станет равной силе тяжести, тело остановится и будет плавать на поверхности жидкости, частично погрузившись в нее.

Полученный вывод легко проверить на опыте.

В отливной сосуд нальем воду до уровня отливной трубки. После этого погрузим в сосуд плавающее тело, предварительно взвесив его в воздухе. Опустившись в воду, тело вытесняет объем воды, равный объему погруженной в нее части тела. Взвесив эту воду, находим, что ее вес (архимедова сила) равен силе тяжести, действующей на плавающее тело, или весу этого тела в воздухе.

Проделав такие же опыты с любыми другими телами, плавающими в разных жидкостях — в воде, спирте, растворе соли, можно убедиться, что если тело плавает в жидкости, то вес вытесненной им жидкости равен весу этого тела в воздухе
.

Легко доказать, что если плотность сплошного твердого тела больше плотности жидкости, то тело в такой жидкости тонет. Тело с меньшей плотностью всплывает в этой жидкости
. Кусок железа, например, тонет в воде, но всплывает в ртути. Тело же, плотность которого равна плотности жидкости, остается в равновесии внутри жидкости.

Плавает на поверхности воды лед, так как его плотность меньше плотности воды.

Чем меньше плотность тела по сравнению с плотностью жидкости, тем меньшая часть тела погружена в жидкость

.

При равных плотностях тела и жидкости тело плавает внутри жидкости на любой глубине.

Две несмешивающиеся жидкости, например вода и керосин, располагаются в сосуде в соответствии со своими плотностями: в нижней части сосуда — более плотная вода (ρ = 1000 кг/м 3), сверху — более легкий керосин (ρ = 800 кг/м 3).

Средняя плотность живых организмов, населяющих водную среду, мало отличается от плотности воды, поэтому их вес почти полностью уравновешивается архимедовой силой. Благодаря этому водные животные не нуждаются в столь прочных и массивных скелетах, как наземные. По этой же причине эластичны стволы водных растений.

Плавательный пузырь рыбы легко меняет свой объем. Когда рыба с помощью мышц опускается на большую глубину, и давление воды на нее увеличивается, пузырь сжимается, объем тела рыбы уменьшается, и она не выталкивается вверх, а плавает в глубине. Таким образом, рыба может в определенных пределах регулировать глубину своего погружения. Киты регулируют глубину своего погружения за счет уменьшения и увеличения объема легких.

Плавание судов.

Суда, плавающие по рекам, озерам, морям и океанам, построены из разных материалов с различной плотностью. Корпус судов обычно делается из стальных листов. Все внутренние крепления, придающие судам прочность, также изготовляют из металлов. Для постройки судов используют различные материалы, имеющие по сравнению с водой как бóльшие, так и меньшие плотности.

Благодаря чему суда держатся на воде, принимают на борт и перевозят большие грузы?

Опыт с плавающим телом (§ 50) показал, что тело вытесняет своей подводной частью столько воды, что по весу эта вода равна весу тела в воздухе. Это также справедливо и для любого судна.

Вес воды, вытесняемой подводной частью судна, равен весу судна с грузом в воздухе или силе тяжести, действующей на судно с грузом
.

Глубина, на которую судно погружается в воду, называется осадкой

. Наибольшая допускаемая осадка отмечена на корпусе судна красной линией, называемой ватерлинией

(от голланд. ватер
— вода).

Вес воды, вытесняемой судном при погружении до ватерлинии, равный силе тяжести, действующей на судно с грузом, называется водоизмещением судна
.

В настоящее время для перевозки нефти строятся суда водоизмещением 5 000 000 кН (5 · 10 6 кН) и больше, т. е. имеющие вместе с грузом массу 500 000 т (5 · 10 5 т) и более.

Если из водоизмещения вычесть вес самого судна, то мы получим грузоподъемность этого судна. Грузоподъемность показывает вес груза, перевозимого судном.

Судостроение существовало еще в Древнем Египте, в Финикии (считается, что Финикийцы были одними из лучших судостроителей), Древнем Китае.

В России судостроение зародилось на рубеже 17-18 вв. Сооружались главным образом военные корабли, но именно в России были построены первый ледокол, суда с двигателем внутреннего сгорания, атомный ледокол «Арктика».

Воздухоплавание.

Рисунок с описанием шара братьев Монгольфье 1783 года: «Вид и точные размеры „Аэростата Земной шар“, который был первым». 1786

С давних времен люди мечтали о возможности летать над облаками, плавать в воздушном океане, как они плавали по морю. Для воздухоплавания

вначале использовали воздушные шары, которые наполняли или нагретым воздухом, или водородом либо гелием.

Для того, чтобы воздушный шар поднялся в воздух, необходимо, чтобы архимедова сила (выталкивающая) F
А, действующая на шар, была больше силы тяжести F
тяж, т. е. F
А > F
тяж.

По мере поднятия шара вверх архимедова сила, действующая на него, уменьшается (F
А = gρV
), так как плотность верхних слоев атмосферы меньше, чем у поверхности Земли. Чтобы подняться выше, с шара сбрасывается специальный балласт (груз) и этим облегчает шар. В конце концов шар достигает своей своей предельной высоты подъема. Для спуска шара из его оболочки при помощи специального клапана выпускается часть газа.

В горизонтальном направлении воздушный шар перемещается только под действием ветра, поэтому он называется аэростатом

(от греч аэр
— воздух, стато
— стоящий). Для исследования верхних слоев атмосферы, стратосферы еще не так давно применялись огромные воздушные шары — стратостаты

.

До того как научились строить большие самолеты для перевозки по воздуху пассажиров и грузов, применялись управляемые аэростаты — дирижабли
. Они имеют удлиненную форму, под корпусом подвешивается гондола с двигателем, который приводит в движение пропеллер.

Воздушный шар не только сам поднимается вверх, но может поднять и некоторый груз: кабину, людей, приборы. Поэтому для того, чтобы узнать, какой груз может поднять воздушный шар, необходимо определить его подъемную силу
.

Пусть, например, в воздух запущен шар объемом 40 м 3 , наполненный гелием. Масса гелия, заполняющая оболочку шара, будет равна:
m Ге = ρ Ге ·V = 0,1890 кг/м 3 · 40 м 3 = 7,2 кг,
а его вес равен:
P Ге = g·m Ге; P Ге = 9,8 Н/кг · 7,2 кг = 71 Н.
Выталкивающая же сила (архимедова), действующая на этот шар в воздухе, равна весу воздуха объемом 40 м 3 , т. е.
F А = g·ρ возд V; F А = 9,8 Н/кг · 1,3 кг/м 3 · 40 м 3 = 520 Н.

Значит, этот шар может поднять груз весом 520 Н — 71 Н = 449 Н. Это и есть его подъемная сила.

Шар такого же объема, но наполненный водородом, может поднять груз 479 Н. Значит, подъемная сила его больше, чем шара, наполненного гелием. Но все же чаще используют гелий, так как он не горит и поэтому безопаснее. Водород же горючий газ.

Гораздо проще осуществить подъем и спуск шара, наполненного горячим воздухом. Для этого под отверстием, находящимся в нижней части шара, располагается горелка. При помощи газовой горелки можно регулировать температуру воздуха внутри шара, а значит, его плотность и выталкивающую силу. Чтобы шар поднялся выше, достаточно сильнее нагреть воздух в нем, увеличив пламя горелки. При уменьшении пламени горелки температура воздуха в шаре уменьшается, и шар опускается вниз.

Можно подобрать такую температуру шара, при которой вес шара и кабины будет равен выталкивающей силе. Тогда шар повиснет в воздухе, и с него будет легко проводить наблюдения.

По мере развития науки происходили и существенные изменения в воздухоплавательной технике. Появилась возможность использования новых оболочек для аэростатов, которые стали прочными, морозоустойчивыми и легкими.

Достижения в области радиотехники, электроники, автоматики позволили сконструировать беспилотные аэростаты. Эти аэростаты используются для изучения воздушных течений, для географических и медико-биологических исследований в нижних слоях атмосферы.

Жидкости и газы передают по всем направлениям не только оказываемое на них внешнее давление, но и то давление, которое существует внутри их благодаря весу собственных частей. Верхние слои жидкости давят на средние, те — на нижние, а последние — на дно.

Давление, оказываемое покоящейся жидкостью, называется гидростатическим
.

Получим формулу для расчета гидростатического давления жидкости на произвольной глубине h
(в окрестности точки А
на рисунке 98). Сила давления, действующая в этом месте со стороны вышележащего узкого вертикального столба жидкости, может быть выражена двумя способами:
во-первых, как произведение давления в основании этого столба на площадь его сечения:

F = pS
;

во-вторых, как вес того же столба жидкости, т. е. произведение массы жидкости (которая может быть найдена по формуле m = ρV
, где объем V = Sh
) на ускорение свободного падения g
:

F = mg = ρShg
.

Приравняем оба выражения для силы давления:

pS = ρShg
.

Разделив обе части этого равенства на площадь S, найдем давление жидкости на глубине h
:

p = ρgh

. (37.1)

Мы получили формулу гидростатического давления
. Гидростатическое давление на любой глубине внутри жидкости не зависит от формы сосуда, в котором находится жидкость, и равно произведению плотности жидкости, ускорения свободного падения и глубины, на которой рассматривается давление.

Одно и то же количество воды, находясь в разных сосудах, может оказывать разное давление на дно. Поскольку это давление зависит от высоты столба жидкости, то в узких сосудах оно будет больше, чем в широких. Благодаря этому даже небольшим количеством воды можно создать очень большое давление. В 1648 г. это очень убедительно продемонстрировал Б. Паскаль. Он вставил в закрытую бочку, наполненную водой, узкую трубку и, поднявшись на балкон второго этажа дома, вылил в эту трубку кружку воды. Из-за малой толщины трубки вода в ней поднялась до большой высоты, и давление в бочке увеличилось настолько, что крепления бочки не выдержали, и она треснула (рис. 99).
Полученные нами результаты справедливы не только для жидкостей, но и для газов. Их слои также давят друг на друга, и потому в них тоже существует гидростатическое давление.

1. Какое давление называют гидростатическим? 2. От каких величин зависит это давление? 3. Выведите формулу гидростатического давления на произвольном глубине. 4. Каким образом с помощью небольшого количества воды можно создать большое давление? Расскажите об опыте Паскаля.
Экспериментальное задание.
Возьмите высокий сосуд и сделайте в его стенке три небольших отверстия на разной высоте. Закройте отверстия пластилином и наполните сосуд водой. Откройте отверстия и проследите за струями вытекающей воды (рис. 100). Почему вода вытекает из отверстий? Из чего следует, что давление воды увеличивается с глубиной?

Рассмотрим, как можно рассчитать давление жидкости на дно и стенки сосуда. Решим сначала задачу с числовыми данными.
Прямоугольный бак наполнен водой (рис. 96). Площадь дна бака 16 м2, высота его 5 м. Определим давление воды на дно бака.

Сила, с которой вода давит на дно сосуда, равна весу столба воды высотой 5 м и площадью основания 16 м2, иначе говоря, эта сила равна весу всей воды в баке.

Чтобы найти вес воды, надо знать ее массу. Массу воды можно вычислить по объему и плотности. Найдем объем воды в баке, умножив площадь дна бака на его высоту: V= 16 м2*5 м=80 м3.
Теперь определим массу воды, для этого умножим ее плотность p = 1000 кг/м3 на объем: m =
1000 кг/м3 * 80 м3 = 80 000 кг. Мы знаем, что для определения веса тела надо его массу умножить на 9,8 Н/кг, так как тело массой 1 кг весит 9,8 Н.

Следовательно, вес воды в баке равен P =
9,8 Н/кг * 80 000 кг ≈ 800 000 Н. С такой силой вода давит на дно бака.

Разделив вес воды на площадь дна бака, найдем давление p:

p = 800000 H/16 м2 = 50 000 Па = 50 кПа.

Давление жидкости на дно сосуда можно рассчитать, пользуясь формулой, что значительно проще. Чтобы вывести эту формулу, вер­немся к задаче, но только решим ее в общем виде.

Обозначим высоту столба жидкости в сосуде буквой h, а площадь дна сосуда S.

Объем столба жидкости V=
Sh.

Масса жидкости
т
= pV,или m = pSh.

Вес этой жидкости P =
gm,
или P =
gpSh.

Так как вес столба жидкости равен силе, с которой жидкость давит на дно сосуда, то, разделив вес P
на площадь S,
получим давление р:

p = P/S, или p = gpSh/S

p =
gph.

Мы получили формулу для расчета давления жидкости на дно со­суда. Из этой формулы видно, что давление жидкости на дно сосуда прямо пропорционально плотности и высоте столба жидкости.

По этой формуле можно вычислять и давление на стенки, сосуда, а также давление внутри жидкости, в том числе давление снизу вверх, так как давление на одной и той же глубине одинаково по всем направлениям.

При расчете давления по формуле:

p =
gph

надо плотность p выражать в килограммах на кубический метр (кг/м3), а высоту столба жидкости h
— в метрах (м), g
= 9,8 Н/кг, тогда давление будет выражено в, паскалях (Па).

Пример. Определить давление нефти на дно цистерны, если высота столба нефти 10 м, а плотность ее 800 кг/м3.

Вопросы.
1. От каких величин зависит давление жидкости на дно сосуда? 2. Как зависит давление жидкости на дно сосуда от высоты столба жидкости? 3. Как зависит давление жидкости на дно сосуда от плотности жидкости?
4. Какие величины надо знать, чтобы рассчитать давление жидкости на стенки сосуда? 5. По какой формуле рассчитывают давление жидкости на дно и стенки сосуда?

Упражнения.
1. Определите давление на глубине 0,6 м в воде, керосине, ртути. 2. Вычислите давление воды на дно одной из глубочайших морских впадин, глубина, которой 10 900 м, Плотность морской воды 1030 кг/м3. 3. На рисунке 97 изображена футбольная камера, соединенная с вертикально расположенной стеклянной трубкой. В камере и трубке находится вода.
На камеру положена дощечка, а на нее — гиря массой 5 кг. Высота столба воды в трубке 1 м. Определите площадь соприкосновения дощечки с камерой.

Задания.
1. Возьмите высокий сосуд. В боковой поверхности его по прямой, на разной высоте от дна сделайте три небольших отверстия. Закройте отверстия спичками и налейте в сосуд до верха воды. Откройте отверстия и проследите за струйками вытекающей воды (рис. 98). Ответьте на вопросы: почему вода вытекает из отверстий? Из чего следует, что давление увеличивается с глубиной?
2. Прочтите в конце учебника параграфы «Гидростатический парадокс. Опыт Паскаля», «Давление на дне морей и океанов. Исследование морских глубин».

Давление — это физическая величина, которая играет особую роль в природе и жизни человека. Это незаметное глазу явление не только влияет на состояние окружающей среды, но и очень хорошо ощущается всеми. Давайте разберемся, что это такое, какие виды его существуют и как находить давление (формула) в разных средах.

Что называется давлением в физике и химии

Данным термином именуется важная термодинамическая величина, которая выражается в соотношении перпендикулярно оказываемой силы давления на площадь поверхности, на которую она воздействует. Это явление не зависит от размера системы, в которой действует, поэтому относится к интенсивным величинам.

В состоянии равновесия, по давление одинаково для всех точек системы.

В физике и химии оное обозначается с помощью буквы «Р», что является сокращением от латинского названия термина — pressūra.

Если речь идет об осмотическом давлении жидкости (равновесие между давлением внутри и снаружи клетки), используется буква «П».

Единицы давления

Согласно стандартам Международной системы СИ, рассматриваемое физическое явление измеряется в паскалях (кириллицей — Па, латиницей — Ра).

Исходя из формулы давления получается, что один Па равен одному Н (ньютон — разделенному на один квадратный метр (единица измерения площади).

Однако на практике применять паскали довольно сложно, поскольку эта единица очень мала. В связи с этим, помимо стандартов системы СИ, данная величина может измеряться по-другому.

Ниже приведены наиболее известные ее аналоги. Большинство из них широко используется на просторах бывшего СССР.

  • Бары
    . Один бар равен 105 Па.
  • Торры, или миллиметры ртутного столба.
    Приблизительно один торр соответствует 133, 3223684 Па.
  • Миллиметры водяного столба.
  • Метры водяного столба.
  • Технические атмосферы.
  • Физические атмосферы.
    Одна атм равна 101 325 Па и 1,033233 ат.
  • Килограмм-силы на квадратный сантиметр.
    Также выделяются тонна-сила и грамм-сила. Помимо этого, есть аналог фунт-сила на квадратный дюйм.

Общая формула давления (физика 7-го класса)

Из определения данной физической величины можно определить способ ее нахождения. Выглядит он таким образом, как на фото ниже.

В нем F — это сила, а S — площадь. Иными словами, формула нахождения давления — это его сила, разделенная на площадь поверхности, на которую оно воздействует.

Также она может быть записана так: Р = mg / S или Р = pVg / S. Таким образом, эта физическая величина оказывается связанной с другими термодинамическими переменными: объемом и массой.

Для давления действует следующий принцип: чем меньше пространство, на которое влияет сила — тем большее количество давящей силы на него приходится. Если, же площадь увеличивается (при той же силе) — искомая величина уменьшается.

Формула гидростатического давления

Разные агрегатные состояния веществ, предусматривают наличие у них отличных друг от друга свойств. Исходя из этого, способы определения Р в них тоже будут другими.

К примеру, формула давления воды (гидростатического) выглядит вот так: Р = pgh. Также она применима и к газам. При этом ее нельзя использовать для вычисления атмосферного давления, из-за разности высот и плотностей воздуха.

В данной формуле р — плотность, g — ускорение свободного падения, а h — высота. Исходя из этого, чем глубже погружается предмет или объект, тем выше оказываемое на него давление внутри жидкости (газа).

Рассматриваемый вариант является адаптацией классической примера Р = F / S.

Если вспомнить, что сила равна производной массы на скорость свободного падения (F= mg), а масса жидкости — это производная объема на плотность (m = pV), то формулу давление можно записать как P = pVg / S. При этом объем — это площад, умноженная на высоту (V = Sh).

Если вставить эти данные, получится, что площадь в числителе и знаменателе можно сократить и на выходе — вышеупомянутая формула: Р = pgh.

Рассматривая давление в жидкостях, стоит помнить, что, в отличие от твердых тел, в них часто возможно искривление поверхностного слоя. А это, в свою очередь, способствует образованию дополнительного давления.

Для подобных ситуаций применяется несколько другая формула давления: Р = Р 0 + 2QH. В данном случае Р 0 — давление не искривленного слоя, а Q — поверхность натяжения жидкости. Н — это средняя кривизна поверхности, которую определяют по Закону Лапласа: Н = ½ (1/R 1 + 1/R 2). Составляющие R 1 и R 2 — это радиусы главной кривизны.

Парциальное давление и его формула

Хотя способ Р = pgh применим как для жидкостей, так и для газов, давление в последних лучше вычислять несколько другим путем.

Дело в том, что в природе, как правило, не очень часто встречаются абсолютно чистые вещества, ведь в ней преобладают смеси. И это касается не только жидкостей, но и газов. А как известно, каждый из таких компонентов осуществляет разное давление, называемое парциальным.

Определить его довольно просто. Оно равно сумме давления каждого компонента рассматриваемой смеси (идеальный газ).

Из этого следует, что формула парциального давления выглядит таким образом: Р = Р 1 + Р 2 + Р 3 … и так далее, согласно количеству составляющих компонентов.

Нередки случаи, когда необходимо определить давление воздуха. Однако некоторые по ошибке проводят вычисления только с кислородом по схеме Р = pgh. Вот только воздух — это смесь из разных газов. В нем встречаются азот, аргон, кислород и другие вещества. Исходя из сложившейся ситуации, формула давления воздуха — это сумма давлений всех его составляющих. А значит, следует приметь вышеупомянутую Р = Р 1 + Р 2 + Р 3 …

Наиболее распространенные приборы для измерения давления

Несмотря на то что высчитать рассматриваемую термодинамическую величину по вышеупомянутым формулам не сложно, проводить вычисление иногда попросту нет времени. Ведь нужно всегда учитывать многочисленные нюансы. Поэтому для удобства за несколько столетий был разработан ряд приборов, делающих это вместо людей.

Фактически почти все аппараты такого рода являются разновидностями манометра (помогает определять давление в газах и жидкостях). При этом они отличаются по конструкции, точности и сфере применения.

  • Атмосферное давление измеряется с помощью манометра, именуемого барометром. Если необходимо определить разряжение (то есть давление ниже атмосферного) — применяются другая его разновидность, вакуумметр.
  • Для того чтобы узнать артериальное давление у человека, в ход идет сфигмоманометр. Большинству он более известен под именем неинвазивного тонометра. Таких аппаратов существуют немало разновидностей: от ртутных механических до полностью автоматических цифровых. Их точность зависит от материалов, из которых они изготавливаются и места измерения.
  • Перепады давления в окружающей среде (по-английски — pressure drop) определяются с помощью или дифнамометров (не путать с динамометрами).

Виды давления

Рассматривая давление, формулу его нахождения и ее вариации для разных веществ, стоит узнать о разновидностях этой величины. Их пять.

  • Абсолютное.
  • Барометрическое
  • Избыточное.
  • Вакуумметрическое.
  • Дифференциальное.

Абсолютное

Так называется полное давление, под которым находится вещество или объект, без учета влияния других газообразных составляющих атмосферы.

Измеряется оно в паскалях и являет собою сумму избыточного и атмосферного давлений. Также он является разностью барометрического и вакуумметрического видов.

Вычисляется оно по формуле Р = Р 2 + Р 3 или Р = Р 2 — Р 4 .

За начало отсчета для абсолютного давления в условиях планеты Земля, берется давление внутри емкости, из которой удален воздух (то есть классический вакуум).

Только такой вид давления используется в большинстве термодинамических формул.

Барометрическое

Этим термином именуется давление атмосферы (гравитации) на все предметы и объекты, находящие в ней, включая непосредственно поверхность Земли. Большинству оно также известно под именем атмосферного.

Его причисляют к а его величина меняется относительно места и времени измерения, а также погодных условий и нахождения над/ниже уровня моря.

Величина барометрического давления равна модулю силы атмосферы на площади единицу по нормали к ней.

В стабильной атмосфере величина данного физического явления равна весу столпа воздуха на основание с площадью, равной единице.

Норма барометрического давления — 101 325 Па (760 мм рт. ст. при 0 градусов Цельсия). При этом чем выше объект оказывается от поверхности Земли, тем более низким становится давление на него воздуха. Через каждый 8 км оно снижается на 100 Па.

Благодаря этому свойству в горах вода в чайниках закивает намного быстрее, чем дома на плите. Дело в том, что давление влияет на температуру кипения: с его снижением последняя уменьшается. И наоборот. На этом свойстве построена работа таких кухонных приборов, как скороварка и автоклав. Повышение давления внутри их способствуют формированию в посудинах более высоких температур, нежели в обычных кастрюлях на плите.

Используется для вычисления атмосферного давления формула барометрической высоты. Выглядит она таким образом, как на фото ниже.

Р — это искомая величина на высоте, Р 0 — плотность воздуха возле поверхности, g — свободного падения ускорение, h — высота над Землей, м — молярная масса газа, т — температура системы, r — универсальная газовая постоянная 8,3144598 Дж⁄(моль х К), а е — это число Эйклера, равное 2.71828.

Часто в представленной выше формуле давления атмосферного вместо R используется К — постоянная Больцмана. Через ее произведение на число Авогадро нередко выражается универсальная газовая постоянная. Она более удобна для расчетов, когда число частиц задано в молях.

При проведении вычислений всегда стоит брать во внимание возможность изменения температуры воздуха из-за смены метеорологической ситуации или при наборе высоты над уровнем моря, а также географическую широту.

Избыточное и вакуумметрическое

Разницу между атмосферным и измеренным давлением окружающей среды называют избыточным давлением. В зависимости от результата, меняется название величины.

Если она положительная, ее называют манометрическим давлением.

Если же полученный результат со знаком минус — его именуют вакуумметрическим. Стоит помнить, что он не может быть больше барометрического.

Дифференциальное

Данная величина является разницей давлений в различных точках измерения. Как правило, ее используют для определения падения давления на каком-либо оборудовании. Особенно это актуально в нефтедобывающей промышленности.

Разобравшись с тем, что за термодинамическая величина называется давлением и с помощью каких формул ее находят, можно сделать вывод, что это явление весьма важно, а потому знания о нем никогда не будут лишними.

Поделиться с друзьями:

Формула давления твердых тел. Формула давления в физике

Давление – очень важная физическая величина, играющая огромную роль, как в окружающей природе, так и жизни человека. Внешне незаметное человеческому глазу давление может очень хорошо ощущаться каждым из нас. Особенно хорошо это усвоили люди в возрасте, часто страдающие от повышенного давления (или наоборот от пониженного). Но в нашей статье мы больше поговорим именно о давлении в физике, о том, как оно измеряется и рассчитывается, какие есть формулы для расчетов давления разных субстанций: воздуха, жидкости или твердого тела.

Определение давления в физике

Под давлением в физике понимается термодинамическая величина, выраженная соотношением перпендикулярной силы давления на площадь поверхности, на которую она воздействует. При этом согласно закону Паскаля если система находится в состоянии равновесия, то давление на нее будет одинаковым для всех точек системы.

В физике, как впрочем и химии, давление обозначают большой буквой Р, идущей от латинского слова «pressura» – давление. (В английском языке давление так и осталось почти без изменения – pressure).

Общая формула давления

Из классического определения того, что такое давление можно вывести общую формулу для его расчета. Выглядеть она будет таким образом:

Где F – это сила давления, а S – площадь поверхности на которую она действует. То есть иными словами формула нахождения давления – это сила, воздействующая на определенную поверхность, разделенная на площадь этой самой поверхности.

Как видно из формулы, при расчете давления всегда действует следующий принцип: чем меньше пространство, на которое влияет сила, тем большее количество давящей силы на него приходится и наоборот.

Это можно проиллюстрировать простым жизненным примером: хлеб легче всего порезать острым ножом, потому что у острого ножа заточенное лезвие, то есть площадь поверхности S из формулы у него минимальна, а значит, давление ножа на хлеб будет максимально равно приложенной силе F того кто держит нож. А вот тупым ножом порезать хлеб уже сложнее, так как у его лезвия большая площадь поверхности S, и давление ножа на хлеб будет меньшим, и значит, чтобы отрезать себе кусок хлеба нужно приложить большее количество силы F.

Общая формула давления, по сути, отлично описывает формулу давления твердого тела.

Единицы давления

Согласно стандартам Международной метрической системы давление измеряется в паскалях. Один паскаль из классической формулы равен одному Ньютону (Как мы знаем, Ньютон у нас единица измерения силы) разделенному на один квадратный метр.

Но увы на практике паскаль оказывается очень маленькой единицей и использовать его для измерения давления не всегда удобно, поэтому часто для измерения давления применяют другие единицы:

  • Бары – один бар равен 105 паскалей
  • Миллиметры водяного столпа
  • Метры водяного столпа
  • Технические и физические атмосферы

Формула гидростатического давления

Как мы знаем, разные агрегатные состояния вещества, имеют разные физические свойства. Жидкости своими свойствами отличаются от твердых тел, а газы в свою очередь отличаются от них всех. Поэтому вполне логично, что способы определения давления для жидкостей, твердых тел и газов также будут разными. Так, например, формула давления воды (или гидростатического давления) будет иметь следующий вид:

Где маленькая p – плотность вещества, g – ускорение свободного падения, h – высота.

В частности эта формула объясняет, почему при погружении водолазов (или батискафа или подводной лодки) на глубину все больше возрастает давление окружающей воды. Также из этой формулы понятно, почему на предмет, погруженный в какой-нибудь кисель, будет воздействовать большее давление, чем на предмет, погруженный просто в воду, так как плотность киселя (p) выше, чем у воды, а чем выше плотность жидкости, тем выше ее гидростатическое давление.

Приведенная нами формула гидростатического давления справедлива не только для жидкостей, но и для газов. Поэтому поднимаясь высоко в горы (где воздух более разрежен, а значит меньшее давление), как и спускаясь в подводные глубины, человек, водолаз или альпинист должен пройти специальную адаптацию, привыкнуть к тому, что на него будет воздействовать другое давление.

Резкая смена давления может привести к кессоной болезни (в случае с водолазами) или к «горной» болезни (в случае с альпинистами). И «кесонка» и «горняшка», как их сленгово называют водолазы и альпинисты, вызвана резкой сменной давления окружающей среды. То есть, если не подготовленный человек начнет вдруг подниматься на Эверест, то он быстро словит «горняшку», а если этот же человек начнет опускаться на дно Мариинской впадины, то гарантировано получит «кесонку». В первом случае причиной будет не адаптация организма к пониженному давлению, а во втором – к повышенному.

Американские водолазы в декомпрессионой камере, призванной подготовить их к глубоководным погружениям и адаптировать организм к высокому давлению океанских глубин.

Парциальное давление и его формула

Хотя формула гидростатического давления применима для газов, но давления для них удобнее вычислять по другой формуле, формуле парциального давления.

Дело в том, что в природе редко встречаются абсолютно чистые вещества, причем это касается как жидкостей, так и газов. Обычно на практике в окружающем мире преобладают различные смеси, и логично, что каждый из компонентов такой смеси может оказывать разное давление, такое разное давление и называют парциальным. Определить парциальное давление просто – оно равно суме давлений каждого компонента рассматриваемой смеси. Отсюда формула парциального давления будет иметь следующий вид:

P = P 1 +P 2 +P 3

Где P 1 , P 2 и P 3 – давления каждого из компонентов газовой смеси, так званный «идеальный газ».

К примеру, чтобы определить давления воздуха обычной формулы гидростатического давления проделанной только с недостаточно, так как воздух в реальности представляет собой смесь разных газов, где помимо основного компонента кислорода, которым мы все дышим, есть и другие: азот, аргон и т. д.

Такие расчеты нужно проделывать при помощи формулы парциального давления.

Формула давления идеального газа

Также стоит заметить, что давление идеального газа, то есть каждого отдельного из компонентов газовой смеси удобно посчитать по формуле молекулярно-кинетической теории.

Где n – концентрация молекул газа, T – абсолютная температура газа, k – постоянная Больцмана (указывает на взаимосвязь между кинетической энергией частицы газа и ее абсолютной температурой), она равна 1,38*10 -23 Дж/К.

Приборы для измерения давления

Разумеется, человечество изобрело многие приборы, позволяющие быстро и удобно измерять уровень давления. Для измерения давления окружающей среды, оно же атмосферное давление используют такой прибор как манометр или барометр.

Чтобы узнать артериальное давление у человека, часто служащее причиной недомоганий используется прибор известный большинству под названием неинвазивный тонометр. Таких приборов существует множество разновидностей.

Также биологи в своих исследованиях занимаются расчетами осмотического давления – это давление внутри и снаружи клетки. А метеорологи, в частности по перепадам давления в окружающей среде предсказывают нам погоду.

  • Кузнецов В. Н. Давление. Большая Российская Энциклопедия. Дата обращения 27 августа 2016.
  • E.R. Cohen et al, «Quantities, Units and Symbols in Physical Chemistry», IUPAC Green Book, 3rd Edition, 2nd Printing, IUPAC & RSC Publishing, Cambridge (2008). — p. 14.

Расчет давления жидкости на дно и стенки сосуда, видео

Человек на лыжах, и без них.

По рыхлому снегу человек идёт с большим трудом, глубоко проваливаясь при каждом шаге. Но, надев лыжи, он может идти, почти не проваливаясь в него. Почему? На лыжах или без лыж человек действует на снег с одной и той же силой, равной своему весу. Однако действие этой силы в обоих случаях различно, потому что различна площадь поверхности, на которую давит человек, с лыжами и без лыж. Площадь поверхности лыж почти в 20 раз больше площади подошвы. Поэтому, стоя на лыжах, человек действует на каждый квадратный сантиметр площади поверхности снега с силой, в 20 раз меньшей, чем стоя на снегу без лыж.

Ученик, прикалывая кнопками газету к доске, действует на каждую кнопку с одинаковой силой. Однако кнопка, имеющая более острый конец, легче входит в дерево.

Значит, результат действия силы зависит не только от её модуля, направления и точки приложения, но и от площади той поверхности, к которой она приложена (перпендикулярно которой она действует).

Этот вывод подтверждают физические опыты.

Опыт.Результат действия данной силы зависит от того, какая сила действует на единицу площади поверхности.

По углам небольшой доски надо вбить гвозди. Сначала гвозди, вбитые в доску, установим на песке остриями вверх и положим на доску гирю. В этом случае шляпки гвоздей лишь незначительно вдавливаются в песок. Затем доску перевернем и поставим гвозди на острие. В этом случае площадь опоры меньше, и под действием той же силы гвозди значительно углубляются в песок.

Опыт. Вторая иллюстрация.

От того, какая сила действует на каждую единицу площади поверхности, зависит результат действия этой силы.

В рассмотренных примерах силы действовали перпендикулярно поверхности тела. Вес человека был перпендикулярен поверхности снега; сила, действовавшая на кнопку, перпендикулярна поверхности доски.

Величина, равная отношению силы, действующей перпендикулярно поверхности, к площади этой поверхности, называется давлением
.

Чтобы определить давление, надо силу, действующую перпендикулярно поверхности, разделить на площадь поверхности:

давление = сила / площадь
.

Обозначим величины, входящие в это выражение: давление — p
, сила, действующая на поверхность, — F
и площадь поверхности — S
.

Тогда получим формулу:

p = F/S

Понятно, что бóльшая по значению сила, действующую на ту же площадь, будет производить большее давление.

За единицу давления принимается такое давление, которое производит сила в 1 Н, действующая на поверхность площадью 1 м 2 перпендикулярно этой поверхности
.

Единица давления — ньютон на квадратный метр
(1 Н / м 2). В честь французского ученого Блеза Паскаля

она называется паскалем (Па
). Таким образом,

1 Па = 1 Н / м 2
.

Используется также другие единицы давления: гектопаскаль
(гПа
) и килопаскаль
(кПа
).

1 кПа = 1000 Па;

1 гПа = 100 Па;

1 Па = 0,001 кПа;

1 Па = 0,01 гПа.

Запишем условие задачи и решим её.

Дано

: m = 45 кг, S = 300 см 2 ; p = ?

В единицах СИ: S = 0,03 м 2

Решение:

p
= F
/S
,

F
= P
,

P
= g·m
,

P
= 9,8 Н · 45 кг ≈ 450 Н,

p
= 450/0,03 Н / м 2 = 15000 Па = 15 кПа

«Ответ»: p = 15000 Па = 15 кПа

Способы уменьшения и увеличения давления.

Тяжелый гусеничный трактор производит на почву давление равное 40 — 50 кПа, т. е. всего в 2 — 3 раза больше, чем давление мальчика массой 45 кг. Это объясняется тем, что вес трактора распределяется на бóльшую площадь за счёт гусеничной передачи. А мы установили, что чем больше площадь опоры, тем меньше давление, производимое одной и той же силой на эту опору

.

В зависимости от того, нужно ли получить малое или большое давление, площадь опоры увеличивается или уменьшается. Например, для того, чтобы грунт мог выдержать давление возводимого здания, увеличивают площадь нижней части фундамента.

Шины грузовых автомобилей и шасси самолетов делают значительно шире, чем легковых. Особенно широкими делают шины у автомобилей, предназначенных для передвижения в пустынях.

Тяжелые машины, как трактор, танк или болотоход, имея большую опорную площадь гусениц, проходят по болотистой местности, по которой не пройдет человек.

С другой стороны, при малой площади поверхности можно небольшой силой произвести большое давление. Например, вдавливая кнопку в доску, мы действуем на нее с силой около 50 Н. Так как площадь острия кнопки примерно 1 мм 2 , то давление, производимое ею, равно:

p = 50 Н/ 0, 000 001 м 2 = 50 000 000 Па = 50 000 кПа.

Для сравнения, это давление в 1000 раз больше давления, производимого гусеничным трактором на почву. Можно найти еще много таких примеров.

Лезвие режущих и острие колющих инструментов (ножей, ножниц, резцов, пил, игл и др.) специально остро оттачивается. Заточенный край острого лезвия имеет маленькую площадь, поэтому при помощи даже малой силы создается большое давление, и таким инструментом легко работать.

Режущие и колющие приспособления встречаются и в живой природе: это зубы, когти, клювы, шипы и др. — все они из твердого материала, гладкие и очень острые.

Давление

Известно, что молекулы газа беспорядочно движутся.

Мы уже знаем, что газы, в отличие от твердых тел и жидкостей, заполняют весь сосуд, в котором находятся. Например, стальной баллон для хранения газов, камера автомобильной шины или волейбольный мяч. При этом газ оказывает давление на стенки, дно и крышку баллона, камеры или любого другого тела, в котором он находится. Давление газа обусловлено иными причинами, чем давление твердого тела на опору.

Известно, что молекулы газа беспорядочно движутся. При своем движении они сталкиваются друг с другом, а также со стенками сосуда, в котором находится газ. Молекул в газе много, поэтому и число их ударов очень велико. Например, число ударов молекул воздуха, находящегося в комнате, о поверхность площадью 1 см 2 за 1 с выражается двадцатитрехзначным числом. Хотя сила удара отдельной молекулы мала, но действие всех молекул на стенки сосуда значительно, — оно и создает давление газа.

Итак, давление газа на стенки сосуда (и на помещенное в газ тело) вызывается ударами молекул газа

.

Рассмотрим следующий опыт. Под колокол воздушного насоса поместим резиновый шарик. Он содержит небольшое количество воздуха и имеет неправильную форму. Затем насосом откачиваем воздух из-под колокола. Оболочка шарика, вокруг которой воздух становится все более разреженным, постепенно раздувается и принимает форму правильного шара.

Как объяснить этот опыт?

Для хранения и перевозки сжатого газа используются специальные прочные стальные баллоны.

В нашем опыте движущиеся молекулы газа непрерывно ударяют о стенки шарика внутри и снаружи. При откачивании воздуха число молекул в колоколе вокруг оболочки шарика уменьшается. Но внутри шарика их число не изменяется. Поэтому число ударов молекул о внешние стенки оболочки становится меньше, чем число ударов о внутренние стенки. Шарик раздувается до тех пор, пока сила упругости его резиновой оболочки не станет равной силе давления газа. Оболочка шарика принимает форму шара. Это показывает, что газ давит на ее стенки по всем направлениям одинаково
. Иначе говоря, число ударов молекул, приходящихся на каждый квадратный сантиметр площади поверхности, по всем направлениям одинаково. Одинаковое давление по всем направлениям характерно для газа и является следствием беспорядочного движения огромного числа молекул.

Попытаемся уменьшить объем газа, но так, чтобы масса его осталась неизменной. Это значит, что в каждом кубическом сантиметре газа молекул станет больше, плотность газа увеличится. Тогда число ударов молекул о стенки увеличится, т. е. возрастет давление газа. Это можно подтвердить опытом.

На рисунке а
изображена стеклянная трубка, один конец которой закрыт тонкой резиновой пленкой. В трубку вставлен поршень. При вдвигании поршня объем воздуха в трубке уменьшается, т. е. газ сжимается. Резиновая пленка при этом выгибается наружу, указывая на то, что давление воздуха в трубке увеличилось.

Наоборот, при увеличении объема этой же массы газа, число молекул в каждом кубическом сантиметре уменьшается. От этого уменьшится число ударов о стенки сосуда — давление газа станет меньше. Действительно, при вытягивании поршня из трубки объем воздуха увеличивается, пленка прогибается внутрь сосуда. Это указывает на уменьшение давления воздуха в трубке. Такие же явления наблюдались бы, если бы вместо воздуха в трубке находился бы любой другой газ.

Итак, при уменьшении объема газа его давление увеличивается, а при увеличении объема давление уменьшается при условии, что масса и температура газа остаются неизменными
.

А как изменится давление газа, если нагреть его при постоянном объеме? Известно, что скорость движения молекул газа при нагревании увеличивается. Двигаясь быстрее, молекулы будут ударять о стенки сосуда чаще. Кроме того, каждый удар молекулы о стенку будет сильнее. Вследствие этого, стенки сосуда будут испытывать большее давление.

Следовательно, давление газа в закрытом сосуде тем больше, чем выше температура газа
, при условии, что масса газа и объем не изменяются.

Из этих опытов можно сделать общий вывод, что давление газа тем больше, чем чаще и сильнее молекулы ударяют о стенки сосуда

.

Для хранения и перевозки газов их сильно сжимают. При этом давление их возрастает, газы необходимо заключать в специальные, очень прочные баллоны. В таких баллонах, например, содержат сжатый воздух в подводных лодках, кислород, используемый при сварке металлов. Конечно же, мы должны навсегда запомнить, что газовые баллоны нельзя нагревать, тем более, когда они заполнены газом. Потому что, как мы уже понимаем, может произойти взрыв с очень неприятными последствиями.

Закон Паскаля.

Давление передается в каждую точку жидкости или газа.

Давление поршня передается в каждую точку жидкости, заполняющей шар.

Теперь газ.

В отличие от твердых тел отдельные слои и мелкие частицы жидкости и газа могут свободно перемещаться относительно друг друга по всем направлениям. Достаточно, например, слегка подуть на поверхность воды в стакане, чтобы вызвать движение воды. На реке или озере при малейшем ветерке появляется рябь.

Подвижностью частиц газа и жидкости объясняется, что давление, производимое на них, передается не только в направлении действия силы, а в каждую точку
. Рассмотрим это явление подробнее.

На рисунке, а
изображен сосуд, в котором содержится газ (или жидкость). Частицы равномерно распределены по всему сосуду. Сосуд закрыт поршнем, который может перемещаться вверх и вниз.

Прилагая некоторую силу, заставим поршень немного переместиться внутрь и сжать газ (жидкость), находящийся непосредственно под ним. Тогда частицы (молекулы) расположатся в этом месте более плотно, чем прежде(рис, б). Благодаря подвижности частицы газа будут перемещаться по всем направлениям. Вследствие этого их расположение опять станет равномерным, но более плотным, чем раньше (рис, в). Поэтому давление газа всюду возрастет. Значит, добавочное давление передается всем частицам газа или жидкости. Так, если давление на газ (жидкость) около самого поршня увеличится на 1 Па, то во всех точках внутри
газа или жидкости давление станет больше прежнего на столько же. На 1 Па увеличится давление и на стенки сосуда, и на дно, и на поршень.

Давление, производимое на жидкость или газ, передается на любую точку одинаково во всех направлениях

.

Это утверждение называется законом Паскаля
.

На основе закона Паскаля легко объяснить следующие опыты.

На рисунке изображен полый шар, имеющий в различных местах небольшие отверстия. К шару присоединена трубка, в которую вставлен поршень. Если набрать воды в шар и вдвинуть в трубку поршень, то вода польется из всех отверстий шара. В этом опыте поршень давит на поверхность воды в трубке. Частицы воды, находящиеся под поршнем, уплотняясь, передают его давление другим слоям, лежащим глубже. Таким образом, давление поршня передается в каждую точку жидкости, заполняющей шар. В результате часть воды выталкивается из шара в виде одинаковых струек, вытекающих из всех отверстий.

Если шар заполнить дымом, то при вдвигании поршня в трубку из всех отверстий шара начнут выходить одинаковые струйки дыма. Это подтверждает, что и газы передают производимое на них давление во все стороны одинаково
.

Давление в жидкости и газе.

Под действием веса жидкости резиновое дно в трубке прогнется.

На жидкости, как и на все тела на Земле, действует сила тяжести. Поэтому, каждый слой жидкости, налитой в сосуд, своим весом создает давление, которое по закону Паскаля передается по всем направлениям. Следовательно, внутри жидкости существует давление. В этом можно убедиться на опыте.

В стеклянную трубку, нижнее отверстие которой закрыто тонкой резиновой пленкой, нальем воду. Под действием веса жидкости дно трубки прогнется.

Опыт показывает, что, чем выше столб воды над резиновой пленкой, тем больше она прогибается. Но всякий раз после того, как резиновое дно прогнулось, вода в трубке приходит в равновесие (останавливается), так как, кроме силы тяжести, на воду действует сила упругости растянутой резиновой пленки.

Силы, действующие на резиновую пленку,

одинаковы с обеих сторон.

Иллюстрация.

Дно отходит от цилиндра вследствие давления на него силы тяжести.

Опустим трубку с резиновым дном, в которую налита вода, в другой, более широкий сосуд с водой. Мы увидим, что по мере опускания трубки резиновая пленка постепенно выпрямляется. Полное выпрямление пленки показывает, что силы, действующие на нее сверху и снизу, равны. Наступает полное выпрямление пленки тогда, когда уровни воды в трубке и сосуде совпадают.

Такой же опыт можно провести с трубкой, в которой резиновая пленка закрывает боковое отверстие, как это показано на рисунке, а. Погрузим эту трубку с водой в другой сосуд с водой, как это изображено на рисунке, б
. Мы заметим, что пленка снова выпрямится, как только уровни воды в трубке и сосуде сравняются. Это означает, что силы, действующие на резиновую пленку, одинаковы со всех сторон.

Возьмем сосуд, дно которого может отпадать. Опустим его в банку с водой. Дно при этом окажется плотно прижатым к краю сосуда и не отпадет. Его прижимает сила давления воды, направленная снизу вверх.

Будем осторожно наливать воду в сосуд и следить за его дном. Как только уровень воды в сосуде совпадет с уровнем воды в банке, оно отпадет от сосуда.

В момент отрыва на дно давит сверху вниз столб жидкости в сосуде, а снизу вверх на дно передается давление такого же по высоте столба жидкости, но находящейся в банке. Оба эти давления одинаковы, дно же отходит от цилиндра вследствие действия на него собственной силы тяжести.

Выше были описаны опыты с водой, но если взять вместо воды любую другую жидкость, результаты опыта будут те же.

Итак, опыты показывают, что внутри жидкости существует давление, и на одном и том же уровне оно одинаково по всем направлениям. С глубиной давление увеличивается
.

Газы в этом отношении не отличаются от жидкостей, ведь они тоже имеют вес. Но надо помнить, что плотность газа в сотни раз меньше плотности жидкости. Вес газа, находящегося в сосуде, мал, и его «весовое» давление во многих случаях можно не учитывать.

Расчет давления жидкости на дно и стенки сосуда.

Расчет давления жидкости на дно и стенки сосуда.

Рассмотрим, как можно рассчитывать давление жидкости на дно и стенки сосуда. Решим сначала задачу для сосуда, имеющего форму прямоугольного параллелепипеда.

Сила F
, с которой жидкость, налитая в этот сосуд, давит на его дно, равна весу P
жидкости, находящейся в сосуде. Вес жидкости можно определить, зная ее массу m
. Массу, как известно, можно вычислить по формуле: m = ρ·V
. Объем жидкости, налитой в выбранный нами сосуд, легко рассчитать. Если высоту столба жидкости, находящейся в сосуде, обозначить буквой h
, а площадь дна сосуда S
, то V = S·h
.

Масса жидкости m = ρ·V
, или m = ρ·S·h
.

Вес этой жидкости P = g·m
, или P = g·ρ·S·h
.

Так как вес столба жидкости равен силе, с которой жидкость давит на дно сосуда, то, разделив вес P
на площадь S
, получим давление жидкости p
:

p = P/S , или p = g·ρ·S·h/S,

Мы получили формулу для расчета давления жидкости на дно сосуда. Из этой формулы видно, что давление жидкости на дно сосуда зависит только от плотности и высоты столба жидкости
.

Следовательно, по выведенной формуле можно рассчитывать давление жидкости, налитой в сосуд любой формы
(строго говоря, наш расчет годится только для сосудов, имеющих форму прямой призмы и цилиндра. В курсах физики для института доказано, что формула верна и для сосуда произвольной формы). Кроме того, по ней можно вычислить и давление на стенки сосуда. Давление внутри жидкости, в том числе давление снизу вверх, также рассчитывается по этой формуле, так как давление на одной и той же глубине одинаково по всем направлениям.

При расчете давления по формуле p = gρh
надо плотность ρ
выражать в килограммах на кубический метр (кг/м 3), а высоту столба жидкости h
— в метрах (м), g
= 9,8 Н/кг, тогда давление будет выражено в паскалях (Па).

Пример
. Определите давление нефти на дно цистерны, если высота столба нефти 10 м, а плотность ее 800 кг/м 3 .

Запишем условие задачи и запишем ее.

Дано

:

ρ = 800 кг/м 3

Решение

:

p = 9.8 Н/кг · 800 кг/м 3 · 10 м ≈ 80 000 Па ≈ 80 кПа.

Ответ

: p ≈ 80 кПа.

Сообщающиеся сосуды.

Сообщающиеся сосуды.

На рисунке изображены два сосуда, соединённые между собой резиновой трубкой. Такие сосуды называются сообщающимися
. Лейка, чайник, кофейник — примеры сообщающихся сосудов. Из опыта мы знаем, что вода, налитая, например, в лейку, стоит всегда на одном уровне в носике и внутри.

Сообщающиеся сосуды встречаются нам часто. Например, им может быть чайник, лейка или кофейник.

Поверхности однородной жидкости устанавливаются на одном уровне в сообщающихся сосудах любой формы.

Разные по плотности жидкости.

С сообщающимися сосудами можно проделать следующий простой опыт. В начале опыта резиновую трубку зажимаем в середине, и в одну из трубок наливаем воду. Затем зажим открываем, и вода вмиг перетекает в другую трубку, пока поверхности воды в обеих трубках не установятся на одном уровне. Можно закрепить одну из трубок в штативе, а другую поднимать, опускать или наклонять в разные стороны. И в этом случае, как только жидкость успокоится, ее уровни в обеих трубках уравняются.

В сообщающихся сосудах любой формы и сечения поверхности однородной жидкости устанавливаются на одном уровне
(при условии, что давление воздуха над жидкостью одинаково) (рис. 109).

Это можно обосновать следующим образом. Жидкость покоится, не перемещаясь из одного сосуда в другой. Значит, давления в обоих сосудах на любом уровне одинаковы. Жидкость в обоих сосудах одна и та же, т. е. имеет одинаковую плотность. Следовательно, должны быть одинаковы и ее высоты. Когда мы поднимаем один сосуд или доливаем в него жидкость, давление в нем увеличивается и жидкость перемещается в другой сосуд до тех пор, пока давления не уравновесятся.

Если в один из сообщающихся сосудов налить жидкость одной плотности, а во второй — другой плотности, то при равновесии уровни этих жидкостей не будут одинаковыми. И это понятно. Мы ведь знаем, что давление жидкости на дно сосуда прямо пропорционально высоте столба и плотности жидкости. А в этом случае плотности жидкостей будут различны.

При равенстве давлений высота столба жидкости с большей плотностью будет меньше высоты столба жидкости с меньшей плотностью (рис.).

Опыт. Как определить массу воздуха.

Вес воздуха. Атмосферное давление.

Существование атмосферного давления.

Атмосферное давление больше, чем давление разреженного воздуха в сосуде.

На воздух, как и на всякое тело, находящееся на Земле, действует сила тяжести, и, значит, воздух обладает весом. Вес воздуха легко вычислить, зная его массу.

На опыте покажем, как вычислить массу воздуха. Для этого нужно взять прочный стеклянный шар с пробкой и резиновой трубкой с зажимом. Выкачаем из него насосом воздух, зажмем трубку зажимом и уравновесим на весах. Затем, открыв зажим на резиновой трубке, впустим в него воздух. Равновесие весов при этом нарушится. Для его восстановления на другую чашку весов придется положить гири, масса которых будет равна массе воздуха в объеме шара.

Опытами установлено, что при температуре 0 °С и нормальном атмосферном давлении масса воздуха объемом 1 м 3 равна 1,29 кг. Вес этого воздуха легко вычислить:

P = g·m, P = 9,8 Н/кг · 1,29 кг ≈ 13 Н.

Воздушная оболочка, окружающая Землю, называется атмосфера

(от греч. атмос
— пар, воздух, и сфера
— шар).

Атмосфера, как показали наблюдения за полетом искусственных спутников Земли, простирается на высоту нескольких тысяч километров.

Вследствие действия силы тяжести верхние слои атмосферы, подобно воде океана, сжимают нижние слои. Воздушный слой, прилегающий непосредственно к Земле, сжат больше всего и, согласно закону Паскаля, передает производимое на него давление по всем направлениям.

В результате этого земная поверхность и телá, находящиеся на ней, испытывают давление всей толщи воздуха, или, как обычно говорится в таких случаях, испытывают атмосферное давление

.

Существованием атмосферного давления могут быть объяснены многие явления, с которыми мы встречаемся в жизни. Рассмотрим некоторые из них.

На рисунке изображена стеклянная трубка, внутри которой находится поршень, плотно прилегающий к стенкам трубки. Конец трубки опущен воду. Если поднимать поршень, то за ним будет подниматься и вода.

Это явление используется в водяных насосах и некоторых других устройствах.

На рисунке показан цилиндрический сосуд. Он закрыт пробкой, в которую вставлена трубка с краном. Из сосуда насосом откачивается воздух. Затем конец трубки помещается в воду. Если теперь открыть кран, то вода фонтаном брызнет в внутрь сосуда. Вода поступает в сосуд потому, что атмосферное давление больше давления разреженного воздуха в сосуде.

Почему существует воздушная оболочка Земли.

Как и все тела, молекулы газов, входящих в состав воздушной оболочки Земли, притягиваются к Земле.

Но почему же тогда все они не упадут на поверхность Земли? Каким образом сохраняется воздушная оболочка Земли, ее атмосфера? Чтобы понять это, надо учесть, что молекулы газов находятся в непрерывном и беспорядочном движении. Но тогда возникает другой вопрос: почему эти молекулы не улетают в мировое пространство, то есть в космос.

Для того, чтобы совсем покинуть Землю, молекула, как и космический корабль или ракета, должна иметь очень большую скорость (не меньше 11,2 км/с). Это так называемая вторая космическая скорость
. Скорость большинства молекул воздушной оболочки Земли значительно меньше этой космической скорости. Поэтому большинство их привязано к Земле силой тяжести, лишь ничтожно малое количество молекул улетает за пределы Земли в космос.

Беспорядочное движение молекул и действие на них силы тяжести приводят в результате к тому, что молекулы газов «парят» в пространстве около Земли, образуя воздушную оболочку, или известную нам атмосферу.

Измерения показывают, что плотность воздуха быстро уменьшается с высотой. Так, на высоте 5,5 км над Землей плотность воздуха в 2 раза меньше его плотность у поверхности Земли, на высоте 11 км — в 4 раза меньше, и т. д. Чем выше, тем воздух разреженнее. И наконец, в самых верхних слоях (сотни и тысячи километров над Землей) атмосфера постепенно переходит в безвоздушное пространство. Четкой границы воздушная оболочка Земли не имеет.

Строго говоря, вследствие действия силы тяжести плотность газа в любом закрытом сосуде неодинакова по всему объему сосуда. Внизу сосуда плотность газа больше, чем в верхних его частях, поэтому и давление в сосуде неодинаково. На дне сосуда оно больше, чем вверху.
Однако для газа, содержащегося в сосуде, это различие в плотности и давлении столь мало, что его можно во многих случаях совсем не учитывать, просто знать об этом. Но для атмосферы, простирающейся на несколько тысяч километров, различие это существенно.

Измерение атмосферного давления. Опыт Торричелли.

Рассчитать атмосферное давление по формуле для вычисления давления столба жидкости (§ 38) нельзя. Для такого расчета надо знать высоту атмосферы и плотность воздуха. Но определенной границы у атмосферы нет, а плотность воздуха на разной высоте различна. Однако измерить атмосферное давление можно с помощью опыта, предложенного в 17 веке итальянским ученым Эванджелиста Торричелли

, учеником Галилея.

Опыт Торричелли состоит в следующем: стеклянную трубку длиной около 1 м, запаянную с одного конца, наполняют ртутью. Затем, плотно закрыв второй конец трубки, ее переворачивают и опускают в чашку с ртутью, где под уровнем ртути открывают этот конец трубки. Как и в любом опыте с жидкостью, часть ртути при этом выливается в чашку, а часть ее остается в трубке. Высота столба ртути, оставшейся в трубке, равна примерно 760 мм. Над ртутью внутри трубки воздуха нет, там безвоздушное пространство, поэтому никакой газ не оказывает давления сверху на столб ртути внутри этой трубки и не влияет на измерения.

Торричелли, предложивший описанный выше опыт, дал и его объяснение. Атмосфера давит на поверхность ртути в чашке. Ртуть находится в равновесии. Значит, давление в трубке на уровне аа
1 (см. рис) равно атмосферному давлению. При изменении атмосферного давления меняется и высота столба ртути в трубке. При увеличении давления столбик удлиняется. При уменьшении давления — столб ртути уменьшает свою высоту.

Давление в трубке на уровне аа1 создается весом столба ртути в трубке, так как в верхней части трубки над ртутью воздуха нет. Отсюда следует, что атмосферное давление равно давлению столба ртути в трубке

, т. е.

p
атм = p
ртути.

Чем больше атмосферное давление, тем выше столб ртути в опыте Торричелли. Поэтому на практике атмосферное давление можно измерить высотой ртутного столба (в миллиметрах или сантиметрах). Если, например, атмосферное давление равно 780 мм рт. ст. (говорят «миллиметров ртутного столба»), то это значит, что воздух производит такое же давление, какое производит вертикальный столб ртути высотой 780 мм.

Следовательно, в этом случае за единицу измерения атмосферного давления принимается 1 миллиметр ртутного столба (1 мм рт. ст.). Найдем соотношение между этой единицей и известной нам единицей — паскалем
(Па).

Давление столба ртути ρ ртути высотой 1 мм равно:

p
= g·ρ·h
, p
= 9,8 Н/кг · 13 600 кг/ м 3 · 0,001 м ≈ 133,3 Па.

Итак, 1 мм рт. ст. = 133,3 Па.

В настоящее время атмосферное давление принято измерять в гектопаскалях (1 гПа = 100 Па). Например, в сводках погоды может быть объявлено, что давление равно 1013 гПа, это то же самое, что 760 мм рт. ст.

Наблюдая ежедневно за высотой ртутного столба в трубке, Торричелли обнаружил, что эта высота меняется, т. е. атмосферное давление непостоянно, оно может увеличиваться и уменьшаться. Торричелли заметил также, что атмосферное давление связано с изменением погоды.

Если к трубке с ртутью, использовавшейся в опыте Торричелли, прикрепить вертикальную шкалу, то получится простейший прибор — ртутный барометр

(от греч. барос
— тяжесть, метрео
— измеряю). Он служит для измерения атмосферного давления.

Барометр — анероид.

В практике для измерения атмосферного давления используют металлический барометр, называемый анероидом

(в переводе с греческого — безжидкостный
). Так барометр называют потому, что в нем нет ртути.

Внешний вид анероида изображен на рисунке. Главная часть его — металлическая коробочка 1 с волнистой (гофрированной) поверхностью (см. др. рис.). Из этой коробочки выкачан воздух, а чтобы атмосферное давление не раздавило коробочку, ее крышка 2 пружиной оттягивается вверх. При увеличении атмосферного давления крышка прогибается вниз и натягивает пружину. При уменьшении давления пружина выпрямляет крышку. К пружине с помощью передаточного механизма 3 прикреплена стрелка-указатель 4, которая продвигается вправо или влево при изменении давления. Под стрелкой укреплена шкала, деления которой нанесены по показаниям ртутного барометра. Так, число 750, против которого стоит стрелка анероида (см. рис.), показывает, что в данный момент в ртутном барометре высота ртутного столба 750 мм.

Следовательно, атмосферное давление равно 750 мм рт. ст. или ≈ 1000 гПа.

Значение атмосферного давления весьма важно для предвидения погоды на ближайшие дни, так как изменение атмосферного давления связано с изменением погоды. Барометр — необходимый прибор для метеорологических наблюдений.

Атмосферное давление на различных высотах.

В жидкости давление, как мы знаем, зависит от плотности жидкости и высоты ее столба. Вследствие малой сжимаемости плотность жидкости на различных глубинах почти одинакова. Поэтому, вычисляя давление, мы считаем ее плотность постоянной и учитываем только изменение высоты.

Сложнее дело обстоит с газами. Газы сильно сжимаемы. А чем сильнее газ сжат, тем больше его плотность, и тем большее давление он производит. Ведь давление газа создается ударами его молекул о поверхность тела.

Слои воздуха у поверхности Земли сжаты всеми вышележащими слоями воздуха, находящимися над ними. Но чем выше от поверхности слой воздуха, тем слабее он сжат, тем меньше его плотность. Следовательно, тем меньшее давление он производит. Если, например, воздушный шар поднимается над поверхностью Земли, то давление воздуха на шар становиться меньше. Это происходит не только потому, что высота столба воздуха над ним уменьшается, но еще и потому, что уменьшается плотность воздуха. Вверху она меньше, чем внизу. Поэтому зависимость давления воздуха от высоты сложнее, чем жидкости.

Наблюдения показывают, что атмосферное давление в местностях, лежащих на уровне моря, в среднем равно 760 мм рт. ст.

Атмосферное давление, равное давлению столба ртути высотой 760 мм при температуре 0 °С, называется нормальным атмосферным давлением
.

Нормальное атмосферное давление
равно 101 300 Па = 1013 гПа.

Чем больше высота над уровнем моря, тем давление меньше.

При небольших подъемах, в среднем, на каждые 12 м подъема давление уменьшается на 1 мм рт. ст. (или на 1,33 гПа).

Зная зависимость давления от высоты, можно по изменению показаний барометра определить высоту над уровнем моря. Анероиды, имеющие шкалу, по которой непосредственно можно измерить высоту над уровнем моря, называются высотомерами

. Их применяют в авиации и при подъеме на горы.

Манометры.

Мы уже знаем, что для измерения атмосферного давления применяют барометры. Для измерения давлений, бóльших или меньших атмосферного, используется манометры

(от греч. манос
— редкий, неплотный, метрео
— измеряю). Манометры бывают жидкостные
и металлические
.

Рассмотрим сначала устройство и действие открытого жидкостного манометра
. Он состоит из двухколенной стеклянной трубки, в которую наливается какая-нибудь жидкость. Жидкость устанавливается в обоих коленах на одном уровне, так как на ее поверхность в коленах сосуда действует только атмосферное давление.

Чтобы понять, как работает такой манометр, его можно соединить резиновой трубкой с круглой плоской коробкой, одна сторона которой затянута резиновой пленкой. Если надавить пальцем на пленку, то уровень жидкости в колене манометра, соединенном в коробкой, понизится, а в другом колене повысится. Чем это объясняется?

При надавливании на пленку увеличивается давление воздуха в коробке. По закону Паскаля это увеличение давления передается и жидкости в том колене манометра, которое присоединено к коробке. Поэтому давление на жидкость в этом колене будет больше, чем в другом, где на жидкость действует только атмосферное давление. Под действием силы этого избыточного давления жидкость начнет перемещаться. В колене со сжатым воздухом жидкость опустится, в другом — поднимется. Жидкость придет в равновесие (остановится), когда избыточное давление сжатого воздуха уравновесится давлением, которое производит избыточный столб жидкости в другом колене манометра.

Чем сильнее давить на пленку, тем выше избыточный столб жидкости, тем больше его давление. Следовательно, об изменении давления можно судить по высоте этого избыточного столба
.

На рисунке показано, как таким манометром можно измерять давление внутри жидкости. Чем глубже погружается в жидкость трубочка, тем больше становится разность высот столбов жидкости в коленах манометра
, тем, следовательно, и большее давление производит жидкость
.

Если установить коробочку прибора на какой-нибудь глубине внутри жидкости и поворачивать ее пленкой вверх, вбок и вниз, то показания манометра при этом не будут меняется. Так и должно быть, ведь на одном и том же уровне внутри жидкости давление одинаково по всем направлениям
.

На рисунке изображен металлический манометр

. Основная часть такого манометра — согнутая в трубу металлическая трубка 1
, один конец которой закрыт. Другой конец трубки с помощью крана 4
сообщается с сосудом, в котором измеряют давление. При увеличении давления трубка разгибается. Движение её закрытого конца при помощи рычага 5
и зубчатки 3
передается стрелке 2
, движущейся около шкалы прибора. При уменьшении давления трубка, благодаря своей упругости, возвращается в прежнее положение, а стрелка — к нулевому делению шкалы.

Поршневой жидкостный насос.

В опыте, рассмотренном нами ранее (§ 40), было установлено, что вода в стеклянной трубке под действием атмосферного давления поднималась вверх за поршнем. На этом основано действие поршневых
насосов.

Насос схематически изображен на рисунке. Он состоит из цилиндра, внутри которого ходит вверх и вниз, плотно прилегая к стенкам сосуда, поршень 1
. В нижней части цилиндра и в самом поршне установлены клапаны 2
, открывающиеся только вверх. При движении поршня вверх вода под действием атмосферного давления входит в трубу, поднимает нижний клапан и движется за поршнем.

При движении поршня вниз вода, находящаяся под поршнем, давит на нижний клапан, и он закрывается. Одновременно под давлением воды открывается клапан внутри поршня, и вода переходит в пространство над поршнем. При следующем движении поршня вверх в месте с ним поднимается и находящаяся над ним вода, которая и выливается в отводящую трубу. Одновременно за поршнем поднимается и новая порция воды, которая при последующем опускании поршня окажется над ним, и вся эта процедура повторяется вновь и вновь, пока работает насос.

Гидравлический пресс.

Закон Паскаля позволяет объяснить действие гидравлической машины

(от греч. гидравликос
— водяной). Это машины, действие которых основано на законах движения и равновесия жидкостей.

Основной частью гидравлической машины служат два цилиндра разного диаметра, снабженные поршнями и соединительной трубкой. Пространство под поршнями и трубку заполняют жидкостью (обычно минеральным маслом). Высоты столбов жидкости в обоих цилиндрах одинаковы, пока на поршни не действуют силы.

Допустим теперь, что силы F
1 и F
2 — силы, действующие на поршни, S
1 и S
2 — площади поршней. Давление под первым (малым) поршнем равно p
1 = F
1 / S
1 , а под вторым (большим) p
2 = F
2 / S
2 . По закону Паскаля давление покоящейся жидкостью во все стороны передается одинаково, т. е. p
1 = p
2 или F
1 / S
1 = F
2 / S
2 , откуда:

F
2 / F
1 = S
2 / S
1 .

Следовательно, сила F
2 во столько раз больше силы
F
1 , во сколько раз площадь большого поршня больше площади малого поршня
. Например, если площадь большого поршня 500 см 2 , а малого 5 см 2 , и на малый поршень действует сила 100 Н, то на больший поршень будет действовать сила, в 100 раз бóльшая, то есть 10 000 Н.

Таким образом, с помощью гидравлической машины можно малой силой уравновесить бóльшую силу.

Отношение F
1 / F
2 показывает выигрыш в силе. Например, в приведенном примере выигрыш в силе равен 10 000 Н / 100 Н = 100.

Гидравлическая машина, служащая для прессования (сдавливания), называется гидравлическим прессом

.

Гидравлические прессы применяются там, где требуется большая сила. Например, для выжимания масла из семян на маслобойных заводах, для прессования фанеры, картона, сена. На металлургических заводах гидравлические прессы используют для изготовления стальных валов машин, железнодорожных колес и многих других изделий. Современные гидравлические прессы могут развивать силу в десятки и сотни миллионов ньютонов.

Устройство гидравлического пресса схематически показано на рисунке. Прессуемое тело 1 (A) кладут на платформу, соединенную с большим поршнем 2 (B). При помощи малого поршня 3 (D) создается большое давление на жидкость. Это давление передается в каждую точку жидкости, заполняющей цилиндры. Поэтому такое же давление действует и на второй, большой поршень. Но так как площадь 2-го (большого) поршня больше площади малого, то и сила, действующая на него, будет больше силы, действующей на поршень 3 (D). Под действием этой силы поршень 2 (B) будет подниматься. При подъеме поршня 2 (B) тело (A) упирается в неподвижную верхнюю платформу и сжимается. При помощи манометра 4 (M) измеряется давление жидкости. Предохранительный клапан 5 (P) автоматически открывается, когда давление жидкости превышает допустимое значение.

Из малого цилиндра в большой жидкость перекачивается повторными движениями малого поршня 3 (D). Это осуществляется следующим образом. При подъеме малого поршня (D) клапан 6 (K) открывается, и в пространство, находящееся под поршнем, засасывается жидкость. При опускании малого поршня под действием давления жидкости клапан 6 (K) закрывается, а клапан 7 (K») открывается, и жидкость переходит в большой сосуд.

Действие воды и газа на погруженное в них тело.

Под водой мы легко можем поднять камень, который с трудом поднимается в воздухе. Если погрузить пробку под воду и выпустить ее из рук, то она всплывет. Как можно объяснить эти явления?

Мы знаем (§ 38), что жидкость давит на дно и стенки сосуда. И если внутрь жидкости поместить какое-нибудь твердое тело, то оно также будет подвергаться давлению, как и стенки сосуда.

Рассмотрим силы, которые действуют со стороны жидкости на погруженное в нее тело. Чтобы легче было рассуждать, выберем тело, которое имеет форму параллелепипеда с основаниями, параллельными поверхности жидкости (рис.). Силы, действующие на боковые грани тела, попарно равны и уравновешивают друг друга. Под действием этих сил тело сжимается. А вот силы, действующие на верхнюю и нижнюю грани тела, неодинаковы. На верхнюю грань давит сверху силой F
1 столб жидкости высотой h
1 . На уровне нижней грани давление производит столб жидкости высотой h
2 . Это давление, как мы знаем (§ 37), передается внутри жидкости во все стороны. Следовательно, на нижнюю грань тела снизу вверх с силой F
2 давит столб жидкости высотой h
2 . Но h
2 больше h
1 , следовательно, и модуль силы F
2 больше модуля силы F
1 . Поэтому тело выталкивается из жидкости с силой F
выт, равной разности сил F
2 — F
1 , т. е.

Но S·h = V, где V — объем параллелепипеда, а ρ ж ·V = m ж — масса жидкости в объеме параллелепипеда. Следовательно,

F выт = g·m ж = P ж,

т. е. выталкивающая сила равна весу жидкости в объеме погруженного в нее тела
(выталкивающая сила равна весу жидкости такого же объёма, как и объём погруженного в нее тела).

Существование силы, выталкивающей тело из жидкости, легко обнаружить на опыте.

На рисунке а
изображено тело, подвешенное к пружине со стрелкой-указателем на конце. Стрелка отмечает на штативе растяжение пружины. При отпускании тела в воду пружина сокращается (рис., б
). Такое же сокращение пружины получится, если действовать на тело снизу вверх с некоторой силой, например, нажать рукой (приподнять).

Следовательно, опыт подтверждает, что на тело, находящееся в жидкости, действует сила, выталкивающая это тело из жидкости
.

К газам, как мы знаем, также применим закон Паскаля. Поэтому на тела, находящиеся в газе, действует сила, выталкивающая их из газа
. Под действием этой силы воздушные шары поднимаются вверх. Существование силы, выталкивающей тело из газа, можно также наблюдать на опыте.

К укороченной чашке весов подвесим стеклянный шар или большую колбу, закрытую пробкой. Весы уравновешиваются. Затем под колбу (или шар) ставят широкий сосуд так, чтобы он окружал всю колбу. Сосуд наполняется углекислым газом, плотность которого больше плотности воздуха (поэтому углекислый газ опускается вниз и заполняет сосуд, вытесняя из него воздух). При этом равновесие весов нарушается. Чашка с подвешенной колбой поднимается вверх (рис.). На колбу, погруженную в углекислый газ, действует бóльшая выталкивающая сила, по сравнению с той, которая действует на нее в воздухе.

Сила, выталкивающая тело из жидкости или газа, направлена противоположно силе тяжести, приложенной к этому телу
.

Поэтому пролкосмосе). Именно этим объясняется, что в воде мы иногда легко поднимаем тела, которые с трудом удерживаем в воздухе.

К пружине подвешивается небольшое ведерко и тело цилиндрической формы (рис., а). Стрелка на штативе отмечает растяжение пружины. Она показывает вес тела в воздухе. Приподняв тело, под него подставляется отливной сосуд, наполненный жидкостью до уровня отливной трубки. После чего тело погружается целиком в жидкость (рис., б). При этом часть жидкости, объем которой равен объему тела, выливается
из отливного сосуда в стакан. Пружина сокращается, и указатель пружины поднимается вверх, показывая уменьшение веса тела в жидкости. В данном случае на тело, кроме силы тяжести, действует еще одна сила, выталкивающая его из жидкости. Если в верхнее ведерко вылить жидкость из стакана (т. е. ту, которую вытеснило тело), то указатель пружины возвратится к своему начальному положению (рис., в).

На основании этого опыта можно заключить, что сила, выталкивающая целиком погруженное в жидкость тело, равна весу жидкости в объеме этого тела

. Такой же вывод мы получили и в § 48.

Если подобный опыт проделать с телом, погруженным в какой-либо газ, то он показал бы, что сила, выталкивающая тело из газа, также равна весу газа, взятого в объеме тела

.

Сила, выталкивающая тело из жидкости или газа, называется архимедовой силой
, в честь ученого Архимеда

, который впервые указал на ее существование и рассчитал ее значение.

Итак, опыт подтвердил, что архимедова (или выталкивающая) сила равна весу жидкости в объеме тела, т. е. F
А = P
ж = g·m
ж. Массу жидкости m ж, вытесняемую телом, можно выразить через ее плотность ρ ж и объем тела V т, погруженного в жидкость (так как V ж — объем вытесненной телом жидкости равен V т — объему тела, погруженного в жидкость), т. е. m ж = ρ ж ·V т. Тогда получим:

F
A = g·ρ
ж ·V
т

Следовательно, архимедова сила зависит от плотности жидкости, в которую погружено тело, и от объема этого тела. Но она не зависит, например, от плотности вещества тела, погружаемого в жидкость, так как эта величина не входит в полученную формулу.

Определим теперь вес тела, погруженного в жидкость (или в газ). Так как две силы, действующие на тело в этом случае, направлены в противоположные стороны (сила тяжести вниз, а архимедова сила вверх), то вес тела в жидкости P 1 будет меньше веса тела в вакууме P = g·m
на архимедову силу F
А = g·m
ж (где m
ж — масса жидкости или газа, вытесненной телом).

Таким образом, если тело погружено в жидкость или газ, то оно теряет в своем весе столько, сколько весит вытесненная им жидкость или газ
.

Пример
. Определить выталкивающую силу, действующую на камень объемом 1,6 м 3 в морской воде.

Запишем условие задачи и решим ее.

Когда всплывающее тело достигнет поверхности жидкости, то при дальнейшем его движении вверх архимедова сила будет уменьшаться. Почему? А потому, что будет уменьшаться объем части тела, погруженной в жидкость, а архимедова сила равна весу жидкости в объеме погруженной в нее части тела.

Когда архимедова сила станет равной силе тяжести, тело остановится и будет плавать на поверхности жидкости, частично погрузившись в нее.

Полученный вывод легко проверить на опыте.

В отливной сосуд нальем воду до уровня отливной трубки. После этого погрузим в сосуд плавающее тело, предварительно взвесив его в воздухе. Опустившись в воду, тело вытесняет объем воды, равный объему погруженной в нее части тела. Взвесив эту воду, находим, что ее вес (архимедова сила) равен силе тяжести, действующей на плавающее тело, или весу этого тела в воздухе.

Проделав такие же опыты с любыми другими телами, плавающими в разных жидкостях — в воде, спирте, растворе соли, можно убедиться, что если тело плавает в жидкости, то вес вытесненной им жидкости равен весу этого тела в воздухе
.

Легко доказать, что если плотность сплошного твердого тела больше плотности жидкости, то тело в такой жидкости тонет. Тело с меньшей плотностью всплывает в этой жидкости
. Кусок железа, например, тонет в воде, но всплывает в ртути. Тело же, плотность которого равна плотности жидкости, остается в равновесии внутри жидкости.

Плавает на поверхности воды лед, так как его плотность меньше плотности воды.

Чем меньше плотность тела по сравнению с плотностью жидкости, тем меньшая часть тела погружена в жидкость

.

При равных плотностях тела и жидкости тело плавает внутри жидкости на любой глубине.

Две несмешивающиеся жидкости, например вода и керосин, располагаются в сосуде в соответствии со своими плотностями: в нижней части сосуда — более плотная вода (ρ = 1000 кг/м 3), сверху — более легкий керосин (ρ = 800 кг/м 3).

Средняя плотность живых организмов, населяющих водную среду, мало отличается от плотности воды, поэтому их вес почти полностью уравновешивается архимедовой силой. Благодаря этому водные животные не нуждаются в столь прочных и массивных скелетах, как наземные. По этой же причине эластичны стволы водных растений.

Плавательный пузырь рыбы легко меняет свой объем. Когда рыба с помощью мышц опускается на большую глубину, и давление воды на нее увеличивается, пузырь сжимается, объем тела рыбы уменьшается, и она не выталкивается вверх, а плавает в глубине. Таким образом, рыба может в определенных пределах регулировать глубину своего погружения. Киты регулируют глубину своего погружения за счет уменьшения и увеличения объема легких.

Плавание судов.

Суда, плавающие по рекам, озерам, морям и океанам, построены из разных материалов с различной плотностью. Корпус судов обычно делается из стальных листов. Все внутренние крепления, придающие судам прочность, также изготовляют из металлов. Для постройки судов используют различные материалы, имеющие по сравнению с водой как бóльшие, так и меньшие плотности.

Благодаря чему суда держатся на воде, принимают на борт и перевозят большие грузы?

Опыт с плавающим телом (§ 50) показал, что тело вытесняет своей подводной частью столько воды, что по весу эта вода равна весу тела в воздухе. Это также справедливо и для любого судна.

Вес воды, вытесняемой подводной частью судна, равен весу судна с грузом в воздухе или силе тяжести, действующей на судно с грузом
.

Глубина, на которую судно погружается в воду, называется осадкой

. Наибольшая допускаемая осадка отмечена на корпусе судна красной линией, называемой ватерлинией

(от голланд. ватер
— вода).

Вес воды, вытесняемой судном при погружении до ватерлинии, равный силе тяжести, действующей на судно с грузом, называется водоизмещением судна
.

В настоящее время для перевозки нефти строятся суда водоизмещением 5 000 000 кН (5 · 10 6 кН) и больше, т. е. имеющие вместе с грузом массу 500 000 т (5 · 10 5 т) и более.

Если из водоизмещения вычесть вес самого судна, то мы получим грузоподъемность этого судна. Грузоподъемность показывает вес груза, перевозимого судном.

Судостроение существовало еще в Древнем Египте, в Финикии (считается, что Финикийцы были одними из лучших судостроителей), Древнем Китае.

В России судостроение зародилось на рубеже 17-18 вв. Сооружались главным образом военные корабли, но именно в России были построены первый ледокол, суда с двигателем внутреннего сгорания, атомный ледокол «Арктика».

Воздухоплавание.

Рисунок с описанием шара братьев Монгольфье 1783 года: «Вид и точные размеры „Аэростата Земной шар“, который был первым». 1786

С давних времен люди мечтали о возможности летать над облаками, плавать в воздушном океане, как они плавали по морю. Для воздухоплавания

вначале использовали воздушные шары, которые наполняли или нагретым воздухом, или водородом либо гелием.

Для того, чтобы воздушный шар поднялся в воздух, необходимо, чтобы архимедова сила (выталкивающая) F
А, действующая на шар, была больше силы тяжести F
тяж, т. е. F
А > F
тяж.

По мере поднятия шара вверх архимедова сила, действующая на него, уменьшается (F
А = gρV
), так как плотность верхних слоев атмосферы меньше, чем у поверхности Земли. Чтобы подняться выше, с шара сбрасывается специальный балласт (груз) и этим облегчает шар. В конце концов шар достигает своей своей предельной высоты подъема. Для спуска шара из его оболочки при помощи специального клапана выпускается часть газа.

В горизонтальном направлении воздушный шар перемещается только под действием ветра, поэтому он называется аэростатом

(от греч аэр
— воздух, стато
— стоящий). Для исследования верхних слоев атмосферы, стратосферы еще не так давно применялись огромные воздушные шары — стратостаты

.

До того как научились строить большие самолеты для перевозки по воздуху пассажиров и грузов, применялись управляемые аэростаты — дирижабли
. Они имеют удлиненную форму, под корпусом подвешивается гондола с двигателем, который приводит в движение пропеллер.

Воздушный шар не только сам поднимается вверх, но может поднять и некоторый груз: кабину, людей, приборы. Поэтому для того, чтобы узнать, какой груз может поднять воздушный шар, необходимо определить его подъемную силу
.

Пусть, например, в воздух запущен шар объемом 40 м 3 , наполненный гелием. Масса гелия, заполняющая оболочку шара, будет равна:
m Ге = ρ Ге ·V = 0,1890 кг/м 3 · 40 м 3 = 7,2 кг,
а его вес равен:
P Ге = g·m Ге; P Ге = 9,8 Н/кг · 7,2 кг = 71 Н.
Выталкивающая же сила (архимедова), действующая на этот шар в воздухе, равна весу воздуха объемом 40 м 3 , т. е.
F А = g·ρ возд V; F А = 9,8 Н/кг · 1,3 кг/м 3 · 40 м 3 = 520 Н.

Значит, этот шар может поднять груз весом 520 Н — 71 Н = 449 Н. Это и есть его подъемная сила.

Шар такого же объема, но наполненный водородом, может поднять груз 479 Н. Значит, подъемная сила его больше, чем шара, наполненного гелием. Но все же чаще используют гелий, так как он не горит и поэтому безопаснее. Водород же горючий газ.

Гораздо проще осуществить подъем и спуск шара, наполненного горячим воздухом. Для этого под отверстием, находящимся в нижней части шара, располагается горелка. При помощи газовой горелки можно регулировать температуру воздуха внутри шара, а значит, его плотность и выталкивающую силу. Чтобы шар поднялся выше, достаточно сильнее нагреть воздух в нем, увеличив пламя горелки. При уменьшении пламени горелки температура воздуха в шаре уменьшается, и шар опускается вниз.

Можно подобрать такую температуру шара, при которой вес шара и кабины будет равен выталкивающей силе. Тогда шар повиснет в воздухе, и с него будет легко проводить наблюдения.

По мере развития науки происходили и существенные изменения в воздухоплавательной технике. Появилась возможность использования новых оболочек для аэростатов, которые стали прочными, морозоустойчивыми и легкими.

Достижения в области радиотехники, электроники, автоматики позволили сконструировать беспилотные аэростаты. Эти аэростаты используются для изучения воздушных течений, для географических и медико-биологических исследований в нижних слоях атмосферы.

Выпуск 16

В видеоуроке физики от Академии занимательных наук профессор Даниил Эдисонович познакомит юных телезрителей с новой физической величиной, которая служит для измерения давления — Паскалем. Посмотрев передачу вы узнаете, какое значение имеет площадь опоры твёрдого тела, как не провалиться под лёд или снег, а также познакомитесь с формулой давления твёрдых тел.

Формула давления твёрдых тел

Как вы наверное помните из прошлой передачи, вес — это сила, с которой тело давит на опору. Почему же один и тот же человек, идя по снегу в ботинках проваливается, а идя на лыжах — нет? Чтобы разобраться в этом вопросе профессор Даниил Эдисонович научит вас формуле давления твёрдых тел. Трактор весит гораздо больше автомобиля, а в рыхлой почве не вязнет. В то же время лёгкий автомобиль попав на такую почву скорее всего застрянет и его придётся вытаскивать трактором. Результат действия силы на поверхность зависит не только от величины этой силы, но и от площади, к которой приложена эта сила. Когда человек наступает в снег, вес его тела распределяется по площади его ступней. А если человек обут в лыжи, то вес распределяется по их площади, которая намного больше площади ступней. Поскольку площадь приложения стала больше, человек не провалится в снег. Давление — это скалярная физическая величина, равная отношению силы давления, приложенной к данной поверхности, к площади этой поверхности. Чтобы определить давление, надо силу, действующую перпендикулярно поверхности, разделить на площадь этой поверхности. Формула давления твёрдых тел записывается так: p=F/S, где р — это давление, F — сила давления, S — площадь опоры. За единицу давления принимается давление, которое производит сила в 1 ньютон, действующая на поверхность площадью 1м2 перпендикулярно этой поверхности. Измеряется давление в паскалях. Таким образом, по формуле давления твёрдых тел, 1 паскаль равен 1 ньютону на квадратный метр. Между силой давления и давлением существует прямо пропорциональная зависимость, то есть чем больше сила, тем больше давление и наоборот, чем меньше сила, тем меньше давление. Если говорить о зависимости давления от площади опоры, то здесь наблюдается обратно пропорциональная зависимость, то есть чем больше площадь опоры, тем меньше давление и наоборот, чем меньше площадь соприкосновения тел, тем давление больше. Величина давления имеет большое значение не только в жизни человека, но и в жизни животных. Например, заяц, оказывающий давление в 1,2 кПа может сравнительно легко убежать от волка, который оказывает давление 12 кПа, по рыхлому снегу, но не спасется от него на твердой почве.

Давление — это физическая величина, которая играет особую роль в природе и жизни человека. Это незаметное глазу явление не только влияет на состояние окружающей среды, но и очень хорошо ощущается всеми. Давайте разберемся, что это такое, какие виды его существуют и как находить давление (формула) в разных средах.

Что называется давлением в физике и химии

Данным термином именуется важная термодинамическая величина, которая выражается в соотношении перпендикулярно оказываемой силы давления на площадь поверхности, на которую она воздействует. Это явление не зависит от размера системы, в которой действует, поэтому относится к интенсивным величинам.

В состоянии равновесия, по давление одинаково для всех точек системы.

В физике и химии оное обозначается с помощью буквы «Р», что является сокращением от латинского названия термина — pressūra.

Если речь идет об осмотическом давлении жидкости (равновесие между давлением внутри и снаружи клетки), используется буква «П».

Единицы давления

Согласно стандартам Международной системы СИ, рассматриваемое физическое явление измеряется в паскалях (кириллицей — Па, латиницей — Ра).

Исходя из формулы давления получается, что один Па равен одному Н (ньютон — разделенному на один квадратный метр (единица измерения площади).

Однако на практике применять паскали довольно сложно, поскольку эта единица очень мала. В связи с этим, помимо стандартов системы СИ, данная величина может измеряться по-другому.

Ниже приведены наиболее известные ее аналоги. Большинство из них широко используется на просторах бывшего СССР.

  • Бары
    . Один бар равен 105 Па.
  • Торры, или миллиметры ртутного столба.
    Приблизительно один торр соответствует 133, 3223684 Па.
  • Миллиметры водяного столба.
  • Метры водяного столба.
  • Технические атмосферы.
  • Физические атмосферы.
    Одна атм равна 101 325 Па и 1,033233 ат.
  • Килограмм-силы на квадратный сантиметр.
    Также выделяются тонна-сила и грамм-сила. Помимо этого, есть аналог фунт-сила на квадратный дюйм.

Общая формула давления (физика 7-го класса)

Из определения данной физической величины можно определить способ ее нахождения. Выглядит он таким образом, как на фото ниже.

В нем F — это сила, а S — площадь. Иными словами, формула нахождения давления — это его сила, разделенная на площадь поверхности, на которую оно воздействует.

Также она может быть записана так: Р = mg / S или Р = pVg / S. Таким образом, эта физическая величина оказывается связанной с другими термодинамическими переменными: объемом и массой.

Для давления действует следующий принцип: чем меньше пространство, на которое влияет сила — тем большее количество давящей силы на него приходится. Если, же площадь увеличивается (при той же силе) — искомая величина уменьшается.

Формула гидростатического давления

Разные агрегатные состояния веществ, предусматривают наличие у них отличных друг от друга свойств. Исходя из этого, способы определения Р в них тоже будут другими.

К примеру, формула давления воды (гидростатического) выглядит вот так: Р = pgh. Также она применима и к газам. При этом ее нельзя использовать для вычисления атмосферного давления, из-за разности высот и плотностей воздуха.

В данной формуле р — плотность, g — ускорение свободного падения, а h — высота. Исходя из этого, чем глубже погружается предмет или объект, тем выше оказываемое на него давление внутри жидкости (газа).

Рассматриваемый вариант является адаптацией классической примера Р = F / S.

Если вспомнить, что сила равна производной массы на скорость свободного падения (F= mg), а масса жидкости — это производная объема на плотность (m = pV), то формулу давление можно записать как P = pVg / S. При этом объем — это площад, умноженная на высоту (V = Sh).

Если вставить эти данные, получится, что площадь в числителе и знаменателе можно сократить и на выходе — вышеупомянутая формула: Р = pgh.

Рассматривая давление в жидкостях, стоит помнить, что, в отличие от твердых тел, в них часто возможно искривление поверхностного слоя. А это, в свою очередь, способствует образованию дополнительного давления.

Для подобных ситуаций применяется несколько другая формула давления: Р = Р 0 + 2QH. В данном случае Р 0 — давление не искривленного слоя, а Q — поверхность натяжения жидкости. Н — это средняя кривизна поверхности, которую определяют по Закону Лапласа: Н = ½ (1/R 1 + 1/R 2). Составляющие R 1 и R 2 — это радиусы главной кривизны.

Парциальное давление и его формула

Хотя способ Р = pgh применим как для жидкостей, так и для газов, давление в последних лучше вычислять несколько другим путем.

Дело в том, что в природе, как правило, не очень часто встречаются абсолютно чистые вещества, ведь в ней преобладают смеси. И это касается не только жидкостей, но и газов. А как известно, каждый из таких компонентов осуществляет разное давление, называемое парциальным.

Определить его довольно просто. Оно равно сумме давления каждого компонента рассматриваемой смеси (идеальный газ).

Из этого следует, что формула парциального давления выглядит таким образом: Р = Р 1 + Р 2 + Р 3 … и так далее, согласно количеству составляющих компонентов.

Нередки случаи, когда необходимо определить давление воздуха. Однако некоторые по ошибке проводят вычисления только с кислородом по схеме Р = pgh. Вот только воздух — это смесь из разных газов. В нем встречаются азот, аргон, кислород и другие вещества. Исходя из сложившейся ситуации, формула давления воздуха — это сумма давлений всех его составляющих. А значит, следует приметь вышеупомянутую Р = Р 1 + Р 2 + Р 3 …

Наиболее распространенные приборы для измерения давления

Несмотря на то что высчитать рассматриваемую термодинамическую величину по вышеупомянутым формулам не сложно, проводить вычисление иногда попросту нет времени. Ведь нужно всегда учитывать многочисленные нюансы. Поэтому для удобства за несколько столетий был разработан ряд приборов, делающих это вместо людей.

Фактически почти все аппараты такого рода являются разновидностями манометра (помогает определять давление в газах и жидкостях). При этом они отличаются по конструкции, точности и сфере применения.

  • Атмосферное давление измеряется с помощью манометра, именуемого барометром. Если необходимо определить разряжение (то есть давление ниже атмосферного) — применяются другая его разновидность, вакуумметр.
  • Для того чтобы узнать артериальное давление у человека, в ход идет сфигмоманометр. Большинству он более известен под именем неинвазивного тонометра. Таких аппаратов существуют немало разновидностей: от ртутных механических до полностью автоматических цифровых. Их точность зависит от материалов, из которых они изготавливаются и места измерения.
  • Перепады давления в окружающей среде (по-английски — pressure drop) определяются с помощью или дифнамометров (не путать с динамометрами).

Виды давления

Рассматривая давление, формулу его нахождения и ее вариации для разных веществ, стоит узнать о разновидностях этой величины. Их пять.

  • Абсолютное.
  • Барометрическое
  • Избыточное.
  • Вакуумметрическое.
  • Дифференциальное.

Абсолютное

Так называется полное давление, под которым находится вещество или объект, без учета влияния других газообразных составляющих атмосферы.

Измеряется оно в паскалях и являет собою сумму избыточного и атмосферного давлений. Также он является разностью барометрического и вакуумметрического видов.

Вычисляется оно по формуле Р = Р 2 + Р 3 или Р = Р 2 — Р 4 .

За начало отсчета для абсолютного давления в условиях планеты Земля, берется давление внутри емкости, из которой удален воздух (то есть классический вакуум).

Только такой вид давления используется в большинстве термодинамических формул.

Барометрическое

Этим термином именуется давление атмосферы (гравитации) на все предметы и объекты, находящие в ней, включая непосредственно поверхность Земли. Большинству оно также известно под именем атмосферного.

Его причисляют к а его величина меняется относительно места и времени измерения, а также погодных условий и нахождения над/ниже уровня моря.

Величина барометрического давления равна модулю силы атмосферы на площади единицу по нормали к ней.

В стабильной атмосфере величина данного физического явления равна весу столпа воздуха на основание с площадью, равной единице.

Норма барометрического давления — 101 325 Па (760 мм рт. ст. при 0 градусов Цельсия). При этом чем выше объект оказывается от поверхности Земли, тем более низким становится давление на него воздуха. Через каждый 8 км оно снижается на 100 Па.

Благодаря этому свойству в горах вода в чайниках закивает намного быстрее, чем дома на плите. Дело в том, что давление влияет на температуру кипения: с его снижением последняя уменьшается. И наоборот. На этом свойстве построена работа таких кухонных приборов, как скороварка и автоклав. Повышение давления внутри их способствуют формированию в посудинах более высоких температур, нежели в обычных кастрюлях на плите.

Используется для вычисления атмосферного давления формула барометрической высоты. Выглядит она таким образом, как на фото ниже.

Р — это искомая величина на высоте, Р 0 — плотность воздуха возле поверхности, g — свободного падения ускорение, h — высота над Землей, м — молярная масса газа, т — температура системы, r — универсальная газовая постоянная 8,3144598 Дж⁄(моль х К), а е — это число Эйклера, равное 2.71828.

Часто в представленной выше формуле давления атмосферного вместо R используется К — постоянная Больцмана. Через ее произведение на число Авогадро нередко выражается универсальная газовая постоянная. Она более удобна для расчетов, когда число частиц задано в молях.

При проведении вычислений всегда стоит брать во внимание возможность изменения температуры воздуха из-за смены метеорологической ситуации или при наборе высоты над уровнем моря, а также географическую широту.

Избыточное и вакуумметрическое

Разницу между атмосферным и измеренным давлением окружающей среды называют избыточным давлением. В зависимости от результата, меняется название величины.

Если она положительная, ее называют манометрическим давлением.

Если же полученный результат со знаком минус — его именуют вакуумметрическим. Стоит помнить, что он не может быть больше барометрического.

Дифференциальное

Данная величина является разницей давлений в различных точках измерения. Как правило, ее используют для определения падения давления на каком-либо оборудовании. Особенно это актуально в нефтедобывающей промышленности.

Разобравшись с тем, что за термодинамическая величина называется давлением и с помощью каких формул ее находят, можно сделать вывод, что это явление весьма важно, а потому знания о нем никогда не будут лишними.

Давление жидкости

Давление определяется как сила на единицу площади:

\ [P = \ frac {F} {A}. \]

Если объект погружен в жидкость на глубину \ (h \), давление жидкости определяется формулой постоянной глубины

\ [P = \ rho gh, \]

где \ (\ rho \) — плотность жидкости, а \ (g \) — ускорение свободного падения.

Давление жидкости — это скалярная величина. У него нет направления, поэтому жидкость оказывает давление одинаково во всех направлениях.Это утверждение известно как закон Паскаля, открытый французским ученым Блезом Паскалем (\ (1623 — 1662 \)). b {\ left [{f \ left (x \ right) — g \ left (x \ right)} \ right] xdx}.3}}}. \)

Пример 2

Прямоугольный бассейн имеет глубину \ (H \) метров, ширину \ (a \) и длину \ (b \) метров. Рассчитать

  1. Сила жидкости \ (F_ {ab} \), действующая на дно бассейна;
  2. Сила жидкости \ (F_ {aH} \), действующая на каждую \ (\ left ({a \ times H} \ right) \ text {m} \) сторону;
  3. Сила жидкости \ (F_ {bH} \), действующая на каждую \ (\ left ({b \ times H} \ right) \ text {m} \) сторону;

Пример 3

Треугольная пластина с основанием \ (a \) и высотой \ (H \) погружается в воду вертикально, так что ее основание лежит на поверхности воды.Найдите гидростатическую силу, действующую с каждой стороны пластины.

Пример 4

Куб со стороной \ (a \) погружен в воду так, что его верхняя грань параллельна поверхности воды и на расстоянии \ (H \) метров под ней. Найдите полную гидростатическую силу, действующую на куб. 3}}}.\)

Решение.

Рис. 2.

Выберем ось \ (x — \), направленную вертикально вниз с началом координат в верхнем основании резервуара.

Рассмотрим тонкий слой на глубине \ (x. \). Если его толщина равна \ (dx, \), площадь боковой поверхности слоя равна

\ [dA = 2 \ pi Rdx. \]

Давление жидкости на глубине \ (x \) равно \ (P = \ rho gx, \), поэтому сила, прилагаемая жидкостью к боковой поверхности, равна

\ [dF = PdA = 2 \ pi \ rho gRxdx.2} \ приблизительно 221671 \, \ text {N} \ приблизительно 222 \, \ text {kN}. \]

Пример 2.

Прямоугольный бассейн имеет глубину \ (H \) метров, ширину \ (a \) и длину \ (b \) метров. Рассчитать

  1. Сила жидкости \ (F_ {ab} \), действующая на дно бассейна;
  2. Сила жидкости \ (F_ {aH} \), действующая на каждую \ (\ left ({a \ times H} \ right) \ text {m} \) сторону;
  3. Сила жидкости \ (F_ {bH} \), действующая на каждую \ (\ left ({b \ times H} \ right) \ text {m} \) сторону;

Решение.

Рис. 3.

  1. Давление на дне бассейна равно \ (P = \ rho gH, \), поэтому гидростатическая сила, действующая на дно, определяется выражением

    \ [{F_ {ab}} = PA = \ rho gHA = \ rho gabH. \]

  2. Чтобы определить силу на \ (\ left ({a \ times H} \ right) \ text {m} \) стороне бассейна, берем тонкую полосу толщиной \ (dx \) на глубине \ ( Икс.\)

    Рисунок 4.

    Площадь полосы равна \ (dA = adx. \). Поскольку давление воды на глубине \ (x \) равно \ (P = \ rho gx, \), сила, действующая на элементарную полосу, равна

    \ [dF = PdA = \ rho gaxdx.2}}} {2}. \]

Пример 3.

Треугольная пластина с основанием \ (a \) и высотой \ (H \) погружается в воду вертикально, так что ее основание лежит на поверхности воды. Найдите гидростатическую силу, действующую с каждой стороны пластины.

Решение.

Рисунок 5.

Из подобных треугольников имеем

\ [\ frac {W} {a} = \ frac {{H — x}} {H}, \; \; \ Rightarrow W = a — \ frac {a} {H} x. \]

Площадь элементарной горизонтальной полосы на глубине \ (x \) составляет

\ [dA = Wdx = \ left ({a — \ frac {a} {H} x} \ right) dx.2} \ left ({2H + a} \ right). \]

См. Другие проблемы на странице 2.

Давление

Каждый человек в то или иное время находился под давлением или в определенных обстоятельствах действительно «чувствовал давление». Однако с научной точки зрения давление имеет очень конкретное определение, и его исследование приводит к некоторым очень важным приложениям.

В физике давление — это эффект силы, действующей на поверхность.Математически это скалярная величина, рассчитываемая как сила, приложенная на единицу площади, где приложенная сила всегда перпендикулярна поверхности. Единица давления в системе СИ, Паскаль (Па), эквивалентна Н / м 2 .

Все состояния вещества могут оказывать давление. Когда вы идете по покрытому льдом озеру, вы оказываете давление на лед, равное силе тяжести вашего тела (вашему весу), деленной на площадь, над которой вы соприкасаетесь со льдом. Вот почему важно распределять вес при движении по хрупким поверхностям.Ваши шансы прорваться сквозь лед чрезвычайно возрастают, если вы идете по льду на высоких каблуках, так как небольшая область, соприкасающаяся со льдом, приводит к высокому давлению. Это также причина того, что у снегоступов такая большая площадь. Они разработаны для уменьшения давления на верхнюю корку снега, чтобы вам было легче ходить, не проваливаясь в снежные заносы.

Жидкости также могут оказывать давление. Все жидкости оказывают внешнее давление во всех направлениях по бокам любого контейнера, содержащего жидкость.Даже атмосфера Земли оказывает давление, которое вы испытываете прямо сейчас. Однако давление внутри и снаружи вашего тела настолько хорошо сбалансировано, что вы редко замечаете 101 325 Паскалей из-за атмосферы (примерно 10 Н / см 2 ). Если вы летите в самолете и быстро меняете высоту (и, следовательно, давление), вы могли испытать ощущение «хлопка» в ушах — это происходит из-за давления внутри уха, уравновешивающего давление снаружи уха при переносе воздуха через маленькие трубки, соединяющие внутреннее ухо с горлом.

Вопрос: Давление воздуха составляет примерно 100 000 Па. Какая сила действует на эту книгу, когда она лежит на столе? Площадь обложки книги 0,035 м 2 .

Ответ:

Вопрос: Рыбак массой 75 кг засыпает на своем четвероногом стуле массой 5 ​​кг. Если каждая ножка стула имеет площадь 2.5 × 10 -4 м 2 в контакте с землей, какое среднее давление, оказываемое рыбаком и стулом на землю?

Ответ: Приложенная сила является силой тяжести, поэтому мы можем написать:

Вопрос: На поверхности планеты Физика помещена шкала, показывающая 0 в космическом вакууме. На поверхности планеты шкала показывает силу в 10 000 ньютонов.Вычислите площадь поверхности шкалы, учитывая, что атмосферное давление на поверхности Physica составляет 80 000 Паскалей.

Ответ:

Вопрос: Оцените следующее от самого высокого давления до самого низкого давления на землю:

  • Атмосфера на уровне моря
  • Слон массой 7000 кг общей площадью 0,5 м 2 , соприкасающийся с землей
  • Дама 65 кг на высоких каблуках общей площадью 0 см.005 м 2 в контакте с землей
  • Автомобиль массой 1600 кг с общей площадью контакта шины 0,2 м 2

Ответ: От максимального давления до самого низкого давления:

  • Слон (137000 Па)
  • Дама на высоких каблуках (127000 Па)
  • Атмосфера (100000 Па)
  • Автомобиль (78400 Па)

Давление, которое жидкость оказывает на объект, погруженный в эту жидкость, можно рассчитать почти так же просто.Если объект погружен на глубину (h), давление определяется путем умножения плотности жидкости на глубину погружения, умноженную на ускорение свободного падения.

Это называется манометрическим давлением, потому что это значение, которое вы можете наблюдать на манометре. Если над жидкостью также есть атмосфера, как, например, здесь, на Земле, вы можете определить абсолютное давление или общее давление, добавив атмосферное давление (P 0 ), которое равно примерно 100000 Паскалей.

Вопрос: Саманта замечает закопанные сокровища во время подводного плавания на Карибских каникулах. Если ей придется спуститься на глубину 40 метров, чтобы проверить давление, какое манометрическое давление она увидит на своем акваланге? Плотность морской воды 1025 кг / м 3 .

Ответ:

Вопрос: Какое абсолютное давление оказывает на дайвера в предыдущей задаче вода и атмосфера?

Ответ:

Вопрос: Манометр дайвера показывает 250 000 Паскалей в пресной воде (ρ = 1000 кг / м 3 ).Насколько глубоко ныряльщик?

Ответ:

Fluid Power Formulas

Основные формулы гидравлической жидкости / Гидравлика / Пневматика

Переменная

Словесная формула с единицами

Упрощенная формула

Давление жидкости — P (фунт / кв. Дюйм) = сила (фунты) / площадь (кв.Дюймы) P = F / A
Расход жидкости — Q галлонов в минуту = расход (галлоны) / единица времени (минуты) Q = V / T
Мощность жидкости в лошадиных силах — л.с. лошадиных сил = давление (фунт / кв. Дюйм) × расход (галлонов в минуту) / 1714 л.с. = PQ / 1714

Формулы привода

Переменная

Словесная формула с единицами

Упрощенная формула

Площадь цилиндра — A (кв.В.) =? × Радиус (дюйм) 2 А =? × R 2
(кв. Дюйм) =? × Диаметр (дюйм) 2 /4 А =? × Д 2 /4
Сила цилиндра — F (Фунты) = Давление (psi) × Площадь (кв. Дюйм) F = P × A
Скорость цилиндра — v (футов / сек) = (231 × расход (галлонов в минуту)) / (12 × 60 × площадь) v = (0.3208 × галлонов в минуту) / A
Объем цилиндра — V Объем =? × Радиус 2 × Ход (дюймы) / 231 В =? × R 2 × L / 231
(L = длина хода)
Расход цилиндра — Q Объем = 12 × 60 × скорость (футы / сек) × полезная площадь (дюймы) 2 /231 Q = 3,11688 × v × A
Крутящий момент гидравлического двигателя — T Крутящий момент (дюйм.фунты) = Давление (фунт / кв. дюйм) × дисп. (дюймы 3 / об.) / 6,2822 T = P × d / 6,2822
Крутящий момент = л.с. × 63025 / об / мин T = HP × 63025 / n
Крутящий момент = расход (галлонов в минуту) × давление × 36,77 / об / мин T = 36,77 × Q × P / n
Скорость гидромотора — n Скорость (об / мин) = (231 × галл / мин) / дисп. (дюймы) 3 n = (231 × галлонов в минуту) / d
Жидкостный двигатель, л.с. HP = крутящий момент (дюйм.фунтов) × об / мин / 63025 HP = T × n / 63025

Формулы для насосов

Переменная

Словесная формула с единицами

Упрощенная формула

Расход насоса на выходе — галлонов в минуту галлонов в минуту = (Скорость (об / мин) × дисп.(куб. дюймов)) / 231 галлонов в минуту = (n × d) / 231
Входная мощность насоса, л.с. л.с. = галлонов в минуту × давление (фунт / кв. Дюйм) / 1714 × КПД л.с. = (Q × P) / 1714 × E
КПД насоса — E Общий КПД = Выходная мощность / Входная мощность E Общий = HP Out / HP In X 100
Общий КПД = Объемный КПД.× Механический эффект. E Общий = Eff Vol. × Eff мех.
Объемный КПД насоса — E Объемный КПД = Фактический выходной расход (галлонов в минуту) / Теоретический выходной расход (галлонов в минуту) × 100 Eff Vol. = Q Закон. / Q Тео. Х 100
Механический КПД насоса — E Механический КПД = Теоретический крутящий момент для привода / Фактический крутящий момент для привода × 100 Eff Mech = T Theo. / T Закон. × 100
Объем насоса — CIPR Рабочий объем (дюймы 3 / об) = расход (галлонов в минуту) × 231 / частота вращения насоса CIPR = галлонов в минуту × 231 / об / мин
Крутящий момент насоса — T Крутящий момент = Лошадиная сила × 63025 / об / мин T = 63025 × л.с. / об / мин
Крутящий момент = Давление (PSIG) × Объем насоса (CIPR) / 2? Т = П × CIPR / 6.28

Видео с вопросом: Расчет давления жидкости в точке на основе глубины точки и плотности жидкости

Стенограмма видео

Какое давление оказывает
вода на глубине 2,5 метра? Используйте значение 1000 килограммов на
кубический метр для плотности воды.

Итак, в этом примере скажем, что мы
есть столб воды. И мы заинтересованы в
давление, оказываемое этой водой на глубине 2.5 метров ниже поверхности. Итак, давайте скажем, что это точка здесь, в
наша водная толща. Давление на данный момент мы
метка создается за счет веса всей воды, находящейся выше этой точки в нашем
столб воды. И, кстати, это не делает
Разница в ширине колонны. Какой бы широкой он ни был,
Давление будет таким, пока у нас есть эта определенная глубина, 2,5 метра.

Чтобы ответить на этот вопрос о том, что
давление, оказываемое водой в этот момент, мы можем вспомнить, что давление
созданная жидкостью равна плотности этой жидкости, умноженной на ее высоту.
под поверхностью жидкости умножает на 𝑔 ускорение свободного падения.Напомним, что 𝑔 составляет 9,8 метра на
во втором квадрате, когда дело доходит до плотности нашей жидкости, мы получаем это в нашем
постановка задачи, 1000 килограммов на кубический метр. И нам также задана высота ℎ,
2,5 метра. А это значит, что мы можем сразу перейти к
вычисляя это давление. Это равно плотности
вода, умноженная на высоту под поверхностью воды, умноженную на ускорение
из-за силы тяжести.

Теперь, прежде чем мы умножим эти
числа вместе, обратите внимание на участвующие единицы. Что все единицы находятся в базовом блоке
форма. Мы видим, что в числителе нашего
единиц, у нас есть эти два фактора 𝑚, расстояние в метрах. В знаменателе мы имеем
метров в кубе. Это означает, что если бы мы умножили
всех задействованных единиц вместе, мы получим общий результат в килограммах на
метр второй в квадрате.Это эквивалентно ньютону на
метр в квадрате. И мы можем вспомнить, что ньютон на
метр в квадрате равен паскаль, который является единицей измерения давления. Это означает, что единицы, которые мы закончим
после того, как мы сделаем наш расчет, будут паскалями. Когда мы умножаем эти три
вместе мы находим результат 24500 паскалей. Это давление со стороны
вода на этой глубине.

Статическое давление

vs.Головка

Давление указывает нормальную силу на единицу площади в данной точке, действующую на данную плоскость. Поскольку в покоящейся жидкости отсутствуют касательные напряжения, давление в жидкости не зависит от направления.

Для жидкостей — жидкостей или газов — в состоянии покоя градиент давления в вертикальном направлении зависит только от удельного веса жидкости.

Как изменение давления в жидкости с подъемом может быть выражено как

Δp = — γ Δh (1)

, где

Δ p = изменение давления (Па, фунт / кв. Дюйм)

Δ h = изменение высоты (м, дюйм)

γ = удельный вес жидкости (Н / м 3 , фунт / фут 3 )

Градиент давления в вертикальное направление отрицательное — давление снижается вверх.

Удельный вес

Удельный вес жидкости можно выразить как:

γ = ρ г (2)

где

ρ = плотность жидкости (кг / м 3 , снарядов / фут 3 )

г = ускорение свободного падения (9,81 м / с 2 , 32,174 фут / с 2 )

в целом удельный вес — γ — постоянен для жидкостей.Для газов удельный вес — γ — изменяется в зависимости от высоты (и сжатия).

Давление, оказываемое статической жидкостью, зависит только от

  • глубины жидкости
  • плотности жидкости
  • ускорения свободного падения

Статического давления в жидкости

Для несжимаемой жидкости — как жидкость — перепад давления между двумя отметками может быть выражен как:

Δ p = p 2 — p 1

= — γ (h 2 — h 1 ) (3)

где

p 2 = давление на уровне 2 (Па, фунт / кв. Дюйм)

p 1 = давление (на уровне 1 Па) , psi)

h 2 = уровень 2 (м, фут)

h 1 = уровень 1 (м, фут)

(3) можно преобразовать в:

Δ p = p 1 — p 2

= 2 (h — h 1 ) (4)

или

p 1 — p 2 = γ Δ h 02 (5)

63 где

63 где

Δ h = h 2 — h 1 = разница высот — глубина вниз от местоположения h 2 до h 1 (м, фут)

или

p 1 = γ Δ h + p 2 (6)

Пример — Давление в жидкости

Абсолютное давление на глубине 10 м можно рассчитать как:

p 1 = γ Δ h + p 2

= (1000 кг / м 3 ) (9.81 м / с 2 ) (10 м) + (101,3 кПа)

= (98100 кг / мс 2 или Па) + (101300 Па)

= 199400 Па

= 199,4 кПа

где

ρ = 1000 кг / м 3

г = 9,81 м / с 2

p 2 = атмосферное давление давление = 101,3 кПа

Манометрическое давление можно рассчитать, установив p 2 = 0

p 1 = γ Δ h + p 2 + p 2

= (1000 кг / м 3 ) (9.81 м / с 2 ) (10 м)

= 98100 Па

= 98,1 кПа

Зависимость давления от напора

(6) может быть преобразована в:

Δ h = (p 2 — p 1 ) / γ (7)

Δ h86 экспресс 9 — перепад высот столба жидкости с удельным весом — γ — требуется для получения перепада давления Δp = p 2 — p 1 .

Пример — Давление в зависимости от напора

Перепад давления в 5 фунтов на кв. Дюйм ( фунтов / дюйм 2 ) эквивалентен напору в воде

(5 фунтов футов / дюйм 2 ) (12 дюймов / фут) (12 дюймов / фут) / (62,4 фунта / фут 3 )

= 11,6 футов водяного столба

или напор в Меркурии

(5 фунтов f / дюйм 2 ) (12 дюймов / фут) (12 дюймов / фут) / (847 фунтов / фут 3 )

= 0.85 футов ртути

Удельный вес воды составляет 62,4 (фунт / фут 3 ) , а удельный вес ртути составляет 847 (фунт / фут 3 ) .

Fluids

Принцип Паскаля можно сформулировать следующим образом: Давление, приложенное в одной точке замкнутой жидкости в условиях равновесия, передается одинаково на все части жидкости. Это правило используется в гидравлических системах. На рисунке 1 нажатие на цилиндрический поршень в точке a поднимает объект в точке b.

Принцип Паскаля используется для легкого подъема автомобиля.

Пусть нижние индексы a и b обозначают количества на каждом поршне. Давления равны; следовательно, P a = P b . Подставьте выражение для давления в единицах силы и площади, чтобы получить f a / A a = ( F b / A

b ) .Подставьте π r 2 вместо площади круга, упростите и решите для F b : F b = ( F a ) ( ) r b 2 / r a 2 ). Поскольку сила, прилагаемая в точке a , умножается на квадрат отношения радиусов и r b > r a , небольшое усилие на маленький поршень может поднимите относительно больший вес на поршне b .

Вода обычно обеспечивает частичную поддержку любого помещенного в нее объекта. Сила, направленная вверх, действующая на объект, помещенный в жидкость, называется выталкивающей силой . Согласно принципу Архимеда , величина выталкивающей силы на полностью или частично погруженный объект всегда равна весу жидкости, вытесняемой этим объектом.

Принцип Архимеда можно проверить нематематическим аргументом. Рассмотрим кубический объем воды в емкости с водой, показанной на рисунке 2.Этот объем находится в равновесии с действующими на него силами — весом и выталкивающей силой; следовательно, направленная вниз сила груза ( W ) должна уравновешиваться восходящей выталкивающей силой ( B ), которая обеспечивается остальной водой в контейнере.

Рисунок 2

Вес уравновешивается выталкивающей силой в объеме воды.

Если твердое тело плавает частично погруженным в жидкость, объем вытесняемой жидкости меньше объема твердого тела.Сравнение плотности твердого тела и жидкости, в которой оно плавает, приводит к интересному результату. Формулы для определения плотности: D s = m s / V s и D

l =

90 V l , где D — плотность, V — объем, м — масса, а нижние индексы s и l относятся к количествам, связанным с твердым телом и жидкость соответственно.Решение для масс приводит к m s = D s V s и m l = D l 90 л . Согласно принципу Архимеда, массы твердого тела и вытесненной жидкости равны. Поскольку веса просто равны массе, умноженной на константу (г) , массы также должны быть равны; следовательно, D s V s = D l V l или D s 902 906 л = V л V л .Теперь V = Ah , где A — это площадь поперечного сечения, а h — высота. Для твердого тела, плавающего в жидкости, A л = A s и h l — высота погруженного твердого тела, h sub . С этими заменами указанное выше соотношение становится D s / D l = h sub / h s ; следовательно, фракционная часть погруженного твердого вещества равна отношению плотности твердого вещества к плотности окружающей жидкости, в которой оно плавает.Например, около 90 процентов айсберга находится под поверхностью морской воды, потому что плотность льда составляет около девяти десятых плотности морской воды.

Представьте себе жидкость, протекающую через участок трубы с одним концом, имеющим меньшую площадь поперечного сечения, чем труба на другом конце. Течение жидкостей очень сложное; следовательно, это обсуждение будет предполагать условия плавного течения несжимаемой жидкости через стенки без сопротивления. Скорость жидкости в суженном конце должна быть больше, чем скорость на большем конце, если поддерживается устойчивый поток; то есть объем, проходящий за раз, одинаков во всех точках.Быстро движущиеся жидкости оказывают меньшее давление, чем медленно движущиеся жидкости. Уравнение Бернулли применяет закон сохранения энергии для формализации этого наблюдения: P + (1/2) ρ v 2 + ρ gh = константа. Уравнение утверждает, что сумма давления (P) , кинетической энергии на единицу объема и потенциальной энергии на единицу объема имеет одинаковое значение по всей трубе.

Что такое напор в механике жидкостей?

🕑 Время чтения: 1 минута

Напор в механике жидкости — это давление, оказываемое столбом жидкости на дно контейнера.Он представлен как высота столба жидкости. Напорная головка также называется статической головкой или статической напорной головкой , которая обозначается буквой Z.

Уравнение для определения напора жидкости выведено и объяснено ниже.

Формула напорного напора

Напор определяется по формуле:

Где,

Z = напор

p = давление жидкости (Давление на единице площади)

г = ускорение свободного падения

Определение напора (Z)

Напор в механике жидкости можно определить, рассматривая давление, действующее в любой точке на жидкость в состоянии покоя.Согласно гидростатическому закону, скорость увеличения давления в вертикальном направлении вниз должна быть равна удельному весу жидкости в этой точке.

Рассмотрим элемент жидкости ABCD, показанный на рисунке 1 ниже.

Площадь поперечного сечения жидкостного элемента — дельта (A), а высота жидкостного элемента — дельта (Z).

Рис.1.Сила, действующая на элемент жидкости

Давление на грань AB равно «p». Расстояние элемента жидкости от свободной поверхности равно «Z».

Из рисунка выше, силы, действующие на поверхности элементов, равны:

1. Сила на поверхности AB:

Сила действует перпендикулярно грани AB в направлении вниз.

2. На поверхности DC:

Эта сила действует перпендикулярно грани DC в направлении вверх.

3 . Вес элемента жидкости ABCD:

На рисунке видно, что силы, действующие на AB, равны силам, действующим на BC, и противоположны им.Следовательно, для равновесия жидкого элемента

Следовательно, уравнение-1 утверждает, что скорость увеличения давления в вертикальном направлении равна весу жидкости. Это гидростатический закон.

Путем интеграции уравнения 1,

Z — высота точки от свободной поверхности, которая образует напор. «p» — это давление выше атмосферного.

Добавить комментарий

Ваш адрес email не будет опубликован. Обязательные поля помечены *